Week 1: Insurance Midterms (Weeks 1 To 8)

You might also like

Download as docx, pdf, or txt
Download as docx, pdf, or txt
You are on page 1of 79

INSURANCE MIDTERMS (Weeks 1 to 8)

WEEK 1 XPN: Liability insurance may be required by law in certain


instances.
3. ALEATORY – the liability of the insurer depends upon some contingent
event.
GENERAL 4. UNILATERAL – it imposes legal duties only on the insurer who promises to
indemnify in case of loss.
Laws Governing contracts of insurance in the Philippines: 5. CONDITIONAL – it is subject to conditions, the principal one of which is
the happening of the event insured against.
1. RA 10607 6. CONTRACT OF INDEMNITY – recovery is commensurate with the amount
2. New Civil Code of the loss suffered.
3. Special Laws GR: The insurer promises to make good only the loss of the
insured.
Sec. 2. Whenever used in this Code, the following terms shall have the respective
meanings hereinafter set forth or indicated, unless the context otherwise requires: XPN: the principle is not applicable to life and accident insurance
where the result is death because life is not capable of pecuniary
(1) A "contract of insurance" is an agreement whereby one undertakes estimation. The only situation where the principle of indemnity is
for a consideration to indemnify another against loss, damage or liability applicable to life insurance is when the interest of a person
arising from an unknown or contingent event. insured is capable of exact pecuniary measurement.
7. PERSONAL – Each party having in view the character, credit and conduct
A contract of suretyship shall be deemed to be an insurance contract, within of the other.
the meaning of this Code, only if made by a surety who or which, as such, is doing 8. PROPERTY – since insurance is a contract, it is property in legal
an insurance business as hereinafter provided. contemplation.
9. RISK-DISTRIBUTING DEVICE – insurance serves to distribute the risk of
Requisites: economic loss among as many as possible of those who are subject to the
1. Subject Matter (Object) same kind of loss.
2. Meeting of the Minds (Consent) 10. ONEROUS – there is a valuable consideration called the premium.
3. Consideration, which is the premium paid by the insured, for the insurer’s
promise to indemnify the former upon the happening of the event or peril Art. 1318. There is no contract unless the following requisites concur:
insured against. (1) Consent of the contracting parties;
(2) Object certain which is the subject matter of the contract;
NATURE AND CHARACTERISTICS (3) Cause of the obligation which is established. 
1. CONSENSUAL – perfected by the meeting of the minds fo the parties. Art. 1319. Consent is manifested by the meeting of the offer and the acceptance
There should be acceptance of the application for insurance. upon the thing and the cause which are to constitute the contract. The offer must
2. VOLUNTARY – the parties may incorporate such terms and conditions as be certain and the acceptance absolute. A qualified acceptance constitutes a
they may deem convenient: provided they do not contravene any counter-offer.
provision of law and are not opposed to public policy, law, morals, good
customs, or public order.
GR: The taking out of an insurance contract is not compulsory.

3 – EXECUTIVE | Ab Initio Group

1
INSURANCE MIDTERMS (Weeks 1 to 8)
Acceptance made by letter or telegram does not bind the offerer except
from the time it came to his knowledge. The contract, in such a case, is presumed
to have been entered into in the place where the offer was made. HELD: NO. Judgment is reversed, and the Enriquez shall have and recover from
the Sun Life the sum of P6,000 with legal interest from November 20, 1918, until
Art. 1353. The statement of a false cause in contracts shall render them void, if it paid, without special finding as to costs in either instance. So ordered.
should not be proved that they were founded upon another cause which is true
and lawful. Perez v. CA, GR 112329. 28 January 2000

Enriquez v. Sun Life Assurance, GR 15895, 29 Nov. 1920 Facts:


Primitivo B. Perez had been insured with the BF Lifeman Insurance Corporation for
FACTS: P20,000.00. Sometime in October 1987, an agent of the insurance corporation,
 September 24, 1917: Joaquin Herrer made application to the Sun Life visited Perez in Quezon and convinced him to apply for additional insurance
Assurance Company of Canada through its office in Manila for a life coverage of P50,000.00. Virginia A. Perez, Primitivo’s wife, paid P2,075.00 to the
annuity agent. The receipt issued indicated the amount received was a "deposit."
 2 days later: he paid P6,000 to the manager of the company's Manila Unfortunately, the agent lost the application form accomplished by Perez and he
office and was given a receipt asked the latter to fill up another application form. The agent sent the application
 according to the provisional receipt, 3 things had to be accomplished by for additional insurance of Perez to the Quezon office. Such was supposed to
the insurance company before there was a contract:  forwarded to the Manila office.
 (1) There had to be a medical examination of the applicant; Perez drowned. His application papers for the additional insurance of P50,000.00
-check were still with the Quezon. It was only after some time that the papers were
 (2) there had to be approval of the application by the head office brought to Manila. Without knowing that Perez died, BF Lifeman Insurance
of the company; and - check Corporation approved the application and issued the corresponding policy for the
 (3) this approval had in some way to be communicated by the P50,000.00.
company to the applicant - ? Petitioner Virginia Perez went to Manila to claim the benefits under the insurance
 November 26, 1917: The head office at Montreal, Canada gave notice of policies of the deceased. She was paid P40,000.00 under the first insurance policy
acceptance by cable to Manila but this was not mailed for P20,000.00 but the insurance company refused to pay the claim under the
 December 4, 1917: policy was issued at Montreal additional policy coverage of P50,000.00, the proceeds of which amount to
 December 18, 1917: attorney Aurelio A. Torres wrote to the Manila office P150,000.00.
of the company stating that Herrer desired to withdraw his application The insurance company maintained that the insurance for P50,000.00 had not
 December 19, 1917: local office replied to Mr. Torres, stating that the been perfected at the time of the death of Primitivo Perez. Consequently, the
policy had been issued, and called attention to the notification of insurance company refunded the amount paid.
November 26, 1917 BF Lifeman Insurance Corporation filed a complaint against Virginia Perez seeking
 December 21, 1917 morning: received by Mr. Torres the rescission and declaration of nullity of the insurance contract in question.
 December 20, 1917: Mr. Herrer died Petitioner Virginia A. Perez, on the other hand, averred that the deceased had
 Rafael Enriquez, as administrator of the estate of the late Joaquin Ma. fulfilled all his prestations under the contract and all the elements of a valid
Herrer filed to recover from Sun Life Assurance Company of contract are present.
Canada through its office in Manila for a life annuity On October 25, 1991, the trial court rendered a decision in favor of petitioner
 RTC: favored Sun Life Insurance ordering respondent to pay 150,000 pesos. The Court of Appeals, however,
ISSUE: W/N Mr. Herrera received notice of acceptance of his application reversed the decision of the trial court saying that the insurance contract for
thereby perfecting his life annuity
3 – EXECUTIVE | Ab Initio Group

2
INSURANCE MIDTERMS (Weeks 1 to 8)
P50,000.00 could not have been perfected since at the time that the policy was condition depends upon the sole will of the debtor, the conditional obligation shall
issued, Primitivo was already dead. be void.
Petitioner’s motion for reconsideration having been denied by respondent court, The following conditions were imposed by the respondent company for the
the instant petition for certiorari was filed on the ground that there was a perfection of the contract of insurance: a policy must have been issued, the
consummated contract of insurance between the deceased and BF Lifeman premiums paid, and the policy must have been delivered to and accepted by the
Insurance Corporation. applicant while he is in good health.

Issue: WON the widow can receive the proceeds of the 2 nd insurance The third condition isn’t potestative, because the health of the applicant at the
policy time of the delivery of the policy is beyond the control or will of the insurance
company. Rather, the condition is a suspensive one whereby the acquisition of
Held: No. Petition dismissed. rights depends upon the happening of an event which constitutes the condition. In
this case, the suspensive condition was the policy must have been delivered and
Ratio: accepted by the applicant while he is in good health. There was non-fulfillment of
Perez’s application was subject to the acceptance of private respondent BF the condition, because the applicant was already dead at the time the policy was
Lifeman Insurance Corporation. The perfection of the contract of insurance issued.
between the deceased and respondent corporation was further conditioned with
the following requisites stated in the application form: As stated above, a contract of insurance, like other contracts, must be assented to
by both parties either in person or by their agents. So long as an application for
"there shall be no contract of insurance unless and until a policy is issued on this insurance has not been either accepted or rejected, it is merely an offer or
application and that the said policy shall not take effect until the premium has proposal to make a contract. The contract, to be binding from the date of
been paid and the policy delivered to and accepted by me/us in person while application, must have been a completed contract.
I/We, am/are in good health." The insurance company wasn’t negligent because delay in acting on the
application does not constitute acceptance even after payment. The corporation
BF Lifeman didn’t give its assent when it merely received the application form and may not be penalized for the delay in the processing of the application papers due
all the requisite supporting papers of the applicant. This happens only when it to the fact that process in a week wasn’t the usual timeframe in fixing the
gives a policy. application. Delay could not be deemed unreasonable so as to constitute gross
negligence.
It is not disputed, however, that when Primitivo died on November 25, 1987, his
application papers for additional insurance coverage were still with the branch
office of respondent corporation in Quezon. Consequently, there was absolutely no De Lim v. Sun Life of Canada, GR 15774, Nov. 29, 1920
way the acceptance of the application could have been communicated to the
applicant for the latter to accept inasmuch as the applicant at the time was already Facts:
dead. Luis Lim of Zamboanga applied for a Sun Life policy for Php 5,000. He designated
his wife, Pilar, as beneficiary. The first premium of P433 was paid by Lim, then the
Petitioner insists that the condition imposed by BF that a policy must have been company issued a "provisional policy." Lim died after the issuance of the
delivered to and accepted by the proposed insured in good health is potestative, provisional policy but before approval of the application.
being dependent upon the will of the corporation and is therefore void. The court
didn’t agree. A potestative condition depends upon the exclusive will of one of the Pilar brought an action to recover from Sun Life the sum of P5,000, the
parties and is considered void. The Civil Code states: When the fulfillment of the amount named in the provisional policy. She lost in the trial court hence this
appeal.
3 – EXECUTIVE | Ab Initio Group

3
INSURANCE MIDTERMS (Weeks 1 to 8)
subordinated to the act of the company in approving or rejecting; so in life
The "provisional policy" reads as follows: insurance a "binding slip" or "binding receipt" does not insure of itself.

The above-mentioned life is to be assured in accordance with the terms The court held that this second point applied to the case.
and conditions contained or inserted by the Company in the policy which may be Steinle vs. New York Life Insurance Co.-  the amount of the first premium
granted by it in this particular case for four months only from the date of the had been paid to an insurance agent and a receipt was given. The paper declared
application, provided that the Company shall confirm this agreement by issuing a that if the application was accepted by the company, the insurance shall take
policy on said application when the same shall be submitted to the Head Office in effect from the date of the application but that if the application was not accepted,
Montreal. Should the Company not issue such a policy, then this agreement shall the money shall be returned. The court held that there was no perfection of the
be null and void ab initio, and the Company shall be held not to have been on the contract.
risk at all, but in such case the amount herein acknowledged shall be returned.
Cooksey vs. Mutual Life Insurance Co.- the person applying for the life
Issue: WON there was a perfected contract of insurance insurance paid and amount equal to the first premium, but the application and the
receipt for the money paid, stipulated that the insurance was to become effective
Held: No. Petition dismissed. only when the application was approved and the policy issued. There was also no
perfection.
Ratio:
The policy for four months is expressly made subjected to the affirmative condition A binding receipt is a custom where temporary insurance pending the
that "the company shall confirm this agreement by issuing a policy on said consideration of the application was given until the policy be issued or the
application when the same shall be submitted to the head office in Montreal." application rejected, and such contracts are upheld and enforced when the
  Should the company not issue such a policy, then this agreement shall be null applicant dies before the issuance of a policy or final rejection of the application.
and void ab initio, and the company shall be held not to have been on the risk."
This means that the agreement should not go into effect until the home office of However, there was no perfected contract because of the clause in the
the company should confirm it by issuing a policy. The provisional application and the receipt stipulate expressly that the insurance shall become
policy amounts to nothing but an acknowledgment on behalf of the company, that effective only when the "application shall be approved and the policy duly signed
it has received from the person named therein the sum of money agreed upon as by the secretary at the head office of the company and issued." The premium of
the first year's premium upon a policy to be issued upon the application, if the 433 must be returned.
application is accepted by the company.
There can be no contract of insurance unless the minds of the parties have met Musngi v. West Coast Life, GR 41794,30 Aug. 1935
in agreement. In this case, the contract of insurance was not consummated by the
parties. Facts: Plaintiffs Musngi and Garia filed a suit against West Coast Life to recover
the value of the two insurance policies applied for by Arsenio Garcia (the insured).
The general rule concerning the agent's receipt pending approval or issuance of In applying for the said insurance, the insured was asked if he consulted a
policy is in several points, according to Joyce: physician or practitioner, and if so, for what ailment or illness; he answered
“None” and “No”, respectively. These answers were one of the causes or
Where an agreement is made between the applicant and considerations for the issuance of the policies. Upon the death of the insured, the
the agent whether by signing an application containing such condition, or plaintiffs, as beneficiaries, demanded West Coast to pay the value of the policies.
otherwise, that no liability shall attach until the principal approves the risk and a However, West Coast found out that the insured had been treated by a lady
receipt is given buy the agent, such acceptance is merely conditional, and it physician in the General Hospital in Manila for different ailments such as incipient
pulmonary tuberculosis, chronic bronchitis, and chronic suppurative pyelocystisis.
3 – EXECUTIVE | Ab Initio Group

4
INSURANCE MIDTERMS (Weeks 1 to 8)
It contended that the two policies did not create any valid obligation because they
were fraudulently obtained by the insured. The appealed decision held that the
health of the insured, before acceptance of his application and the issuance of the
policies, could neither be discussed nor questioned by the defendant company
because three physicians of the company examined the insured; however, all the
WEEK 2
physicians signified that the insured was in good health. Nevertheless, the findings
of the company physicians were nit the main cause for the issuance (or non- ESSENTIAL ELEMENTS
issuance) of the policies.
Gulf Resorts v. Philippine Charter lns., GR 156167, 16 May 2005
Issues:
1.) W/N the statements of the insured regarding his health were Facts – Petitioner Gulf Resorts owned Playa Resorts at Agoo, La Union, and it
false entered into an insurance contract with the respondent American Home Assurance
2.) W/N such statements were the causes which induced the Company which insured Plaza Resort’s properties against loss or damage due to
defendant company to issue the policies earthquakes. On 16 July 1990, an earthquake struck Central Luzon and Northern
Luzon and the properties in Playa Resort were damaged including the two (2)
Held/Ratio: swimming pools. On 11 August 1990, Gulf Resorts filed its formal demand for
1.) Yes, the statements of the insured regarding his health were settlement of the damage to all of its properties in the Agoo Playa Resort, but on
false because at the time of application, he already knew that 23 August 1990, American Home Assurance Company denied Gulf Resorts’ claim
he had been suffering from ailments. on the ground that its insurance policy only covered the two swimming pools of
Playa Resort against earthquake shock, and not the other properties damaged by
2.) Yes, such statements were the causes which induced the the said earthquake. Gulf Resorts contended that pursuant to this rider, no
defendant company to issue the policies. The concealment qualifications were placed on the scope of the earthquake shock coverage, and
and false statements of the insured constituted fraud; the thus, the policy extended earthquake shock coverage to its properties.
insurance company, by reason of such statement, accepted
the risk associated with the policy nut for which it could have On 24 January 1991, Gulf Resorts filed a complaint with the RTC Pasig where it
rejected. It was further held that the principal question must asked for payment for P5, 427,779.00 as amount of the lost/damaged properties,
be whether the assurer (the insurance company) was misled attorney fees, lost income, etc. However, on 21 February 1994, the said RTC ruled
or deceived into entering a contract of obligation or in fixing in favor of American Home Assurance Company, where it ruled that the insurance
the premium of the insurance by withholding of material coverage against earthquake is limited only to the two swimming pools of Gulf
information or facts within the assured’s knowledge or Resorts’ Playa Resort and does not extend against the other properties damaged
presumed knowledge. Under Art. 1276 of the Civil Code, the by the earthquake.
statement of a false consideration shall render the contract
void. Such matter is not specifically provided for by the Gulf Resort filed a motion for the trial court to reconsider its decision, but to no
Insurance Law, and so the general rules of the civil law avail. Gulf Resorts filed an appeal with the Court of Appeals, but the Court of
regarding contracts (there being a life insurance contract) Appeals affirmed the trial court’s decision.
shall apply. Thus, a valid cause or consideration is a requisite
for a valid insurance contract. Issue – W/N the insurance policy issued by American Home Assurance
Company (AHAC-AIU) to Gulf Resort’s coverage is not limited only to the
two swimming pools of Gulf Resorts’ Playa Resort?

3 – EXECUTIVE | Ab Initio Group

5
INSURANCE MIDTERMS (Weeks 1 to 8)
Held – The Supreme Court held that the insurance policy issued to Gulf Resorts is After Ernani was discharged from the MMC, he was attended by a physical
only limited to the two swimming pools and the other properties of Playa Resort therapist at home. Later, he was admitted at the Chinese General Hospital. Due to
are not covered by the property insurance issued by American Home Assurance financial difficulties, however, respondent brought her husband home again. In the
Company (AHAC-AIU). The Court held that there is no ambiguity in the insurance morning of April 13, 1990, Ernani had fever and was feeling very weak.
contract and the earthquake shock rider, as Gulf Resorts stated that the swimming Respondent was constrained to bring him back to the Chinese General Hospital
pools are the only items covered by the insurance against loss due to earthquakes. where he died on the same day.
The Court stated that provisions in the insurance policy should be examined and
interpreted in consonance with each other, and should not be construed Julita filed an action for damages and reimbursement of her expenses plus moral
piecemeal. All parts of the insurance contract reflect the true intent of the parties. damages attorney’s fees against Philamcare and its president, Dr. Benito
The Supreme Court also defined contracts of adhesion as contracts where one Reverente. The Regional Trial court or Manila rendered judgment in favor of Julita.
party prepares the stipulations in the contract while the other party merely affixes On appeal, the decision of the trial court was affirmed but deleted all awards for
his/her signature thereto, citing the case of Philippine National Bank vs. Court of damages and absolved petitioner Reverente. Hence, this petition for review raising
Appeals (196 SCRA 536). Any ambiguity is resolved against the insurer (who the primary argument that a health care agreement is not an insurance contract;
prepared the contract) and construed liberally in the insured’s favor. However, hence the “incontestability clause” under the Insurance Code does not apply.
since the policy and its riders are clear about the insurance coverage against
earthquake shock, the Gulf Resorts cannot use the doctrine of contract of adhesion ISSUES –
and liberal interpretation of insurance contract in the insured’s favor in case of (1) Whether or not the health care agreement is not an insurance
ambiguity. contract
(2) Whether or not there is concealment of material fact made by Ernani
Philamcare Health Systems v. CA, GR 125678, 18 March 2002
HELD - (1)YES. Section2 (1)of the Insurance Code defines a contract of insurance
FACTS - Ernani Trinos applied for a health care coverage with Philamcare Health as an agreement whereby one undertakes for a consideration to indemnify another
Systems, Inc. To the question ‘Have you or any of your family members ever against loss, damage, or liability arising from an unknown or contingent event.
consulted or been treated for high blood pressure, heart trouble, diabetes, cancer,
liver disease, asthma or peptic ulcer?’, Ernani answered ‘No’. Under the Section 3 of the Insurance Code states that any contingent or unknown event,
agreement, Ernani is entitled to avail of hospitalization benefits and out-patient whether past or future, which my damnify a person having an insurable against
benefits. The coverage was approved for a period of one year from March 1, 1988 him, may be insured against. Every person has an insurable interest in the life and
to March 1, 1989. The agreement was however extended yearly until June 1, 1990 health of himself.
which increased the amount of coverage to a maximum sum of P75,000 per
disability. Section 10 provides that every person has an insurable interest in the life and
health (1) of himself, of his spouse and of his children.
During the period of said coverage, Ernani suffered a heart attack and was
confined at the Manila Medical Center (MMC) for one month. While in the hospital, The insurable interest of respondent’s husband in obtaining the health care
his wife Julita tried to claim the benefits under the health care agreement. agreement was his own health. The health care agreement was in the nature of
However, the Philamcare denied her claim alleging that the agreement was void non-life insurance, which is primarily a contract of indemnity. Once the member
because Ernani concealed his medical history. Doctors at the MMC allegedly incurs hospital, medical or any other expense arising from sickness, injury or other
discovered at the time of Ernani’s confinement that he was hypertensive, diabetic stipulated contingent, the health care provider must pay for the same to the
and asthmatic, contrary to his answer in the application form. Thus, Julita paid for extent agreed upon under the contract.
all the hospitalization expenses.

3 – EXECUTIVE | Ab Initio Group

6
INSURANCE MIDTERMS (Weeks 1 to 8)
(2) NO. The answer assailed by petitioner was in response to the question relating
to the medical history of the applicant. This largely depends on opinion rather than In the case referred to in No. 1, the action for declaration of nullity may be
fact, especially coming from respondent’s husband who was not a medical doctor. brought by the spouse of the donor or donee; and the guilt of the donor and
Where matters of opinion or judgment are called for answers made I good faith donee may be proved by preponderance of evidence in the same action.
and without intent to deceive will not avoid a policy even though they are untrue.

The fraudulent intent on the part of the insured must be established to warrant lnsular Life v. Ebrado, GR 44059, 28 October 1977
rescission of the insurance contract. Concealment as a defense for the health care
provider or insurer to avoid liability is an affirmative defense and the duty to FACTS - Buenaventura Cristor Ebrado was married to Pascuala Ebrado. During his
establish such defense by satisfactory and convincing evidence rests upon the lifetime, he was living with his common-law wife, Carponia Ebrado, although he
provider or insurer. In any case, with or without the authority to investigate, was not legally separated from his legal wife. Buenaventura was issued by The
petitioner is liable for claims made under the contract. Having assumed a Insular Life Assurance Co., Ltd., Policy No. 009929 on a whole-life plan for PhP
responsibility under the agreement, petitioner is bound to answer to the extent 5,8882.00 with a rider for Accidental Death Benefits for the same amount.
agreed upon. In the end, the liability of the health care provider attaches once the Buenaventura designated Carponia Ebrado as the revocable beneficiary in his
member is hospitalized for the disease or injury covered by the agreement or policy. Buenaventura died as a result of an accident when he was hit by a falling
wherever he avails of the covered benefits which he has prepaid. branch of a tree. As the insurance policy was still in force, The Insular Life
Assurance Co., Ltd stands liable to pay the coverage.
Being a contract of adhesion, the terms of an insurance contract are to be
construed strictly against the party which prepared the contract – the insurer. By Carponia Ebrado, his common-law wife, filed with the insurer a claim for the
reason of the exclusive control of the insurance company over the terms and proceeds of the policy as the designated beneficiary therein. Pascuala Vda. de
phraseology of the insurance contract, ambiguity must be strictly interpreted Ebrado also filed her claim as the widow of the deceased insured. She asserts that
against the insurer and liberally in favor of the insured, especially to avoid she is the one entitled to the insurance proceeds, not the common-law wife.
forfeiture. This is equally applicable to Health Care Agreements. In doubt as to whom the insurance proceeds shall be paid, the insurer, The Insular
Life Assurance Co., Ltd, commenced an action for Interpleader before the CFI of
Art. 2011. The contract of insurance is governed by special laws. Matters not Rizal.
expressly provided for in such special laws shall be regulated by this Code.
ISSUE - Can a common-law wife of a man who was not legally separated
Art. 2012. Any person who is forbidden from receiving any donation under Article from his legal wife be a beneficiary of his life insurance plan?
739 cannot be named beneficiary of a life insurance policy by the person who
cannot make any donation to him, according to said article. HELD/RULING - No.
The Insurance Act (RA 2327, as amended) or even the new Insurance Code (PD
Art. 739. The following donations shall be void: No. 612, as amended) does not contain any specific provision grossly resolutory to
(1) Those made between persons who were guilty of adultery or concubinage the question at hand. Section 50 of the Insurance Act, which provides that “(t)he
at the time of the donation; insurance shall be applied exclusively to the proper interest of the person in whose
name it is made,” cannot be interpreted that it includes the beneficiary because a
(2) Those made between persons found guilty of the same criminal offense, contract of insurance is personal in character. The general rules of civil law should
in consideration thereof; be applied to resolve this void in the Insurance Law. Article 2011 of the New Civil
Code states: “The contract of insurance is governed by special laws. Matters not
(3) Those made to a public officer or his wife, descedants and ascendants, by expressly provided for in such special laws shall be regulated by this Code.” Article
reason of his office. 2012 of the same Code states that, “any person who is forbidden from receiving
3 – EXECUTIVE | Ab Initio Group

7
INSURANCE MIDTERMS (Weeks 1 to 8)
any donation under Article 739 cannot be named beneficiary of a life insurance for moral damages in the amount of P10,000.00, exemplary damages of
policy by the person who cannot make a donation to him.” Therefore, common- P5,000.00, attorney's fees of P3,000.00 and litigation expenses of P3,000.00.
law spouses are barred from receiving donations from each other.
On June 4, 1986, a decision was rendered by the trial court ordered the insurance
Article 739 provides: company to pay the insured the damage incurred plus interest at the rate of twice
“The following donations shall be void: the prevailing interest rates, moral damages (20,000, that is twice the amount the
Those made between persons who were guilty of adultery or concubinage at the insured prayed for), exemplary damages, attorney's fees, litigation expenses and
time of the donation; Those made between persons found guilty of the same costs. The CA affirmed the decision of the trial court.
criminal offense, in consideration thereof; Those made to a public officer or his
wife, descendants or ascendants by reason of his office. ISSUE - Whether the insurer is liable to the insured for moral and
exemplary damages as ordered by the trial court
In the case referred to in No. 1, the action for declaration of nullity may be
brought by the spouse of the donor or donee; and the guilt of the donee may be HELD - The decision of the lower court is modified to which, the moral damages is
proved by preponderance of evidence in the same action.” reduced to 10,000 and the award of exemplary damages is deleted

In essence, a life insurance policy is no different from a civil donation insofar as In case of unreasonable delay in the payment of the proceeds of an insurance
the beneficiary is concerned. Both are founded upon the same consideration: policy, the damages that may be awarded are: 1) attorney's fees; 2) other
liberality. As a consequence, the proscription in Art. 739 of the New Civil Code expenses incurred by the insured person by reason of such unreasonable denial or
should equally operate in life insurance contracts. withholding of payment; 3) interest at twice the ceiling prescribed by the Monetary
Board of the amount of the claim due the injured; and 4) the amount of the claim.
In the case at bar, the requisite proof of common-law relationship between the (Insurance Code Sec. 244) In awarding moral damages in case of breach of
insured and the beneficiary has been supplied by the stipulations between the contract, there must be a showing that the breach was wanton and deliberately
parties in the pre-trial conference. It was agreed an stipulated that the deceased injurious or the one responsible acted fraudulently or in bad faith. The act of
insured Buenaventura Ebrado was married to Pascuala Ebrado and that, during the petitioner of delaying payment for two months cannot be considered as so wanton
lifetime of the deceased insured, he was living with his common-law wife, or malevolent to justify an award of P20,000.00 as moral damages, taking into
Carponia Ebrado. Based on the foregoing, Carponia Ebrado is hereby declared consideration the actual damage. The reason for petitioner's failure to indemnify
disqualified to be the beneficiary of the late Buenaventura Ebrado in his life private respondent within the two-month period was that the parties could not
insurance policy. The proceeds of the policy are hereby held payable to the estate come to an agreement as regards the amount of the actual damage on the car.
of the deceased insured.
On the other hand, exemplary or corrective damages are imposed by way of
Zenith lnsurance v. CA, GR 85296, 14 May 1990 example or correction for the public good (Art. 2229, NCC). In the case of Noda v.
Cruz-Arnaldo, G.R. No. 57322, June 22,1987; 151 SCRA 227, exemplary damages
FACTS - On January 25, 1983, private respondent Lawrence Fernandez insured were not awarded as the insurance company had not acted in wanton, oppressive
with the insurer his car for "own damage". The car figured in an accident and or malevolent manner. The same is true in the case at bar.
suffered actual damages in the amount of P3,640.00. The insurer offered to pay
the claim of Fernandez pursuant to the terms and conditions of the contract which, Art. 2027. No annuity shall be claimed without first proving the existence of the
the private respondent rejected. After allegedly being given a run around by Zenith person upon whose life the annuity is constituted.
for two (2) months, Fernandez filed a complaint with the Regional Trial Court for
sum of money and damages resulting from the refusal of Zenith to pay the
amount claimed. Aside from actual damages and interests, Fernandez also prayed Fireman's Fund et al v Jamila et al., GR27427, 7 April 1976
3 – EXECUTIVE | Ab Initio Group

8
INSURANCE MIDTERMS (Weeks 1 to 8)
order to help Fireman's Fund to recover the amount of the loss from Jamila and
FACTS - Jamila or the Veterans Philippine Scouts Security Agency contracted to First Quezon City Insurance Co., Inc. Firestone had tacitly assigned to Fireman's
supply security guards to Firestone; that Jamila assumed responsibility for the acts Fund its cause of action against Jamila for breach of contract. Sufficient ultimate
of its security guards; that First Quezon City Insurance Co., Inc. executed a bond facts are alleged in the complaint to sustain that cause of action.
in the sum of P20,000 to guarantee Jamila's obligations under that contract; that
on May 18, 1963 properties of Firestone valued at P11,925 were lost allegedly due 2) Fireman's Fund's action against Jamila is squarely sanctioned by article 2207. As
to the acts of its employees who connived with Jamila's security guard; that the insurer, Fireman's Fund is entitled to go after the person or entity that violated
Fireman's Fund, as insurer, paid to Firestone the amount of the loss; that its contractual commitment to answer for the loss insured against (PAL vs. Heald
Fireman's Fund was subrogated to Firestone's right to get reimbursement from Lumber Co).
Jamila, and that Jamila and its surety, First Quezon City Insurance Co., Inc., failed
to pay the amount of the loss in spite of repeated demands. CFI erred in applying to this case the rules on novation. F&F in alleging in their
complaint that Fireman's Fund "became a party in interest in this case by virtue of
Upon defendant's motions, the lower court in its order of July 22, 1966 dismissed a subrogation right given in its favor by" Firestone, were not relying on the
the complaint as to Jamila on the ground that there was no allegation that it had novation by change of creditors as contemplated in NCC 1291 and 1300 to 1303
consented to the subrogation and, therefore, Fireman's Fund had no cause of but rather on NCC 2207.
action against it.
Article 2207 is a restatement of a settled principle of American jurisprudence.
Subrogation has been referred to as the doctrine of substitution. It "is an arm of
EQUITY that may guide or even force one to pay a debt for which an obligation
ISSUES - was incurred but which was in whole or in part paid by another" (83 C.J.S. 576).
1) Whether the complaint of Firestone states a cause of action against
Jamila? "Subrogation is founded on principles of JUSTICE AND EQUITY, and its operation
2) Whether the complaint of Fireman's Fund states a cause of action is governed by principles of equity. It rests on the principle that substantial justice
against Jamila? should be attained regardless of form, that is, its basis is the doing of complete,
3) Whether Jamila should reimburse Fireman’s Fund? essential, and perfect justice between all the parties without regard to form"(83
HELD - CFI Decision's order of dismissal is legally untenable so it was C.J.S. 579- 80)
SET ASIDE with costs against Jamila & Co., Inc.
Subrogation is a normal incident of indemnity insurance (Aetna L. Ins. Co. vs
RATIO - Moses). Upon payment of the loss, the insurer is entitled to be subrogated pro
[F&F’s counsel gratuitously alleged in their brief that Firestone and Jamila entered tanto to any right of action which the insured may have against the third person
into a "contract of guard services" on June 1, 1965. That allegation was uncalled whose negligence or wrongful act caused the loss (44 Am. Jur. 2nd 745).
for because it is not found in the complaint and so created confusion which did not
exist. No copy of the contract was annexed to the complaint. That confusing The right of subrogation is of the highest EQUITY. The LOSS IN THE FIRST
statement was an obvious error since it was expressly alleged in the complaint INSTANCE is that of the INSURED but AFTER reimbursement or compensation, it
that the loss occurred on May 18, 1963. The fact that such an error was becomes the LOSS OF THE INSURER (44 Am. Jur. 2d 746).
committed is another instance substantiating the observation that F&F's counsel
had not exercised due care in the presentation of his case.] "Although many policies including policies in the standard form, now provide for
subrogation, and thus determine the rights of the insurer in this respect, the
1) Firestone is really a nominal party in this case as it had already been equitable right of subrogation as the legal effect of payment inures to the insurer
indemnified for the loss which it had sustained. It joined as a party-plaintiff in
3 – EXECUTIVE | Ab Initio Group

9
INSURANCE MIDTERMS (Weeks 1 to 8)
without any formal assignment or any express stipulation to that effect in the
policy" (44 Am. Jur. 2nd 746). The Supreme Court remanded the case back to the trial court.

Stated otherwise, when the insurance company pays for the loss, such payment Right of Subrogation of the Insurer
operates as an equitable assignment to the insurer of the property and all • Article 2207 of the Civil Code is founded on the well-settled principle
remedies which the insured may have for the recovery thereof. That right is not of subrogation. If the insured property is destroyed or damaged
dependent upon, nor does it grow out of, any privity of contract, or upon written through the fault or negligence of a party other than the assured, then
assignment of claim, and payment to the insured makes the insurer an assignee in the insurer, upon payment to the assured, will be subrogated to the
equity (Shambley v. Jobe-Blackley Plumbing and Heating Co). rights of the assured to recover from the wrongdoer to the extent that
the insurer has been obligated to pay.
3) Whether the plaintiffs would be able to prove their cause of action against • Payment by the insurer to the assured operates as an equitable
Jamila is another question. assignment to the former of all remedies which the latter may have
against the third party whose negligence or wrongful act caused the
Pan Malayan lnsurance v CA, GR 81026, 3 April 1990 loss.
• The right of subrogation is not dependent upon, nor does it grow out
FACTS - Canlubang Automotive Resources Corp. obtained from Pan Malay a of, any privity of contract or upon written assignment of claim. It
motor vehicle insurance policy for its Mitsubishi Colt Lancer. While the policy was accrues simply upon payment of the insurance claim by the insurer.
still in effect, the insured car was allegedly hit by a pick-up owned by Erlinda Fabie
but driven by another person. The car suffered damages in the amount of P42K. There are three exceptions to this rule:
Pan Malay defrayed the cost of repair of the insured car. It then demanded 1. where the assured by his own act releases the wrongdoer or third
reimbursement from Fabie and her driver of said amount, but to no avail. Pan party liable for the loss or damage
Malay filed a complaint for damages with the RTC of Makati against Fabie and the 2. where the insurer pays the assured the value of the lost goods without
driver. It averred that the damages caused to the insured car was settled under notifying the carrier who has in good faith settled the assured's claim
the “own damage” coverage of the insurance policy. Private respondents filed a for loss
motion to dismiss alleging that Pan Malay had no cause of action since the “won 3. where the insurer pays the assured for a loss which is not a risk
damage” clause of the policy precluded subrogation under Art. 2207 of the Civil covered by the policy, thereby effecting "voluntary payment"
Code. They contended that indemnification under said article is on the assumption
that there was no wrongdoer or no 3rd party at fault. The RTC dismissed Pan None of these exceptions are present in this case.
Malay’s complaint and ruled that payment under the “own damage” clause was an
admission by the insurer that the damage was caused by the assured and/or its As to the trial court’s ruling:
representatives. CA affirmed but on different ground. Applying the ejusdem When PanMalay utilized the phrase "own damage" — a phrase which is not found
generis rule, CA held that Section III-I of the policy, which was the basis for the in the insurance policy — to define the basis for its settlement of Canlubang's
settlement of the claim against insurance, did not cover damage arising from claim under the policy, it simply meant that it had assumed to reimburse the costs
collision or overturning due to the negligence of 3rd parties as one of the insurable for repairing the damage to the insured vehicle. It is in this sense that the so-
risks. called "own damage" coverage under Section III of the insurance policy is
differentiated from Sections I and IV-1 which refer to "Third Party Liability"
ISSUE - Was Pan Malay subrogated to the rights of Canlubang against coverage (liabilities arising from the death of, or bodily injuries suffered by, third
the driver and his employer? parties) and from Section IV-2 which refer to "Property Damage" coverage
(liabilities arising from damage caused by the insured vehicle to the properties of
HELD - Yes. third parties).
3 – EXECUTIVE | Ab Initio Group

10
INSURANCE MIDTERMS (Weeks 1 to 8)
After trial the lower court September 28, 1963, rendered its decisions dismissing
As to the Court of Appeals’ ruling: the complaint and awarding P10,000 as attorney's fees to defendant.  
The Court of Appeals' ruling on the coverage of insured risks stems from an
erroneous interpretation of the provisions of the policy. It violates a fundamental ISSUE:
rule on the interpretation of property insurance contracts where interpretation 1. W/N there was shortshipment on defendants part?
should be liberally in favor of the assured and strictly against the insurer in cases 2. W/N the non-presentation of the insurance policy fatal to
of disagreement between the parties. The meaning advanced by PanMalay plaintiff's case?
regarding the coverage of the policy is undeniable more beneficial to Canlubang 3. W/N defendant was merely acting as an agent of
than that insisted upon by the CA. In any case, the very parties to the policy were Louis Dreyfus & Co., who was the real shipper?
not shown to be in disagreement regarding the meaning and coverage of Section
III-I. Hence, it was improper for CA to assert its own interpretation of the contract RULING:
that is contrary to the clear understanding and intention of the parties to it. 1. None. Plaintiff's cause of action suffers from several fatal defects and
inconsistencies.  The alleged existence of the Karlshamn bills of lading
* Even assuming for the sake of argument that the insurance policy does not is negative by the fact that Exhibits A and B - the bills of lading
cover damage to the insured vehicle caused by negligent acts of third parties, and presented by plaintiff - show that the 2,032,000 kilos of copra loaded
that PanMalay's settlement of Canlubang's claim for damages allegedly arising in Siain were for Gdynia only.  Further destroying its case is the
from a collision due to private respondents' negligence would amount to testimony of plaintiff's own witness, Mr. Claro Pasicolan, who on direct
unwarranted or "voluntary payment", insurer may still recover from the third party examination affirmed[5] that these two exhibits constituted the
responsible for the damage to the insured property under Article 1236 of the Civil complete set of documents which them shipping agent in charge of
Code. the vessel S.S. NAGARA issued covering the copra cargo loaded at
Slain.  In view of this admission and for want of evidentiary support,
Sveriges Anfartygs Assurance v Qua Chee Gan, L-22146, 5 Sept 1967 plaintiff's belated claim that there is another complete set of
documents cannot be seriously taken.
FACTS:
  Qua Chee Gan, shipped on board the S.S. NAGARA, 2,032,000 kilos of bulk 2. No. The lower court ruled so, reasoning that unless the same — as the
copra at Quezon, consigned to DAL International Trading Co., in Gdynia, best evidence — were presented, it could not be conclusively
Poland.  The vessel first called at the port of Karlshamn, Sweden, where it unload- determined if "liability for short shipment" was a covered risk. And the
ed 696,419 kilos of bulk copra.  Then, it proceeded to Gdynia where it unloaded rule is that an insurer who pays the insured for loss or liability not
the remaining copra shipment.  The actual outturn weights in the latter port covered by the policy is not subrogated to the latter. 2 However, even
showed that only 1,569,429 kilos were discharged. Because of the alleged assuming that there was unwarranted — or "volunteer" — payment,
confirmed cargo shortage, the Polish cargo insurers had to indemnify the plaintiff could still recover what it paid — in effect — to the carrier
consignee for the value thereof.  Thereafter, the former sued the ship-owner.   from defendant shipper under Art. 1236 of the Civil Code which allows
Settlement was effected between the Polish cargo insurers and the ship owner.   a third person who pays on behalf of another to recover from the
On August 16, 1954, claiming to have been subrogated to the rights of the carrier, latter, although there is no subrogation. But since the payment here
plaintiff sued defendant before the Court of First Instance of Manila to was without the knowledge and consent of defendant, plaintiff's right
recover U.S. $60,733.53 plus 17% exchange tax, with legal interest, as the value of recovery is defeasible by the former's defenses since the Code is
of the alleged cargo short shipment and P10,000 as attorney's fees.  Defendant clear that the recovery is only up to the amount by which the
answered in due time and countered with a P15,000 counterclaim for attorney's defendant was benefited.
fees.

3 – EXECUTIVE | Ab Initio Group

11
INSURANCE MIDTERMS (Weeks 1 to 8)
3. Yes, suffice it to say that although on Exhibit A and B his name contract complained of, the insurance company shall be subrogated to the right of
appears as the shipper, yet the very loading certificate, Exhibit 3 [5- the insured against the wrong-doer or the person who has violated the contract. If
Deposition of Horle], issued and signed by the Chief Mate, and Master the amount paid by the insurance company doer not fully cover the injury or loss,
of the S.S. NAGARA shows that defendant was acting merely for the aggrieved party shall be entitled to recover the deficiency from the person
account of Louis Dreyfus & Co.  The other documentary causing the loss or injury.
exhibits[7] confirm this.  Anyway, in whatever capacity defendant is
considered, it cannot be liable since no shortshipment was shown. The insurance have no greater right than the party in interest thereof.

Plaintiff's action against defendant cannot, however, be considered as clearly St. Paul Fire v Macondray, GR L-27796,25 March 1976
unfounded as to warrant an award of attorney's fees as damages to defendant
under par. 4, Art. 2208 of the Civil Code.  The facts do not show that plaintiff's Facts:
cause of action was so frivolous or untenably as to amount to gross and evident -Winthrop Products, Inc., of New York shipped aboard the SS “Tai Ping”, owned
bad faith.[8] and operated by Wilhelm Wilhelmsen218 cartons and drums of drugs and
WHEREFORE, but for the award of attorney's fees to defendant which is medicine with Winthrop-Stearns Inc., Manila, Philippines as consingee.
eliminated, the decision appealed from is, in all other respects, hereby BarberSteamship Lines, Inc., agent of Wilhelm Wilhelmsen issued Bill of Lading No.
affirmed.  Costs against plaintiff-appellant. 34, in the name of Winthrop Products.
-The shipment was insured by the shipper against loss and/or damage with the St.
Rizal Surety v Manila Railroad, GR L-24043,25 April 1968 Paul Fire & Marine InsuranceCompany.
-“Tai Ping” arrived at the Port of Manila.
Facts: -The said shipment was discharged complete and in good order with the exception
On Nov 29, 1960, a vessel named SS Flying Trader, loaded on board a cargo of one (1) drum and several cartonswhich were in bad condition.
which is an offset press machine, from Italy to Manila. Upon reaching the port of -Because consignee failed to receive the whole shipment and as several cartons of
destination and upon unloading it, it was dropped b the crane which resulted to medicine were received in badorder condition, Winthrop-Sterns Philippines filed
damages of the machine. The plaintiff as the insurer had paid the consignee, the corresponding claim in the amount of Pl,109.67 representingthe C.I.F. value of
Suter, Inc. the amount of P16.5k for the machine and P180.70 for the the damaged drum and cartons of medicine with the carrier and the arrestre.
International Adjustment Bureau as adjuster’s fee. However, the arrastre charges -However, both refused to pay.
in this particular shipment was paid on the weight or measurement basis -Winthrop-Sterns Philippines filed its claim with the insurer, St. Paul Fire & Marine
whichever is higher, and not on the value thereof. insurance.
-The insurance company, on the basis of such claim, paid to the consignee the
Issue: insured value of the lost and damagedgoods, including other expenses in
Can the insurance get an amount greater than what was declared? connection therewith, in the total amount of $1,134.46.
-As subrogee of the rights of the shipper and/or consignee, the insurer, St. Paul
Fire & Marine Insurance Co., instituted with the Court of First Instance the present
Held: action against the defendants for the recovery of said amount of $1,134.46, plus
Plaintiff Insurance Company cannot recover from defendants an amount greater costs.
than that to which the consignee could lawfully lay claim. The management -The Lower court rendered judgment ordering defendants Macondray & Co., Inc.,
contract is clear, the amount is limited to P500. Barber Steamship Lines, Inc. andWilhelm Wilhelmsen to pay to the plaintiff
P300.00. It also held defendants Manila Railroad Company and Manila PortService
If the plaintiff’s property has been insured, and he has received indemnity from to pay to plaintiff, jointly and severally, the sum of P809.67.
the insurance company for the injury or loss arising out of the wrong or breach of
3 – EXECUTIVE | Ab Initio Group

12
INSURANCE MIDTERMS (Weeks 1 to 8)
-The Insurer, , contending that it should recover the amount of $1,134.46 or its -In the case at bar, the liabilities of the defendants- appellees with respect to the
equivalent in pesos (the rate of P3.90,instead of P2.00, for every US$1.00), filed lost or damaged shipments areexpressly limited to the C.I.F. value of the goods as
a motion for reconsideration, but this was denied. per contract of sea carriage embodied in the bill of lading, whichreads:
-The Insurer argues that, as subrogee of the consignee, it should be entitled to o Whenever the value of the goods is less than $500 per package or other freight
recover from the defendants-appelleesthe amount of $1,134.46 which it actually unit, their value in the calculation and adjustment of claims for which the Carrier
paid to the consignee and which represents the value of the lost anddamaged may be liable shall for the purpose of avoiding uncertainties and difficulties in
shipment as well as other legitimate expenses such as the duties and cost of fixing value be deemed to be the invoice value, plus freight and insurance if paid,
survey of said shipment, andthat the exchange rate on the date of the judgment, irrespective of whether any other value is greater or less.
which was P3.90 for every US$1.00.
-Defendants-appellees countered that: National Union Fire lnsurance v. Stolt Nielsen, GR 87958, 26 April 1990
o   Their liability is limited to the C.I.F. value of the goods, pursuant to contract of
sea carriage embodied in the bill of lading that the consignee’s (Winthrop-Stearns EMERGENCY RECIT: United Coconut Chemicals (SHIPPER) shipped distilled fatty acid on
board MT “StoltSceptre” (CARRIER). The shipment was insured under a marine cargo
Inc.) claim against the carrier (Macondray & Co., Inc.,Barber Steamship Lines,
policy with National Union Fire Insurance Co (INSURER). Upon receipt of the cargo by the
Inc., Wilhelm Wilhelmsen and the arrastre operators (Manila Port Service and consignee in Netherlands, it was totally contaminated. Hence, claim was made on the
Manila Railroad Company) was only for the sum of Pl,109.67 INSURER of the cargo. The INSURER as subrogee filed a claim for damages against the
CARRIER with RTC Manila. The CARRIER invoked that arbitration must be done pursuant
to the Charter. The INSURER opposed, arguing that the provision on arbitration was not
ISSUE(S): included in the Bill of Lading. SC: The INSURER cannot avoid the binding effect of the
1.Whether or not, in case of loss or damage, the liability of the carrier to arbitration clause. By subrogation, it became privy to the Charter Party as fully as the
the consignee is limited to the C.I.F value of the goods which were lost SHIPPER before the latter was indemnified, because as subrogee it stepped into the shoes
or damaged of the SHIPPER and is subrogated merely to the latter's rights.
2.Whether the insurer who has paid the claim in dollars to the consignee
should be reimbursed in its peso equivalent on the date of discharge of FACTS:
the cargo or on the date of the decision. On 9 January 1985, United Coconut Chemicals, Inc. shipped 404.774 metric tons
of distilled C6-C18 fatty acid on board MT "StoltSceptre," a tanker owned by Stolt-
HELD: Nielsen Philippines Inc., from Bauan, Batangas, Philippines, consigned to
The appeal is without merit and the judgement of the lower court is affirmed. "NieuweMatex" at Rotterdam, Netherlands, covered by Tanker Bill of Lading BL
-The purpose of the bill of lading is to provide for the rights and liabilities of the No. BAT-1.
parties in reference to the contract tocarry. The shipment was insured under a marine cargo policy with Petitioner National
-The stipulation in the bill of lading limiting the common carrier’s liability to the Union Fire InsuranceCompany of Pittsburg (hereinafter referred to as INSURER), a
value of the goods appearing in thebill, unless the shipper or owner declares a non-life American insurance corporation, through its settling agent in the
greater value, is valid and binding. Philippines, the American International Underwriters (Philippines), Inc., the other
-This limitation of the carrier’s liability is sanctioned by the freedom of the petitioner herein.
contracting parties to establish suchstipulations, clauses, terms, or conditions as Upon receipt of the cargo by the consignee in the Netherlands, it was found to be
they may deem convenient, provided they are not contrary to law,morals, good discoloured and totally contaminated. Hence, a claim was made on the Insurer of
customs and public policy. the cargo. The insurer as subrogee filed a claim for damages against the carrier
-A stipulation fixing or limiting the sum that may be recovered from the carrier on with the RTC of Manila.
the loss or deterioration of the goods is valid, provided it is: The carrier filed a motion to dismiss on the ground that the case was arbritrable
(a) reasonable and just under the circumstances, and and pursuant to the charter party as embodied in the bill of lading, arbitration
(b) has been fairly and freely agreed upon. must be done. The insurer opposed the motion by arguing that the provision on

3 – EXECUTIVE | Ab Initio Group

13
INSURANCE MIDTERMS (Weeks 1 to 8)
arbitration was not included in the bill of lading and even if it was included, it was rights. It can recover only the amount that is recoverable by the assured. And
nevertheless unjust and unreasonable. since the right of action of the SHIPPER-ASSURED is governed by the provisions of
The RTC denied the motion but upon reconsideration, the resolution on the motion the Bill of Lading, which includes by reference the terms of the Charter Party,
to dismiss was suspended or deferred. necessarily, a suit by the INSURER is subject to the same agreements. It has not
The carrier then filed a petition for review on certiorari with preliminary been shown that the arbitral clause in question is null and void, inoperative, or
injunction/TRO which was granted by the CA. incapable of being performed. Nor has any conflict been pointed out between the
Charter Party and the Bill of Lading.
ISSUE:
Are the terms of the Charter Party, particularly the provision on Cebu Shipyard vWilliam Lines, GR 132607,5 May 1999
arbitration, binding on the INSURER?
FACTS:
Cebu Shipyard and Engineering Works, Inc. (CSEW) is engaged in the business of
HELD: dry-docking and repairing of marine vessels while the Prudential Guarantee and
Yes. The pertinent portion of the Bill of Lading in issue provides in part: Assurance, Inc. (Prudential) is in the non-life insurance business.
xxx [A]ll the terms whatsoever of the said Charter  except the rate and payment of William Lines, Inc. is in the shipping business. It was the owner of M/V Manila City,
freight specified therein apply to and govern the rights of the parties concerned in a luxury passenger-cargo vessel, which caught fire and sank.
this shipment.xxx At the time of the unfortunate occurrence sued upon, subject vessel was insured
with Prudential for P45M for hull and machinery. The Hull Policy included an
The provision on arbitration in the Charter Party reads: “Additional Perils (INCHMAREE)” Clause covering loss of or damage to the vessel
4. Arbitration. Any dispute arising from the making, performance or termination of through the negligence of, among others, ship repairmen
this Charter Party shall be settled in New York, Owner and Charterer each Petitioner CSEW was also insured by Prudential for third party liability under a
appointing an arbitrator, who shall be a merchant, broker or individual experienced Shiprepairer’s Legal Liability Insurance Policy. The policy was for P10 million only,
in the shipping business; the two thus chosen, if they cannot agree, shall nominate under the limited liability clause
a third arbitrator who shall be an admiralty lawyer. Such arbitration shall be On Feb. 5, 1991, William Lines, Inc. brought its vessel, M/V Manila City, to the
conducted in conformity with the provisions and procedure of the United States Cebu Shipyard in Lapulapu City for annual dry-docking and repair.
arbitration act, and a judgment of the court shall be entered upon any award On Feb. 6, 1991, an arrival conference was held between representatives of
made by said arbitrator. Nothing in this clause shall be deemed to waive Owner's William Lines, Inc. and CSEW to discuss the work to be undertaken on the M/V
right to lien on the cargo for freight, deed of freight, or demurrage. Manila City. The contracts, denominated as Work Orders, were signed thereafter.,
Clearly, the Bill of Lading incorporates by reference the terms of the Charter Party. with the following stipulations:
It is settled law that the charter may be made part of the contract under which the 10. The Contractor shall replace at its own work and at its own cost any  work or
goods are carried by an appropriate reference in the Bill of Lading. As the material which can be shown to be defective and which is communicated in writing
respondent Appellate Court found, the INSURER "cannot feign ignorance of the 20. The insurance on the vessel should be maintained by the customer and/or
arbitration clause since it was already charged with notice of the existence of the owner of the vessel during the period the contract is in effect.
charter party due to an appropriate reference thereof in the bill of lading and, by The total liability of the Contractor to the Customer or of any sub-contractor shall
the exercise of ordinary diligence, it could have easily obtained a copy thereof be limited in respect of any defect or event to the sum of 1M.
either from the shipper or the charterer. While the M/V Manila City was undergoing dry-docking and repairs within the
We hold, therefore, that the INSURER cannot avoid the binding effect of the premises of CSEW, the master, officers and crew of M/V Manila City stayed in the
arbitration clause. By subrogation, it became privy to the Charter Party as fully as vessel, using their cabins as living quarters. Other employees hired by William
the SHIPPER before the latter was indemnified, because as subrogee it stepped Lines to do repairs and maintenance work on the vessel were also present during
into the shoes of the SHIPPER-ASSURED and is subrogated merely to the latter's the dry-docking.
3 – EXECUTIVE | Ab Initio Group

14
INSURANCE MIDTERMS (Weeks 1 to 8)
On February 16, 1991, after subject vessel was transferred to the docking quay, it The factual findings by the CA are conclusive on the parties and are not
caught fire and sank, resulting to its eventual total loss reviewable by this Court. 
On February 21, 1991, William Lines, Inc. filed a complaint for damages against
CSEW, alleging that the fire which broke out in M/V Manila City was caused by WON the doctrine of res ipsa loquitur applies against the crew- YES
CSEWs negligence and lack of care. For the doctrine of res ipsa loquitur to apply to a given situation, the
Prudential was impleaded as co-plaintiff, after it paid William Lines, Inc. the value following conditions must concur: (1) the accident was of a kind which
of the hull and machinery insurance on the M/V Manila City. As a result of such does not ordinarily occur unless someone is negligent; and (2) that the
payment Prudential was subrogated to the claim of P45 million, representing the instrumentality or agency which caused the injury was under the
value of the said insurance it paid. exclusive control of the person charged with negligence.
Trial Court:   CSEW to pay William Lines and Prudential (45M)
CA: Affirmed TC. Ordered the partial dismissal of the case insofar as CSEW and The facts and evidence reveal the presence of these conditions. First, the
William Lines were concerned. fire would not have happened in the ordinary course of things if
CSEW claims that the insurance policy does not cover loss resulting from the fault reasonable care and diligence had been exercised.
of negligent charterers that are assured in the same policy and by virtue of clause
20, it is deemed a co-assured. WON the provisions limiting CSEW’s liability for negligence to a
maximum of Php 1 million are valid- NO
ISSUE/S & RATIO:
Although contracts of adhesion have been consistently upheld as valid,
WON CSEW is co-assured, thus losses caused by it are not covered by reliance on such contracts cannot be favored especially where the facts
the policy-NO and circumstances warrant that subject stipulations be disregarded.
Tthe facts and circumstances vis-a-vis the nature of the provision sought
The fact that clause 20 benefited petitioner, does not automatically to be enforced should be considered, bearing in mind the principles of
make it a co-assured of William Lines. equity and fair play.
Intention of parties to make each other co-assured is to be gleaned from
the insurance policy itself and not from any other contract because the RULING: Petition denied
policy denominates the assured and the beneficiaries.
Manila Mahogany v CA, GR L-52756,12 October 1987
Prudential named only William Lines, Inc. as the assured. There was no
manifestation of any intention of William Lines Inc to make CSEW a co- FACTS:
assured. When the terms of a contract are clear, its stipulations control. Petitioner Manila Mahogany Manufacturing Corporation insured its Mercedes
Benz 4-door sedan withrespondent Zenith Insurance Corporation. The insured
If CSEW were deemed co-assured, it would nullify any claim of William vehicle was bumped and damaged by a truck owned bySan Miguel Corporation.
Lines Inc. No shipowner would agree to make shiprepairer a co-assured For the damage caused, respondent company paid petitioner five thousand
because any claim it has under the policy would be invalidated. Such pesos(P5,000.00) in amicable settlement. Petitioner's general manager executed a
result could not have been intended by William Lines Inc. Release of Claim, subrogatingrespondent company to all its right to action against
San Miguel Corporation.Thereafter, respondent company wrote Insurance
WON CSEW had “management and supervisory control“ of the ship at Adjusters, Inc. to demand reimbursement from SanMiguel Corporation of the
the time the fire broke out- YES amount it had paid petitioner. Insurance Adjusters, Inc. refused
reimbursement,alleging that San Miguel Corporation had already paid petitioner
P4,500.00 for the damages to petitioner'smotor vehicle, as evidenced by a cash
3 – EXECUTIVE | Ab Initio Group

15
INSURANCE MIDTERMS (Weeks 1 to 8)
voucher and a Release of Claim executed by the General Manager of  petitioner in such a case, the insurer will be entitled to recover from the insured whatever
discharging San Miguel Corporation from "all actions, claims, demands the rights it  has paid to the latter, unless the release was made with the consent of the
of action that nowexist or hereafter develop arising out of or as a consequence of insurer 
the
accident."Respondent insurance company thus demanded from petitioner reimburs F .F . Cruz v CA, GR L-52732, 29 August 1988
ement of the sum of P4,500.00 paid by San Miguel Corporation. Petitioner refused;
hence, the instant case. Facts:
A fire broke up from the furniture shop of the petitioner in Caloocan city early
ISSUE: September 6, 1974. Prior to that, neighbor of the said shop requested that the
Whether or not the respondent insurance company is subrogated to the petitioner should build a firewall but failed to do so. The cause of the fire was
rights of the petitioner againstSan Miguel Corporation. never discovered. Private respondent got P35k from the insurance on their house
and contents thereof.
HELD:
YES Issue:
RULING: Whether or not the 35k be deducted from the damages thereof
The Supreme Court held that if a property is insured and the owner receives the
indemnity from theinsurer, it is provided in [Article 2207 of the New Civil Code] Ruling:
that the insurer is Since P35k had already been claimed by the respondents, the court held that such
deemed subrogated  amount should be deducted from the award of damages in accordance with Art
to therights of the insured against the wrongdoer and if the amount paid by the 2207 NCC
insurer does not fully cover the loss,then the aggrieved party is the one entitled to Art. 2207. If the plaintiff’s property has been insured, and he has received
recover the deficiency. Under this legal provision, indemnity from the insurance company for the injury or loss arising out of the
the real partyin interest with regard to the portion of the indemnity paid is the wrong or breach of contract complained of, the insurance company shall be
insurer and not the insured. subrogated to the rights of the insured against the wrongdoer or the person who
Hence, petitioner is entitled to keep the sum of P4,500.00 paid by San Miguel has violated the contract. If the amount paid by the insurance company does not
Corporation under itsclear right to file a deficiency claim for damages incurred, fully cover the injury or loss, the aggrieved party shall be entitled to recover the
against the wrongdoer, should the insurancecompany not fully pay for the injury deficiency from the person causing the loss or injury.
caused (Article 2207, New Civil Code). Having been indemnified by their insurer, private respondents are entitled only to
However, when petitioner  released San Miguel Corporation from any liability, recover the deficiency from the petitioner.
petitioner's right to retain the sum  of P5,000.00 no longer  existed, thereby Whether or not the insurer should exercise the rights of the insured to which it
entitling private respondent to recover the same had been subrogated lies solely within the former’s sound discretion. Since the
.The right of subrogation can only exist after the insurer has paid the insured insurer is not a party to the case, its identity is not of record and no claim is made
otherwise the insured will be deprived of his right to full indemnity. If the on its behalf, the private respondent’s insurer has to claim his right to
insurance proceeds are not sufficient to cover the damagessuffered by the insured, reimbursement of the P35,000.00 paid to the insured.
then he may sue the party responsible for the damage for the remainder. To the
extentof the amount he has already received from, the insurer enjoys the right of
subrogation.Since the insurer can be subrogated to only such rights as the insured
may have,
WEEK 3
  should the insured,after receiving payment from the insurer, release the
wrongdoer who caused the loss, the insurer loses hisrights against the latter. But Construction of insurance contracts

3 – EXECUTIVE | Ab Initio Group

16
INSURANCE MIDTERMS (Weeks 1 to 8)
the act which produces the injury, the resulting death is within the
De la Cruz v. Capital lnsurance, GR L-21 574, 30 June 1966 protection of policies insuring against death or injury from accident
 while the participation of the insured in the boxing contest is voluntary,
FACTS: the injury was sustained when he slid, giving occasion to the infliction by
 Eduardo de la Cruz, employed as a mucker in the Itogon-Suyoc Mines, his opponent of the blow that threw him to the ropes of the ring is not
Inc. in Baguio, was the holder of an accident insurance policy "against  The fact that boxing is attended with some risks of external injuries does
death or disability caused by accidental means" not make any injuries received in the course of the game not accidental
 January 1, 1957: For the celebration of the New Year, the Itogon-Suyoc  In boxing as in other equally physically rigorous sports, such as basketball
Mines, Inc. sponsored a boxing contest for general entertainment wherein or baseball, death is not ordinarily anticipated to result. If, therefore, it
Eduardo, a non-professional boxer participated ever does, the injury or death can only be accidental or produced by some
 In the course of his bout with another non-professional boxer of the same unforeseen happening or event as what occurred in this case
height, weight, and size, Eduardo slipped and was hit by his opponent on  Furthermore, the policy involved herein specifically excluded from its
the left part of the back of the head, causing Eduardo to fall, with his head coverage —
hitting the rope of the ring (e) Death or disablement consequent upon the Insured engaging in football,
 He was brought to the Baguio General Hospital the following day.  He died hunting, pigsticking, steeplechasing, polo-playing, racing of any kind,
due to hemorrhage, intracranial. mountaineering, or motorcycling.
 Simon de la Cruz, the father of the insured and who was named  Death or disablement resulting from engagement in boxing
beneficiary under the policy, thereupon filed a claim with the insurance contests was not declared outside of the protection of the
company insurance contract 
 The Capital Insurance and Surety co., inc denied stating that the death
caused by his participation in a boxing contest was not accidental Sun lnsurance Office Ltd v CA, GR 92383, 17 July 1992
 RTC: favored Simon
 Facts:
ISSUE: W/N the cause of death was accident Lim accidentally killed himself with his gun after removing the magazine, showing
off, pointing the gun at his secretary, and pointing the gun at his temple. The
widow, the beneficiary, sued the petitioner and won 200,000 as indemnity with
HELD:YES. additional amounts for other damages and attorney’s fees. This was sustained in
 Eduardo slipped, which was unintentional the Court of Appeals then sent to the Supreme court by the insurance company.
 The terms "accident" and "accidental"
 as used in insurance contracts, have not acquired any technical Issue:
meaning and are construed by the courts in their ordinary and 1. Was Lim’s widow eligible to receive the benefits?
common acceptation 2. Were the other damages valid?
 happen by chance or fortuitously, without intention and design,
and which is unexpected, unusual, and unforeseen Held:
 event that takes place without one's foresight or expectation 1. Yes  2. No
 event that proceeds from an unknown cause, or is an unusual Ratio:
effect of a known cause and, therefore, not expected
 where the death or injury is not the natural or probable result of the 1. There was an accident.
insured's voluntary act, or if something unforeseen occurs in the doing of De la Cruz v. Capital Insurance says that "there is no accident when a
deliberate act is performed unless some additional, unexpected,
3 – EXECUTIVE | Ab Initio Group

17
INSURANCE MIDTERMS (Weeks 1 to 8)
independent and unforeseen happening occurs which produces or brings  He was brought to the Manila Central University hospital, and after
about their injury or death." This was true when he fired the gun. receiving first aid there, he went to the National Orthopedic Hospital for
Under the insurance contract, the company wasn’t liable for bodily injury treatment of his injuries.
caused by attempted suicide or by one needlessly exposing himself to  His injuries caused temporary total disability on his left hand so he filed a
danger except to save another’s life. claim against all defendants who rejected the claim reasoning that there it
was not covered in his policy because there was no severance of
Lim wasn’t thought to needlessly expose himself to danger due to the amputation of the left hand
witness testimony that he took steps to ensure that the gun wasn’t  Trial Court: absolved the defendants
loaded. He even assured his secretary that the gun was loaded. 
ISSUE: W/N Ty can claim 
There is nothing in the policy that relieves the insurer of the
responsibility to pay the indemnity agreed upon if the insured is shown
to have contributed to his own accident. HELD: NO. Affirmed.
 can not go beyond the clear and express conditions of the insurance
2. “In order that a person may be made liable to the payment of moral policies, all of which define partial disability as loss of either hand
damages, the law requires that his act be wrongful. The adverse result by amputation through the bones of the wrist
of an action does not per se make the act wrongful and subject the act  Note that the disability of plaintiff's hand was merely temporary, having
or to the payment of moral damages. The law could not have meant been caused by fracture of the index, the middle and the fourth fingers of
to impose a penalty on the right to litigate; such right is so precious that the left hand
moral damages may not be charged on those who may exercise it  agreement contained in the insurance policies is the law between the
erroneously. For these the law taxes costs.” parties

If a party wins, he cannot, as a rule, recover attorney's fees and Calanoc v CA, GR 81 51 , 16 Dec,ember 1955
litigation expenses, since it is not the fact of winning alone that entitles
him to recover such damages of the exceptional circumstances Doctrine: In case of ambiguity in an insurance contract covering accidental
enumerated in Art. 2208. Otherwise, every time a defendant wins, death, the Supreme Court held that such terms shall be construed strictly against
automatically the plaintiff must pay attorney's fees thereby putting a the insurer and liberally in favor of the insured in order to effect the purpose of
premium on the right to litigate which should not be so. For indemnity.
those expenses, the law deems the award of costs as sufficient.” Facts:
Melencio Basilio, a watchman of the Manila Auto Supply, secured a life insurance
Ty v Filipinas Cia de Seguros, GR L-21821-22 and L-21824-27, 31 May policy from the Philippine American Insurance Company in the amount of P2,000
1966 to which was attached a supplemental contract covering death by accident. He
later died from a gunshot wound on the occasion of a robbery committed;
FACTS: subsequently, his widow was paid P2,000 representing the face value of the policy.
 2 months prior to December 24, 1953: Diosdado C. Ty, employed as The widow demanded the payment of the additional sum of P2,000 representing
operator mechanic foreman in the Broadway Cotton Factory insured the value of the supplemental policy which the company refused because the
himself in 18 local insurance companies with Broadway Cotton Factory as deceased died by murder during the robbery and while making an arrest as an
his beneficiary officer of the law which were expressly excluded in the contract. The company’s
 December 24, 1953: fire broke out at the Broadway Cotton Factory where contention which was upheld by the Court of Appeals provides that the
Ty, fighting his way out, injured his left hand by a heavy object.  
3 – EXECUTIVE | Ab Initio Group

18
INSURANCE MIDTERMS (Weeks 1 to 8)
circumstances surrounding Basilio’s death was caused by one of the risks excluded would not apply where death resulted from an injury “intentionally inflicted by
by the supplementary contract which exempts the company from liability. another party.”

Issue: 2. May 20, 1964: While the said life policy and supplementary contract were in full
Is the Philippine American Life Insurance Co. liable to the petitioner for force and effect, the house of insured Juan Biagtan was robbed by a band of
the amount covered by the supplemental contract? robbers who were charged in and convicted for robbery with homicide; that in
Held: committing the robbery, the robbers rushed towards the door of the second floor
Yes. The circumstances of Basilio’s death cannot be taken as purely room, where they suddenly met a person near the door of one of the rooms who
intentional on the part of Basilio to expose himself to the danger. There is no proof turned out to be Juan Biagtan who received thrusts from their sharp-pointed
that his death was the result of intentional killing because there is the possibility instruments, causing wounds on the body of said Juan S. Biagtan resulting in his
that the malefactor had fired the shot merely to scare away the people around. In death at about 7 a.m. on the same day, May 21, 1964
this case, the company’s defense points out that Basilio’s is included among the
risks excluded in the supplementary contract; however, the terms and phraseology 3. Plaintiffs, as beneficiaries of the insured, filed a claim under the policy.
of the exception clause should be clearly expressed within the understanding of
the insured. Art. 1377 of the New Civil Code provides that in case ambiguity, 4. Insular paid P5,000.00 but refused to pay the additional sum of P5,000.00
uncertainty or obscurity in the interpretation of the terms of the contract, it shall under the accidental death benefit clause, on the ground that the insured’s death
be construed against the party who caused such obscurity. Applying this to the resulted from injuries intentionally inflicted by third parties and therefore was not
situation, the ambiguous or obscure terms in the insurance policy are to be covered.
construed strictly against the insurer and liberally in favor of the insured party. The
reason is to ensure the protection of the insured since these insurance contracts 5. Lower court: Rendered judgment in favor of plaintiff beneficiaries since the
are usually arranged and employed by experts and legal advisers acting exclusively parties presented no evidence, there was no "proof that the act of receiving thrust
in the interest of the insurance company. As long as insurance companies insist from the sharp-pointed instrument of the robbers was intended to inflict injuries
upon the use of ambiguous, intricate and technical provisions, which conceal their upon the person of the insured or any other person or merely to scare away any
own intentions, the courts must, in fairness to those who purchase insurance, person so as to ward off any resistance or obstacle that might be offered in the
construe every ambiguity in favor of the insured. pursuit of their main objective which was robbery.

Biagtan v lnsular Life, GR L-25579,29 March 1972


ISSUE:
FACTS: 1. WON the wounds received by the insured at the hands of the robbers
– nine in all, five of them mortal and four non-mortal – were inflicted
1. Juan Biagtan was insured with defendant Insular Life Assurance Company intentionally, hence the benefit clause cannot apply.
(Insular) under policy no. 398075 for the sum of P5,000.00 and under a
supplementary contract named “Accidental Death Benefit Clause, for an additional HELD:
sum of P5,000.00 if “death of the insured resulted directly from bodily injury 1. Yes. The decision appealed from is reversed and the complaint
effected solely through external and violent means sustained in an accident…and dismissed.
independently of all other causes.” The clause however expressly provided that it
RATIO:
3 – EXECUTIVE | Ab Initio Group

19
INSURANCE MIDTERMS (Weeks 1 to 8)
1. Whether the robbers had the intent to kill or merely to scare the victim or to stab wound on the insured. Therefore, said death was committed with deliberate
ward off any defense he might offer, it cannot be denied that the act itself of intent which, by the very nature of a personal accident insurance policy, cannot be
inflicting the injuries was intentional. It should be noted that the exception in the indemnified.
accidental benefit clause invoked by the appellant does not speak of the purpose
— whether homicidal or not — of a third party in causing the injuries, but only of ISSUE:
the fact that such injuries have been "intentionally" inflicted — this obviously to Whether or not death petitioner is correct that results from assault or
distinguish them from injuries which, although received at the hands of a third murder deemed are not included in the terms “accident” and
party, are purely accidental. “accidental”.

But where a gang of robbers enter a house and coming face to face with the HELD:
owner, even if unexpectedly, stab him repeatedly, it is contrary to all reason and NO. Petition for certiorari with restraining order and preliminary injunction was
logic to say that his injuries are not intentionally inflicted, regardless of whether denied for lack of merit.
they prove fatal or not.
The case of Calanoc vs. Court of Appeals, 98 Phil. 79, is relied upon by the trial RATIO:
court in support of its decision. The terms “accident” and “accidental” as used in insurance contracts
have not acquired any technical meaning, and are construed by the
For while a single shot fired from a distance, and by a person who was not even courts in their ordinary and common acceptation. Thus, the terms have
seen aiming at the victim, could indeed have been fired without intent to kill or been taken to mean that which happen by chance or fortuitously,
injure, nine wounds inflicted with bladed weapons at close range cannot without intention and design, and which is unexpected, unusual, and
conceivably be considered as innocent insofar as such intent is concerned. The unforeseen. An accident is an event that takes place without one’s
manner of execution of the crime permits no other conclusion. foresight or expectation — an event that proceeds from an unknown
cause, or is an unusual effect of a known cause and, therefore, not
Thus, it has been held that "intentional" as used in an accident policy excepting expected.
intentional injuries inflicted by the insured or any other person, etc., implies the [I]t is well settled that contracts of insurance are to be construed
exercise of the reasoning faculties, consciousness and volition. liberally in favor of the insured and strictly against the insurer. Thus
ambiguity in the words of an insurance contract should be interpreted in
Where a provision of the policy excludes intentional injury, it is the intention of the favor of its beneficiary.
person inflicting the injury that is controlling. If the injuries suffered by the
insured clearly resulted from the intentional act of a third person the insurer is
relieved from liability as stipulated.
Qua Chee Gan v Law Union and Rock, GR L-4611,17 December 1955
Finman GeneralAssurance v CA, GR 100970, 2 September 1992
FACTS:
FACTS:  Qua Chee Gan, a merchant of Albay, owned four bodegas which he
[P]etitioner filed this petition alleging grove abuse of discretion on the part of the insured with Law Union & Rock Insurance Co., Ltd (Law Union) since
appellate court in applying the principle of “ expresso unius exclusio alterius” in a 1937 and the lose made payable to the Philippine National Bank (PNB) as
personal accident insurance policy since death resulting from murder and/or mortgage of the hemp and crops, to the extent of its interest
assault are impliedly excluded in said insurance policy considering that the cause  July 21, 1940 morning: fire broke out in bodegas 1,2 and 4 which lasted
of death of the insured was not accidental but rather a deliberate and intentional for almost a week. 
act of the assailant in killing the former as indicated by the location of the lone  Qua Chee Gan informed Law Union by telegram
3 – EXECUTIVE | Ab Initio Group

20
INSURANCE MIDTERMS (Weeks 1 to 8)
 Law Union rejected alleging that it was a fraudulent claim that the fire had
been deliberately caused by the insured or by other persons in connivance Del Rosario v. Equitable lnsurance and Casualty, GR L-16215, 29 June
with him 1963
 Que Chee Gan, with his brother, Qua Chee Pao, and some employees of
his, were indicted and tried in 1940 for the crime of arson but was FACTS:
subsequently acquitted  April 13, 1957: Simeon del Rosario, father of the insured who died from
 During the pendency of the suit, Que Chee Gan paid PNB drowning filed a claim for payment with Equitable Ins. and Casualty Co.,
 Law Union states that ff. assignment of errors: Inc. but it refused to pay more than P1,000 php so a case was filed with
 1. memo of warranty requires 11 hydrants instead of 2  the RTC for the P2,000 balance stating that under the policy they are
 2. violation of hemp warranty against storage of gasoline since it entitled to P1,000 to P3,000 as indemnity
prohibits oils  RTC: entitled to recover P3,000 - policy does not positively state any
 3. fire was due to fraud definite amount, there is an ambiguity in this respect in the policy, which
 4. burned bodegas could not possibly have contained the ambiguity must be interpreted in favor of the insured and strictly against
quantities of copra and hemp stated in the fire claims the insurer so as to allow greater indemnity
 
ISSUE: W/N Qua Chee Gan should be allowed to claim. ISSUE: W/N Simeon is entitled to recover P3,000 

HELD: YES. Affirmed. HELD: YES.


 1. It is a well settled rule of law that an insurer which with knowledge of  terms in an insurance policy, which are ambiguous, equivocal or uncertain
facts entitling it to treat a policy as no longer in force, receives and are to be construed strictly against, the insurer, and liberally in favor of
accepts a preium on the policy, estopped to take advantage of the the insured so as to effect the dominant purpose of indemnity or payment
forfeiture to the insured, especially where a forfeiture is involved
 2. oils (animal and/or vegetable and/or mineral and/or their liquid  reason for this rule is that the "insured usually has no voice in the
products having a flash point below 300o Fahrenheit", and is decidedly selection or arrangement of the words employed and that the language of
ambiguous and uncertain; for in ordinary parlance, "Oils" mean the contract is selected with great care and deliberation by expert and
"lubricants" and not gasoline or kerosene legal advisers employed by, and acting exclusively in the interest of, the
 by reason of the exclusive control of the insurance company over insurance company
the terms and phraseology of the contract, the ambiguity must be Verendia v CA, GR 75605, 22 January 1993
held strictly against the insurer and liberraly in favor of the
insured, specially to avoid a forfeiture FACTS:
 3. trial Court found that the discrepancies were a result of the insured's  Rafael (Rex) Verendia's residential building was insured with Fidelity and
erroneous interpretation of the provisions of the insurance policies and Surety Insurance Company, Country Bankers Insurance and Development
claim forms, caused by his imperfect knowledge of English, and that the Insurance with Monte de Piedad & Savings Bank as beneficiary
misstatements were innocently made and without intent to defraud.  December 28, 1980 early morning: the building was completely destroyed
 4. Similarly, the 20 per cent overclaim on 70 per cent of the hemo stock, by fire
was explained by the insured as caused by his belief that he was entitled  Fidelity refused the claim stating that there was a misrepresentation since
to include in the claim his expected profit on the 70 per cent of the hemp, the lessee was not Roberto Garcia but Marcelo Garcia 
because the same was already contracted for and sold to other parties  trial court: favored Fidelity
before the fire occurred  CA: reversed
3 – EXECUTIVE | Ab Initio Group

21
INSURANCE MIDTERMS (Weeks 1 to 8)

ISSUE: W/N there was false declaration which would forfeit his benefits Held: Yes to both. Petition granted.
under Section 13 of the policy
Ratio:
The insurance agency contended that the guards automatically became the
HELD: YES. authorized representatives of the bank when they cited   International Timber
 Section 13 thereof which is expressed in terms that are clear and Corp. vs. NLRC where a contractor is a "labor-only" contractor in the sense that
unambiguous, that all benefits under the policy shall be forfeited "If the there is an employer-employee relationship between the owner of the project and
claim be in any respect fraudulent, or if any false declaration be made or the employees of the "labor-only" contractor.
used in support thereof, or if any fraudulent means or devises are used by They cited Art. 106. Of the Labor Code which said:
the Insured or anyone acting in his behalf to obtain any benefit under the  Contractor or subcontractor. — There is "labor-only" contracting where the person
policy" supplying workers to an employer does not have substantial capital or investment
 Robert Garcia then executed an affidavit before the National Intelligence in the form of tools, equipment, machineries, work premises, among others, and
and Security Authority (NISA) to the effect that he was not the lessee of the workers recruited and placed by such persons are performing activities which
Verendia's house and that his signature on the contract of lease was a are directly related to the principal business of such employer. In such cases, the
complete forgery. person or intermediary shall be considered merely as an agent of
 Worse yet, by presenting a false lease contract, Verendia, reprehensibly the employer who shall be responsible to the workers in the same manner and
disregarded the principle that insurance contracts are uberrimae fidae and extent as if the latter were directly employed by him.
demand the most abundant good faith The bank asserted that the guards were not its employees since it had nothing to
do with their selection and engagement, the payment of their wages, their
Fortune lnsurance v CA, GR 115278,23 May 1995 dismissal, and the control of their conduct.
They cited a case where an employee-employer relationship was governed by (1)
Facts: the selection and engagement of the employee; (2) the payment of wages; (3) the
Producers Bank’s money was stolen while it was being transported from Pasay to power of dismissal; and (4) the power to control the employee's conduct.
Makati. The people guarding the money were charged with the theft. The bank The case was governed by Article 174 of the Insurance Code where it stated
filed a claim for the amount of Php 725,000, and such was refused by the that casualty insurance awarded an amount to loss cause by accident or mishap.
insurance corporation due to the stipulation: “The term "employee," should be read as a person who qualifies as such as
 GENERAL EXCEPTIONS generally and universally understood, or jurisprudentially established in the light of
The company shall not be liable under this policy in report of the four standards in the determination of the employer-employee relationship, or
(b) any loss caused by any dishonest, fraudulent or criminal act of the insured or as statutorily declared even in a limited sense as in the case of Article 106 of the
any officer, employee, partner, director, trustee or authorized representative of the Labor Code which considers the employees under a "labor-only" contract as
Insured whether acting alone or in conjunction with others. . . . employees of the party employing them and not of the party who supplied them to
In the trial court, the bank claimed that the suspects were not any of the above the employer.”
mentioned. They won the case. The appellatecourt affirmed on the basis that the But even if the contracts were not labor-only, the bank entrusted the suspects
bank had no power to hire or dismiss the guard and could only ask for with the duty to safely transfer the money to its head office, thus, they were
replacements from the security agency. representatives. According to the court, “a ‘representative’ is defined as one who
represents or stands in the place of another; one who represents others or
Issue: Did the guards fall under the general exceptions clause of the another in a special capacity, as an agent, and is interchangeable with ‘agent.’”  
insurance policy and thus absolved the insurance company from
liability? Misamis Lumber v Capital lnsurance, GR L-21380, 20 May 1996
3 – EXECUTIVE | Ab Initio Group

22
INSURANCE MIDTERMS (Weeks 1 to 8)
(c) doing any kind of business, including a reinsurance business,
FACTS: specifically recognized as constituting the doing of an insurance
 Misamis Lumber Corporation (Misamis), formerly Lanao Timber Mills, Inc., business within the meaning of this Code;
insured its Ford Falcon motor car with Capital Insurance & Surety
Company (Capital) (d) doing or proposing to do any business in substance equivalent to any
 November 25, 1961 11 pm: The car broke when it hit a hollow block lying of the foregoing in a manner designed to evade the provisions of this
alongside the water hole which the driver did not see because the on- Code.
coming car did not dim its light
 The car was towed and repaired by Morosi Motors In the application of the provisions of this Code the fact that no
costing P302.27  profit is derived from the making of insurance contracts, agreements or
 November 29, 1961: After the repairs were made, Misamis made a report transactions or that no separate or direct consideration is received
to Capital who only admits liability of P150 therefor, shall not be deemed conclusive to show that the making
 CFI: paragraph 4 of the policy is clear and specific and leaves no room for thereof does not constitute the doing or transacting of an insurance
interpretation that the repair liability is limited to P150 business.

ISSUE: W/N Misamis is entitled to an amount exceeding P150 White Gold Marine v Pioneer lnsurance, GR 154514

FACTS: (White Gold > Pioneer > Steamship Mutual)


HELD: NO.
 insurance contract may be rather onerous (one-sided) but that in itself
does not justify the abrogation of its express terms, terms which the  White Gold Marine Services, Inc. (White Gold) procured a protection and
insured accepted or adhered to and which is the law between the indemnity coverage for its vessels from The Steamship Mutual
contracting parties Underwriting Association (Bermuda) Limited (Steamship Mutual) through
Pioneer Insurance and Surety Corporation (Pioneer)
WEEK 4  When White Gold failed to fully pay its accounts, Steamship Mutual
refused to renew the coverage
 Steamship Mutual thereafter filed a case against White Gold for collection
The Business of lnsurance of sum of money to recover the latter’s unpaid balance
 White Gold filed a complaint before the Insurance Commission
Section 2(2) The term "doing an insurance business"  or  "transacting an  Steamship Mutual violated Sections 186[4] and 187[5] of
insurance business", within the meaning of this Code, shall include: the Insurance Code
 Pioneer  violated  Sections  299,[6] 300[7] and 301[8] in
(a) making or proposing to make, as insurer, any insurance contract; relation to Sections 302 and 303, thereof
 Insurance Commission: dismissed the complaint
(b) making or proposing to make, as surety, any contract of suretyship  no need for Steamship Mutual to secure a license because it
as a vocation and not as merely incidental to any other legitimate was a Protection and Indemnity Club (P & I Club) (NOT engaged
business or activity of the surety; in the insurance business)
 Pioneer need not obtain another license as insurance agent and/or
a broker for Steamship Mutual because Steamship Mutual was not
engaged in the insurance business
3 – EXECUTIVE | Ab Initio Group

23
INSURANCE MIDTERMS (Weeks 1 to 8)
 Moreover, Pioneer was already licensed  The test to determine if a contract is an insurance contract or not,
 CA: affirmed Insurance Commission depends on the nature of the promise, the act required to be performed,
ISSUE:  and the exact nature of the agreement in the light of the occurrence,
1. W/N Steamship Mutual, a P & I Club, is engaged in the contingency, or circumstances under which the performance becomes
insurance business in the Philippines - YES. requisite
2. W/N Pioneer as resident agent of Steamship Mutual is required to  a marine insurance undertakes to indemnify the assured against marine
obtain a license as an insurance agent/broker - YES losses, such as the losses incident to a marine adventure
 a mutual insurance company is a cooperative enterprise where the
HELD:  petition is PARTIALLY GRANTED. CA affirmed. the revocation of Pioneer’s members are both the insurer and insured
Certificate of Authority and removal of its directors and officers, is DENIED  the members all contribute, by a system of premiums or
assessments, to the creation of a fund from which all losses and
1. YES liabilities are paid, and where the profits are divided among
Sec. 2(2) themselves, in proportion to their interest
(2) The term "doing an insurance business" or "transacting an insurance  provide 3 types of coverage: 
business", within the meaning of this Code, shall include:  protection and indemnity
 war risks
 defense costs
(a) making or proposing to make, as insurer, any insurance contract;  P & I Club
(b) making or proposing to make, as surety, any contract of suretyship as a  a form of insurance against third party liability, where the third
vocation and not as merely incidental to any other legitimate business or activity party is anyone other than the P & I Club and the members
of the surety;  Steamship Mutual as a P & I Club is a mutual insurance
association engaged in the marine insurance business
 Since a contract of insurance involves public interest, regulation by the
(c) doing any kind of business, including a reinsurance business, specifically State is necessary.  Thus, no insurer or insurance company is allowed to
recognized as constituting the doing of an insurance business within the engage in the insurance business without a license or a certificate of
meaning of this Code; authority from the Insurance Commission

2. YES.
 Although Pioneer is already licensed as an insurance company, it needs a
(d) doing or proposing to do any business in substance equivalent to any of the
separate license to act as insurance agent for Steamship Mutual.  
foregoing in a manner designed to evade the provisions of this Code.
Insurance Code
Sec. 299
In the application of the provisions of this Code the fact that no profit is derived
from the making of insurance contracts, agreements or transactions or that no Sec. 299. No insurance company doing business in the Philippines, nor any
separate or direct consideration is received therefor, shall not be deemed agent  thereof, shall pay any commission or other compensation to any person
conclusive to show that the making thereof does not constitute the doing or for services in obtaining insurance, unless such person shall have first procured
transacting of an insurance business.  from the Commissioner a license to act as an insurance agent of such company
or as an insurance broker as hereinafter provided.

No person shall act as an insurance agent or as an insurance broker in the


3 – EXECUTIVE | Ab Initio Group

24
INSURANCE MIDTERMS (Weeks 1 to 8)
solicitation or procurement of applications for insurance, or receive for services Sec. 414
in obtaining insurance, any commission or other compensation from any Sec. 414. The Insurance Commissioner shall have the duty to see that all laws
insurance company doing business in the Philippines, or any agent thereof, relating to insurance, insurance companies and other insurance matters, mutual
without first procuring a license to act from the Commissioner, which must be benefit associations, and trusts for charitable uses are faithfully executed and to
renewed annually on the first day of January, or within six months thereafter. perform the duties imposed upon him by this Code, and shall, notwithstanding
Such license shall be issued by the Commissioner only upon the written any existing laws to the contrary, have sole and exclusive authority to regulate
application of the person desiring it, such application if for a license to act as the issuance and sale of variable contracts as defined in section two hundred
insurance agent, being approved and countersigned by the company such thirty-two and to provide for the licensing of persons selling such contracts, and
person desires to represent, and shall be upon a form prescribed by the to issue such reasonable rules and regulations governing the same.
Commissioner giving such information as he may require, and upon payment of
the corresponding fee hereinafter prescribed. The Commissioner shall satisfy The Commissioner may issue such rulings, instructions, circulars, orders and
himself as to competence and trustworthiness of the applicant and shall have decision as he may deem necessary to secure the enforcement of the provisions
the right to refuse to issue or renew and to suspend or revoke any such license of this Code, subject to the approval of the Secretary of Finance. Except as
in his discretion. No such license shall be valid after the thirtieth day of June of otherwise specified, decisions made by the Commissioner shall be appealable to
the year following its issuance unless it is renewed. the Secretary of Finance.
Sec. 415
Philamlife v Arnaldo, GR 76452,26 July 1994 Sec. 415. In addition to the administrative sanctions provided elsewhere in this
Code, the Insurance Commissioner is hereby authorized, at his discretion, to
impose upon the insurance companies, their directors and/or officers and/or
FACTS: agents, for any willful failure or refusal to comply with, or violation of any
 Ramon M. Paterno, Jr. sent a letter dated April 17, 1986 to Insurance provision of this Code, or any order, instruction, regulation, or ruling of the
Commissioner alleging certain problems encountered by agents, Insurance Commissioner, or any commission or irregularities, and/or conducting
supervisors, managers and public consumers of the Philippine American business in an unsafe or unsound manner as may be determined by the
Life Insurance Company (Philamlife) Insurance Commissioner, the following:
 During the hearing Ramon stated that the contract of agency is illegal 
 Philamlife through its president De los Reyes contended that the Insurance (a) fines not in excess of five hundred pesos a day; and
Commissioner as a quasi-judicial body cannot rule on the matter (b) suspension, or after due hearing, removal of directors and/or officers and/or
ISSUE:  agents.
1. W/N the Insurance Commissioner has the authority to regulate the business of
insurance - YES  Insurance Commissioner has the authority to regulate the business of
2. W/N the business of insurance covers the contract of agency - NO insurance
2. NO.
 power does not cover the relationship affecting the insurance company
HELD: petition is GRANTED and its agents but is limited to adjudicating claims and complaints filed by
the insured against the insurance company
 While the subject of Insurance Agents and Brokers is discussed under
1. YES. Chapter IV, Title I of the Insurance Code, the provisions of said Chapter
speak only of the licensing requirements and limitations imposed on
Insurance Code insurance agents and brokers.
 Great Pacific Life Assurance Corporation v. Judico, 180 SCRA 445 (1989):
3 – EXECUTIVE | Ab Initio Group

25
INSURANCE MIDTERMS (Weeks 1 to 8)
 insurance company may have two classes of agents who sell its Sec. 7. Anyone except a public enemy may be insured.
insurance policies: 
 (1) salaried employees who keep definite hours and work Filipinas Cia. De Seguros v Christern Huenefeld, GR L-2294,25 May1951
under the control and supervision of the company
- governed by the Contract of Employment and the FACTS:
provisions of the Labor Code Christern obtained from Filipinas a fire insurance policy of P1000,000, covering
 (2) registered representatives, who work on commission merchandise contained in a building located at Binondo. During the Japanese
basis. - governed by the Contract of Agency and the military occupation, the building and insured merchandise were burned. The
provisions of the Civil Code on the Agency respondent its claim under the policy. The total loss suffered by the respondent
was fixed at P92,650.
CONTRACT OF INSURANCE
The petitioner refused to pay the claim on the ground that the policy in favor of
A. What may be insured against the respondent had ceased to be in force on the date the U.S. declared war on
Sec. 3. Any contingent or unknown event, whether past or future, which may Germany with the respondent Corporation being controlled by German subjects
damnify a person having an insurable interest, or create a liability against him, and the petitioner being a company under American jurisdiction (though organized
may be insured against, subject to the provisions of this chapter. by Philippine laws) when the policy was issued on October 1, 1941. The petitioner,
The consent of the husband is not necessary for the validity of an insurance policy however, paid to the respondent the sum of P92,650 on April 19, 1943 under
taken out by a married woman on her life or that of her children. orders from the military government.
Any minor of the age of eighteen years or more, may, notwithstanding such
minority, contract for life, health and accident insurance, with any insurance The insurer filed for a suit to recover the sum. The contention was that the policy
company duly authorized to do business in the Philippines, provided the insurance ceased to be effective because of the outbreak of the war and that the payment
is taken on his own life and the beneficiary appointed is the minor's estate or the made by the petitioner to the respondent corporation during the Japanese military
minor's father, mother, husband, wife, child, brother or sister. occupation was under pressure.
The married woman or the minor herein allowed to take out an insurance policy
may exercise all the rights and privileges of an owner under a policy. The tiral and the appellate courts dismissed the action. The Court of Appeals
All rights, title and interest in the policy of  insurance taken out by an original claimed that a corporation is a citizen of the country or state by and under the
owner on the life or health of a minor shall automatically vest in the minor upon laws of which it was created or organized.
the death of the original owner, unless otherwise provided for in the policy.
Hence this appeal.
Sec. 4. The preceding section does not authorize an insurance for or against the
drawing of any lottery, or for or against any chance or ticket in a lottery drawing a ISSUE: Whether the policy in question became null and void upon the
prize. declaration of war

B. Parties to insurance contracts HELD: Yes. Petition granted.

1. Who may be an insurer RATIO:


Sec. 6. Every person, partnership, association, or corporation duly authorized to The majority of the stockholders of the respondent corporation were German
transact insurance business as elsewhere provided in this code, may be an insurer. subjects. The respondent became an enemy corporation upon the outbreak of the
war. The English and American cases relied upon by the Court of Appeals have lost
2. Who may be insured their force in view of the latest decision of the Supreme Court of the United States
3 – EXECUTIVE | Ab Initio Group

26
INSURANCE MIDTERMS (Weeks 1 to 8)
in Clark vs. Uebersee Finanz Korporation where the controls test has been FACTS:
adopted. Case 1:
 The life of Arcadio Constantino was insured with Asia Life
Measures of blocking foreign funds, the so called freezing regulations, and other Insurance Company (Asia) for a term of 20 years with Paz
administrative practice in the treatment of foreign-owned property in the United Lopez de Constantino as beneficiary.  The first premium
States allowed to large degree the determination of enemy interest in domestic covered the period up to September 26, 1942.  
corporations and thus the application of the control test. In Clark vs. Uebersee, the  After the first premium, no further premiums were paid.
court held that “The property of all foreign interest was placed within the reach of The insured died on September 22, 1944.
the vesting power (of the Alien Property Custodian) not to appropriate friendly or  Asia Life Insurance Company, being an American
neutral assets but to reach enemy interest which masqueraded under those Corp., had to close its branch office in Manila by reason of
innocent fronts. . . . The power of seizure and vesting was extended to all property the Japanese occupation, i.e. from January 2, 1942, until
of any foreign country or national so that no innocent appearing device could the year 1945.
become a Trojan horse.”  Case 2:
 Spouses Tomas Ruiz and Agustina Peralta.  Their premium
The Philippine Insurance Law states that “anyone except a public enemy may be were initially annually but subsequently changed to
insured.” It stands to reason that an insurance policy ceases to be allowable as quarterly.  The last quarterly premium was delivered
soon as an insured becomes a public enemy. on on November 18, 1941 and it covered the period
until January 31, 1942. 
“All individuals therefore, who compose the belligerent powers, exist, as to each  Upon the Japanese occupation, the insurer and insured
other, in a state of utter exclusion, and are public enemies.” were not able to deal with each other
 Because the insured had borrowed on the policy P234.00
Vance- “In the case of an ordinary fire policy, which grants insurance only from in January, 1941, the cash surrender value of the policy
year, or for some other specified term it is plain that when the parties become was sufficient to maintain the policy in force only up to
alien enemies, the contractual tie is broken and the contractual rights of the September 7, 1942. 
parties, so far as not vested, are lost.”  Tomas Ruiz died on February 16, 1945 with Agustina
Peralta as  beneficiary. Her demand for payment was
The respondent having become an enemy corporation on December 10, 1941, the refused on the ground of non-payment of the premiums.
insurance policy issued in its favor on October 1, 1941, by the petitioner had  Plaintiffs: As beneficiaries, they are entitled to receive the
ceased to be valid and enforceable, and since the insured goods were burned after proceeds of the policies minus all sums due for premiums in
December 10, 1941, and during the war, the respondent was not entitled to any arrears. The non-payment of the premiums was caused by the
indemnity under said policy from the petitioner. The premium must be returned for closing of Asia's offices in Manila during the Japanese occupation
the sake of justice. and the impossible circumstances created by war.
 lower court: absolved Asia
It results that the petitioner is entitled to recover the indemnity paid. However, the
petitioner will be entitled to recover only the equivalent of P92,650 paid on April ISSUE: W/N the insurers still have a right to claim.
19, 1943.
HELD: YES. lower court affirmed.
Constantino v Asia Life, GR L-1669, 31 August 1950  it would seem that pursuant to the express terms of the policy,
non-payment of premium produces its avoidance

3 – EXECUTIVE | Ab Initio Group

27
INSURANCE MIDTERMS (Weeks 1 to 8)
 Forfeitures of insurance policies are not favored, but courts policies contained provisions applicable expressly to wartime
cannot for that reason alone refuse to enforce an insurance days. The logical inference, therefore, is that the parties
contract according to its meaning. contemplated uninterrupted operation of the contract even if
 Nevertheless, inasmuch as the non-payment of premium was the armed conflict should ensue.
consequence of war, it should be excused and should not cause  the fundamental character of the undertaking to pay premiums
the forfeiture of the policy and the high importance of the defense of non-payment thereof,
 3 Rules in case of war: was specifically recognized
 Connecticut Rule  adopt the United States Rule: first policy had no reserve value,
 2 elements in the consideration for which the and that the equitable values of the second had been practically
annual premium is paid: returned to the insured in the form of loan and advance for
 mere protection for the year premium
 privilege of renewing the contract for each
succeeding year by paying the premium for Fidela Sales Gonzaga v Crown Life, GR L4197,20 March 1957
that year at the time agreed upon
 payment of premiums is a condition precedent, the FACTS: 
non-performance would be illegal necessarily  September 26, 1939: Crown Life Insurance Co. whose home office is
defeats the right to renew the contract based in Toronto, Canada issued to Ramon Gonzaga through its branch
 New York Rule - greatly followed by a number of cases office in Manila a 20-year endowment policy for P15,000 which had an
 war between states in which the parties reside annual premium of P591. 
merely suspends the contracts of the life  Payment was only until September 6, 1941 because of the outbreak of the
insurance, and that, upon tender of all premiums war since Crown is an enemy corp. order to be closed during the Japanese
due by the insured or his representatives after the occupation.  However, despite that it offered a privilege to accept
war has terminated, the contract revives and premium payments in the place of its employee in Ermita but of which
becomes fully operative Gonzaga did not avail.
 United States Rule  Through the automatic premium loan clause, it continued until June 12,
 contract is not merely suspended, but is abrogated 1943
by reason of non-payments is peculiarly of the  May 1, 1945: It reopened but still Gonzaga did not pay although there was
essence of the contract a reinstatement clause providing certain conditions within three years from
 it would be unjust to allow the insurer to retain the the date of lapse on application of the insured
reserve value of the policy, which is the excess of  June 27, 1945: Gonzaga died from an accident
the premiums paid over the actual risk carried  Crown refused to pay because of the lapse of premium payment
during the years when the policy had been in force  RTC: against Gonzaga
 The business of insurance is founded on the law of average; that
of life insurance eminently so ISSUE: W/N Gonzaga's widow can claim despite the absence of premium
 contract of insurance is sui generis payment during the outbreak of the war
 Whether the insured will continue it or not is optional with
him. There being no obligation to pay for the premium, HELD: NO. Affirmed
they did not constitute a debt.  Non-payment at the day involves absolute forfeiture is such be the terms
 It should be noted that the parties contracted not only for of the contract
peacetime conditions but also for times of war, because the  failure to notify the postal address during the war is not an excuse
3 – EXECUTIVE | Ab Initio Group

28
INSURANCE MIDTERMS (Weeks 1 to 8)
 There is no duty when the law forbids and there is no obligation ▪ Cherie Palileo (debtor-mortgagor) filed a complaint against Beatriz Cosio
without corresponding right enjoyed by another (creditor-mortgagee) praying that their transaction be one of a loan with an
 opening of an interim office partook of the nature of the privilege to the equitable mortgage to secure the payment of the loan. The original counsel of
policy holders to keep their policies operative rather than a duty to them Cosio Atty. Guerrero being appointed Undersecretary of Foreign Affairs so she
under the contract forgot the date of the trial and she was substituted.

WEEK 5 ▪ it is a loan of P12,000 secured by a "Conditional Sale of Residential Building" with


right to repurchase. After the execution of the contract, Cosio insured in her name
the building with Associated Insurance & Surety Co. against fire.
MORTGAGOR/MORTGAGEE ▪ The building was partly destroyed by fire so she claimed an indemnity of P13,107
Sec. 8. Unless the policy otherwise provides, where a mortgagor of property ▪ Palileo demanded that the amount of insurance proceeds be credited to her loan
effects insurance in his own name providing that the loss shall be payable to the
mortgagee, or assigns a policy of insurance to a mortgagee, the insurance is ▪ RTC: it is a loan with equitable mortgage so the insurance proceeds should be
deemed to be upon the interest of the mortgagor, who does not cease to be a credited to the loan and refund the overpayment.
party to the original contract, and any act of his, prior to the loss, which would
otherwise avoid the insurance, will have the same effect, although the property is ISSUE: W/N Cosio as mortgagee is entitled to the insurance proceeds for
in the hands of the mortgagee, but any act which, under the contract of her own benefit
insurance, is to be performed by the mortgagor, may be performed by the
mortgagee therein named, with the same effect as if it had been performed by the HELD: YES. Modify. collection of insurance proceeds shall not be deemed to
mortgagor. have compensated the obligation of the Palileo to Cosio, but bars the Cosio from
claiming its payment from the Palileo; and Cosio shall pay to Palileo P810
Sec. 9. If an insurer assents to the transfer of an insurance from a mortgagor to a representing the overpayment made by Palileo by way of interest on the loan.
mortgagee, and, at the time of his assent, imposes further obligation on the
assignee, making a new contract with him, the act of the mortgagor cannot affect ▪ When the the mortgagee may insure his interest in the property independently
the rights of said assignee. of the mortgagor , upon the destruction of the property the insurance money paid
to the mortgagee will not inure to the benefit of the mortgagor, and the amount
Sec. 13. Every interest in property, whether real or personal, or any relation due
thereto, or liability in respect thereof, of such nature that a contemplated peril under the mortgage debt remains unchanged. The mortgagee, however, is not
might directly damnify the insured, is an insurable interest. allowed to retain his claim against the mortgagor, but it passes by subrogation to
the insurer, to the extent of the insurance money paid
Sec. 53. The insurance proceeds shall be applied exclusively to the proper
interest of the person in whose name or for whose benefit it is made unless ▪ It is true that there are authorities which hold that "If a mortgagee procures
otherwise specified in the policy. insurance on his separate interest at his own expense and for his own benefit,
without any agreement with the mortgagor with respect thereto, the mortgagor
Palileo v Cosio, GR L-7667,28 November 1955 has no interest in the policy, and is not entitled to have the insurance proceeds
applied in reduction of the mortgage debt " But these authorities merely represent
FACTS: the minority view”

3 – EXECUTIVE | Ab Initio Group

29
INSURANCE MIDTERMS (Weeks 1 to 8)
San Miguel Brewery v Law Union, GR L-14300, 19 January 1920 ▪ a change of interest in any part of a thing insured unaccompanied by a
corresponding change of interest in the insurance, suspends the insurance to an
Lessons Applicable: equivalent extent, until the interest in the thing and the interest in the insurance
▪ Mortgagor (Insurance) are vested in the same person
▪ Measure of Insurable Interest (Insurance)
▪ Effect of Change of Interest in Thing Insured (Insurance) ▪ section 55: ▪ the mere transfer of a thing insured does not transfer the policy,
▪ Effect of transfer of thing insured (Insurance) but suspends it until the same person becomes the owner of both the policy and
Laws Applicable: sec. 16,sec. 19 (now sec. 20),sec. 50,sec.55 (now sec. the thing insured
58) of the Insurance Code (all old law)
▪ Undoubtedly these policies of insurance might have been so framed as to have
FACTS: been "payable to the San Miguel Brewery, mortgagee, as its interest may appear,
▪ In the contract of mortgage, the owner P.D. Dunn had agreed, at his own remainder to whomsoever, during the continuance of the risk, may become the
expense, to insure the mortgaged property for its full value and to indorse owner of the interest insured." (Sec 54, Act No. 2427.) Such a clause would have
the policies in such manner as to authorize the Brewery Company to receive the proved an intention to insure the entire interest in the property, not merely the
proceeds in case of loss and to retain such part thereof as might be necessary to insurable interest of the San Miguel Brewery, and would have shown exactly to
satisfy the remainder then due upon the mortgage debt. Instead, however, of whom the money, in case of loss, should be paid. But the policies are not so
effecting the insurance himself Dunn authorized and requested the Brewery written.
Company to procure insurance on the property in the amount of P15,000 at
Dunn's expense. ▪ The blame for the situation thus created rests, however, with the Brewery rather
than with the insurance companies, and there is nothing in the record to indicate
▪ San Miguel insured the property only as mortgagee. that the insurance companies were requested to write insurance upon the
insurable interest of the owner or intended to make themselves liable to that
▪ Dunn sold the propert to Henry Harding. The insurance was not assigned by extent
Dunn to Harding.
▪ If by inadvertence, accident, or mistake the terms of the contract were not fully
▪ When it was destroyed by fire, the two companies settled with San Miguelto the set forth in the policy , the parties are entitled to have it reformed. But to justify
extent of the mortgage credit. the reformation of a contract, the proof must be of the most satisfactory
character, and it must clearly appear that the contract failed to express the real
▪ RTC: Absolved the 2 companies from the difference Henry Harding is not entitled agreement between the parties
to the difference between the mortgage credit and the face value of the policies.
▪ In the case now before us the proof is entirely insufficient to authorize
▪ Henry Harding appealed. reformation.

ISSUE: Gonzalez LaO v Yek Tong Lin Fire, GR 33131, 13 Dec. 1930
1. W/N San Miguel has insurable interest as mortgagor only to the
extent of the mortgage credit - YES Facts:
2. W/N Harding has insurable interest as owner – NO > Gonzales was issued 2 fire insurance policies by Yek for 100T covering his leaf
tobacco prducts.
HELD: affirmed > They were stored in Gonzales’ building on Soler St ., which on Jan. 11, 1928,
▪ section 19 of the Insurance Act: burned down.
3 – EXECUTIVE | Ab Initio Group

30
INSURANCE MIDTERMS (Weeks 1 to 8)
> Art. 3 of the Insurance policies provided that: “Any nsurance in force upon all or Commission found that the petitioner did not violate Condition 3 as he had no
part of the things unsured must be declared in writing by the insured and he knowledge of the existence of the two fire insurance policies obtained from the
(insured) should cause the company to insert or mention it in the policy. Without PFIC; that it was Cebu Tesing Textiles which procured the PFIC policies w/o
such requisite, such policy will be regarded as null and void and the insured will be informing him or securing his consent; and that Cebu Tesing Textile, as his
deprived of all rights of indemnity in case of loss.” creditor, had insurable interest on the stocks.
> Notwithstanding said provision, Gonzales entered into other insurance contracts.
When he sought to claim from Yek after the fire, the latter denied any liability on Issue: Whether or not Geagonia is prohibited from recovering from the
the ground of violation of Art. 3 of the said policies. Country Bankers ?

> Gonzales however proved that the insurer knew of the other insurance policies Held: A policy may declare that a violation of specified provisions thereof shall
obtained by him long efore the fire, and the insurer did NOT rescind the insurance avoid it,otherwise, the breach of an immaterial provision does not avoid the policy.
polices in question but demanded and collected from the insured the premiums. To constitute a violation of the “other insurance” clause, the other insurance must
be upon the same subject matter, the same interest therein, and the same risk
Issue:
Whether or not Yek is still entitled to annul the contract. Saura lmport & Export v Phil lnt'l Surety, GR L-15184,31 May 1963

Held: FACTS: Saura Import & Export Co Inc., mortgaged to the Phil. National Bank, a
NO. parcel of land.
The action by the insurance company of taking the premiums of the insured ▪ The mortgage was amended to guarantee an increased amount, bringing the
notwithstanding knowledge of violations of the provisions of the policies amounted total mortgaged debt to P37,000
to waiver of the right to annul the contract of insurance. ▪ On the land mortgage is a building owned by Saura Import & Export Co Inc.
which was insured with Philippine International Surety (Insurer) even before the
mortgage contract so it was required to endorse to mortgagee PNB
▪ October 15, 1954: Barely 13 days after the issuance of the fire insurance policy,
the insurer cancelled it. Notice of the cancellation was given to PNB (mortgagee).
Geagonia v. CA, GR 114427,6 February 1995 But Saura (insured) was not informed.
▪ April 6, 1955: The building and all its contents worth P40,685.69 were burned so
Facts: Geagonia, owner of a store, obtained from Country Bankers 1year fire Saura filed a claim with the Insurer and mortgagee Bank
insurance covering the stock trading of dry goods. The policy noted the ▪ RTC: dismissed
requirement that"
ISSUE: W/N Philippine International Surety should be held liable for the
3. The insured shall give notice to the Company of any insurance or insurances claim because notice to only the mortgagee is not substantial
already effected, or which may subsequently be effected, covering any of the
property or properties consisting of stocks in trade , goods in process and/or HELD:YES. Appealed from is hereby reversed.
inventories only hereby insured, xxx Philippine International Surety Co., Inc., to pay Saura Import & Export Co., Inc.,
P29,000
The petitioners’ stocks were destroyed by fire. He then filed a claim which was ▪ It was the primary duty of Philippine International Surety to notify the insured,
subsequently denied because the petitioner’s stocks were covered by two other but it did not
fire insurance policies issued by PFIC. The basis of the private respondent's denial ▪ If a mortgage or lien exists against the property insured, and the policy contains
was the petitioner's alleged violation of Condition 3 of the policy. The Insurance a clause stating that loss, if any, shall be payable to such mortgagee or the holder
3 – EXECUTIVE | Ab Initio Group

31
INSURANCE MIDTERMS (Weeks 1 to 8)
of such lien as interest may appear, notice of cancellation to the mortgagee or until approval of the said application. Enclosed to this letter was a check
lienholder alone is ineffective as a cancellation of the policy to the owner of the representing the full refund of the past payments made by Eulogio,
property. amounting to P25,417.
▪ liability attached principally the insurance company , for its failure to give notice
of the cancellation of the policy to Saura Violeta requested for a reconsideration of her claim and returned the
▪ it is unnecessary to discuss the errors assigned against appellee bank check to Insular Life. Insular Life agreed quarterly basin in the amount
of P8,062 with a grace period of 31 days for the payment of each
Phil. National Bank v CA, GR L-57757,31August 1987 premium subsequent to the first. If any premium was not paid on or
before the due date, the policy would be in default, and if the premium
Facts: Violeta is the widow of the Eulogio Lalican. During his lifetime, remained unpaid until the end of the grace period, the policy would
Eulogio applied for an insurance policy with Insular Life on April 24, automatically lapse and become void to conduct a re-evaluation of
1997 which contained a 20-year endowment variable income package Violeta's claim. Without waiting for the result of the re-evaluation,
flexi plan Violeta filed with the RTC a complaint for death claim benefit alleging
worth P500k with two riders worth P500k each. the Insular Life was engaged in unfair claim settlement practice and
deliberately failed to act with reasonable promptness on her insurance
Violeta was named the primary beneficiary. claim.

Under the terms, Eulogio was to pay premiums on a Eulogio paid the Violeta claims for the P1.5M insurance, plus interest, attorney's fees and
premiums, however he failed to pay the premium due on January 24, cost of suit. Insular Life filed with the RTC an answer with counterclaim
1998, even after the lapse of the grace period of 31 days. Therefore, saying that the insurance claim was rendered void due to non-payment
lapsed and become void. Eulogio submitted to the Cabanatuan District of the premium and countered that Violeta should be ordered to pay
Office of Insular Life an application for reinstatement together with the attorney's fees and expenses of litigation incurred by
payment of the premium due on January 24. Insular Life notified Eulogio
that his application for reinstatement could not be fully processed Insular Life.
because of the unpaid interest thereon. Eulogio was likewise advised by RTC declared that Violeta failed to establish by preponderance of
Malaluan (insurance agent) to pay the premiums that subsequently evidence her cause of action against the defendant. Violeta failed to
became due April 1998 and July 1998, plus interest. establish that the receipt of payment by Malaluan amounted to the
reinstatement of the insurance policy. Violeta filed for motion for
September 17, 1998. Eulogio went to Malaluan's house and paid for the reconsideration but was denied as well; hence she elevated her case for
interest which was received by Malaluan's husband. Later that day, review on Certiorari.
Eulogio died. Without the knowledge of Eulogio's death, Malaluan
forwarded to the Insular Life the application for reinstatement and the Issues: (a) Whether the decision of the court can still be reviewed
payment made by Eulogio. However, Insular Life did not act upon such despite having allegedly attained finality and despite the mode of appeal
reinstatement for they knew already of Eulogio's death. of Violeta erroneous.
(b) Whether the RTC has decided the case on a question of law not in
September 28, 1998, Violeta filed for the insurance claim. Insular Life accord with law and applicable decisions of the Supreme Court.
then informed Violeta in a letter that her claim could not be processed
because the insurance policy had lapsed already and that Eulogio failed Ruling:
to reinstate the same and the payment made done thru Malaluan's Petition lacks merit.
husband was, under the insurance policy, was considered a deposit only
3 – EXECUTIVE | Ab Initio Group

32
INSURANCE MIDTERMS (Weeks 1 to 8)
RTC's decision has long acquired finality for Violeta failed to file a notice principals in the insurance business, excepting mutual benefit associations. Unless
of appeal more than five months after the decision was rendered. the context otherwise requires, the terms shall also include professional reinsurers
defined in section two hundred eighty. "Domestic company" shall include
As to the substantial claim of whether there is insurable interest, the companies formed, organized or existing under the laws of the
Court says that the matter of insurable interest is entirely irrelevant and Philippines. "Foreign company" when used without limitation shall include
the real point of contention herein is whether Eulogio was able to companies formed, organized, or existing under any laws other than those of the
reinstate the lapsed insurance policy on his life before his death. Philippines.

The Court rules in the negative, for the insurance policy is clear on the Art. 2012. Any person who is forbidden from receiving any donation under Article
procedure of the reinstatement of the insurance contract, of which 739 cannot be named beneficiary of a life insurance policy by the person who
Eulogio has failed to accomplish before his death. As provided by the cannot make any donation to him, according to said article. 
policy, insurance shall be deemed reinstated upon the approval of the
insurance policy of the application for reinstatement. The approval Art. 739. The following donations shall be void:
should be made during the lifetime of the insured, in the case at bar, it (1) Those made between persons who were guilty of adultery or concubinage at
wasn’t the time of the donation;
(2) Those made between persons found guilty of the same criminal offense, in
INSURABLE INTEREST consideration thereof;
(3) Those made to a public officer or his wife, descedants and ascendants, by
Sec. 18. No contract or policy of insurance on property shall be enforceable reason of his office.
except for the benefit of some person having an insurable interest in the property In the case referred to in No. 1, the action for declaration of nullity may be
insured. brought by the spouse of the donor or donee; and the guilt of the donor and
donee may be proved by preponderance of evidence in the same action.
Sec. 25. Every stipulation in a policy of insurance for the payment of loss whether
the person insured has or has not any interest in the property insured, or that the
policy shall be received as proof of such interest, and every policy executed by
way of gaming or wagering, is void. lnsular Life v Ebrado, GR L-44059, 28 October 1977
lnsurable interest in life insurance
Lessons Applicable: 
Sec. 10. Every person has an insurable interest in the life and health:  Art. 2011 Civil Code (Insurance)
(a) Of himself, of his spouse and of his children;  Invalid Designation (Insurance)
(b) Of any person on whom he depends wholly or in part for education or support,
or in whom he has a pecuniary interest;
(c) Of any person under a legal obligation to him for the payment of money, or FACTS:
respecting property or services, of which death or illness might delay or prevent  September 1, 1968: Buenaventura Cristor Ebrado was issued by The
the performance; and Insular Life Assurance Co., Ltd., Policy on a whole-life for P5,882.00 with
(d) Of any person upon whose life any estate or interest vested in him depends. a, rider for Accidental Death and designated Carponia T. Ebrado as the
revocable beneficiary in his policy
Sec. 184. For purposes of this Code, the term "insurer" or "insurance  October 21, 1969: Buenaventura was hit by a falling branch and died. 
company" shall include all individuals, partnerships, associations, or corporations,
including government-owned or controlled corporations or entities, engaged as
3 – EXECUTIVE | Ab Initio Group

33
INSURANCE MIDTERMS (Weeks 1 to 8)
 Carponia filed a claim as the designated beneficiary, although she admits (2) Those made between persons found guilty of the same criminal offense, in
that they were merely living as husband and wife without the benefit of consideration thereof;
marriage (3) Those made to a public officer or his wife, descedants and ascendants, by
 Pascuala Vda. de Ebrado also filed her claim as the widow of the deceased reason of his office.
insured
 In doubt as to whom the insurance proceeds shall be paid, the insurer, In the case referred to in No. 1, the action for declaration of nullity may be
The Insular Life Assurance Co., Ltd. commenced an action for Interpleader brought by the spouse of the donor or donee; and the guilt of the donor and
bef. the CFI donee may be proved by preponderance of evidence in the same action. 
 CFI: Carponia was disqualified because of adultery  Common-law spouses are, definitely, barred from receiving donations from
 CA: affirmed CFI decision each other
ISSUE: W/N Carponia is disqualified for violating the Civil Code which  In essence, a life insurance policy is no different from a civil donation
supplements the silent Insurance Code. insofar as the beneficiary is concerned. Both are founded upon the same
consideration: liberality. A beneficiary is like a donee, because from the
premiums of the policy which the insured pays out of liberality, the
HELD: YES. CA affirmed.  beneficiary will receive the proceeds or profits of said insurance. As a
consequence, the proscription in Article 739 of the new Civil Code should
equally operate in life insurance contracts.
 We do not think that a conviction for adultery or concubinage is exacted
before the disabilities mentioned in Article 739 may effectuate.
 requisite proof of common-law relationship between the insured and the
beneficiary has been conveniently supplied by the stipulations between the
parties in the pre-trial conference of the case 

Vda. De Consuegra v GSIS, GR L-28093, 30 January 1971


Civil Code
Art. 2011 Lessons Applicable: Invalid Designation (Insurance)
Art. 2011. The contract of insurance is governed by special laws. Matters not
expressly provided for in such special laws shall be regulated by this Code.  FACTS:
Art. 2012  Jose Consuegra contracted two marriages.
 First with Rosario Diaz
Art. 2012. Any person who is forbidden from receiving any donation under
 2 children who predeceased their father: Jose Consuegra,
Article 739 cannot be named beneficiary of a life insurance policy by the person
Jr. and Pedro Consuegra
who cannot make any donation to him, according to said article.
 Second with Basilia Berdin while marriage is still subsisting
Art. 739
 7 children: Juliana, Pacita, Maria Lourdes, Jose, Rodrigo,
Art. 739. The following donations shall be void: Lenida and Luz, all surnamed Consuegra
 Consuegra did not designate any beneficiary who would receive the
(1) Those made between persons who were guilty of adultery or concubinage at retirement insurance benefits due to him
the time of the donation;

3 – EXECUTIVE | Ab Initio Group

34
INSURANCE MIDTERMS (Weeks 1 to 8)
 Rosario Diaz filed a claim with the GSIS asking that the retirement Southern Luzon Employees v Golpeo, GR L-6114, 30 October 1954
insurance benefits be paid to her as the only legal heir of Consuegra,
considering that the deceased did not designate any beneficiary  Lessons Applicable: Invalid Designation (Insurance)
 GSIS: legal heirs were Rosario Diaz (1/2 or 8/16), Basilia Berdin and their
seven children (1/2 or 8/16) (1/16 each)
 Basilia Berdin and her children filed petition for mandamus with FACTS:
preliminary injunction in CFI  Roman A. Concepcion listed as his beneficiaries Aquilina Maloles,
 RTC: dismissed the case Roman M. Concepcion, Jr., Estela M. Concepcion, Rolando M.
 Basilia Berdin and her children appealed contending that because the Concepcion and Robin M. Concepcion for the death benefit of
deceased Jose Consuegra failed to designate the beneficiaries in his an association amounting to P2,505
retirement insurance, the appellants who were the beneficiaries named in  Two sets of claimants presented themselves:
the life insurance should automatically be considered the beneficiaries to  Juanita Golpeo, legal wife and her children, named
receive the retirement insurance benefits (Thus, excluding Rosario Diaz) beneficiaries by the deceased
ISSUE: W/N the beneficiaries named in the life insurance should  Marcelino and Josefina Concepcion intervened in
automatically be considered the beneficiaries to receive the retirement their own right aligning themselves Juanita Golpeo
insurance benefits and her minor children
 Elsie Hicban, another common law wife and her child
 RTC:Aquilina Maloles and her children the sole beneficiaries
HELD: NO. appealed from is affirmed  Only the Juanita Golpeo and her minor children and the
 The insured in a life insurance may designate any person as beneficiary intervenors Marcelino and Josefina Concepcion have appealed to
unless disqualified to be so under the provisions of the Civil Code this court
 And in the absence of any beneficiary named in the life insurance policy, ISSUE: W/N Aquilina Molales common-law wife and her illegitimate
the proceeds of the insurance will go to the estate of the insured children can claim the benefits
 Retirement insurance is primarily intended for the benefit of the employee
— to provide for his old age, or incapacity, after rendering service in the
government for a required number of years. If the employee reaches the HELD:  YES.
age of retirement, he gets the retirement benefits even to the exclusion of  Juanita Golpeo, by her silence and actions, had acquiesced in the illicit
the beneficiary or beneficiaries named in his application for retirement relations between her husband and appellee Aquilina Maloles
insurance. The beneficiary of the retirement insurance can only claim the  new Civil Code recognized certain successional rights of illegitimate
proceeds of the retirement insurance if the employee dies before children
retirement.  If the employee failed or overlooked to state the beneficiary
of his retirement insurance, the retirement benefits will accrue to his
estate and will be given to his legal heirs in accordance with law, as in the Heirs of Maramag v Maramag, GR 181132,5 June 2009
case of a life insurance if no beneficiary is named in the insurance policy.
Lessons Applicable: To whom insurance proceeds payable (Insurance)
 with respect to the right of the second wife, although the second marriage
can be presumed to be void ab initio as it was celebrated while the first
marriage was still subsisting, still there is need for judicial declaration of FACTS:
such nullity  Loreto Maramag designated as beneficiary his concubine Eva de Guzman
Maramag
3 – EXECUTIVE | Ab Initio Group

35
INSURANCE MIDTERMS (Weeks 1 to 8)
 Vicenta Maramag and Odessa, Karl Brian, and Trisha Angelie (heirs of Ernani Trinos applied for a health care coverage with Philam. He answered
Loreto Maramag) and his concubine Eva de Guzman Maramag, also no to a question asking if he or his family members were treated to heart
suspected in the killing of Loreto and his illegitimate children are claiming trouble, asthma, diabetes, etc.
for his insurance.
 Vicenta alleges that Eva is disqualified from claiming The application was approved for 1 year. He was also given hospitalization
 RTC: Granted - civil code does NOT apply benefits and out-patient benefits. After the period expired, he was given an
 CA: dismissed the case for lack of jurisdiction for filing beyond expanded coverage for Php 75,000. During the period, he suffered from
reglementary period heart attack and was confined at MMC. The wife tried to claim the benefits but the
ISSUE: W/N Eva can claim even though prohibited under the civil code petitioner denied it saying that he concealed his medical history by answering no
against donation to the aforementioned question. She had to pay for the hospital bills amounting to
76,000. Her husband subsequently passed away. She filed a case in the trial court
for the collection of the amount plus damages. She was awarded 76,000 for the
HELD: YES. Petition is DENIED.  bills and 40,000 for damages. The CA affirmed but deleted awards for damages.
 Any person who is forbidden from receiving any donation under Article Hence, this appeal.
739 cannot be named beneficiary of a life insurance policy of the person
who cannot make any donation to him Issue: WON a health care agreement is not an insurance contract; hence
 If a concubine is made the beneficiary, it is believed that the the “incontestability clause” under the Insurance Code does not apply.
insurance contract will still remain valid, but the indemnity must
go to the legal heirs and not to the concubine, for evidently, what Held: No. Petition dismissed.
is prohibited under Art. 2012 is the naming of the improper
beneficiary. 
 SECTION 53. The insurance proceeds shall be applied exclusively to the Ratio:
proper interest of the person in whose name or for whose benefit it is  Petitioner claimed that it granted benefits only when the insured is alive during
made unless otherwise specified in the policy. the one-year duration. It contended that there was no indemnification unlike in
 GR: only persons entitled to claim the insurance proceeds are insurance contracts. It supported this claim by saying that it is a health
either the insured, if still alive; or the beneficiary, if the insured is maintenance organization covered by the DOH and not the Insurance Commission.
already deceased, upon the maturation of the policy. Lastly, it claimed that the Incontestability clause didn’t apply because two-year
 EX: situation where the insurance contract was intended to benefit and not one-year effectivity periods were required.  
third persons who are not parties to the same in the form of Section 2 (1) of the Insurance Code defines a contract of insurance as “an
favorable stipulations or indemnity. In such a case, third parties agreement whereby one undertakes for a consideration to indemnify another
may directly sue and claim from the insurer against loss, damage or liability arising from an unknown or contingent event.”
 It is only in cases where the insured has not designated any beneficiary, Section 3 states: every person has an insurable interest in the life and health:
or when the designated beneficiary is disqualified by law to receive the (1)     of himself, of his spouse and of his children.
proceeds, that the insurance policy proceeds shall redound to the benefit In this case, the husband’s health was the insurable interest. The health care
of the estate of the insured agreement was in the nature of non-life insurance, which is primarily a contract of
indemnity. The provider must pay for the medical expenses resulting from sickness
Philamcare v CA, GR 125678, 18 March 2002 or injury.
While petitioner contended that the husband concealed materialfact of his
Facts: sickness, the contract stated that:

3 – EXECUTIVE | Ab Initio Group

36
INSURANCE MIDTERMS (Weeks 1 to 8)
“that any physician is, by these presents, expressly authorized to disclose or give
testimony at anytime relative to any information acquired by him in Nario V Phil Am Life, GR l-22796, 26 June 1967
his professional capacity upon any question affecting the eligibility for health care
coverage of the Proposed Members.” FACTS:
This meant that the petitioners required him to sign authorization to furnish  June 12, 1959: Philippine American Life Insurance Co. issued a life
reports about his medical condition. The contract also authorized Philam to inquire insurance to Mrs. Alejandra Santos-Mario a life insurance policy under a
directly to his medical history. 20-year endowment plan, with a face value of P5,000 designating her
Hence, the contention of concealment isn’t valid. husband Delfin Nario and their unemancipation son Ernesto Nario, as her
They can’t also invoke the “Invalidation of agreement” clause where failure of the irrevocable beneficiaries
insured to disclose information was a grounds for revocation simply because the  June, 1963: She submitted her loan application to the life insurance co.
answer assailed by the company was the heart condition question based on the with signature of her husband in two capacities:
insured’s opinion. He wasn’t a medical doctor, so he can’t accurately gauge his  irrevocable beneficiaries
condition.  father-guardian of minor irrevocable beneficiary Ernesto
Henrick v Fire-  “in such case the insurer is not justified in relying upon such  Insurance Co. denied asking that the legal guardian must be authorized by
statement, but is obligated to make further inquiry.” the court in a competent guardianship proceeding
Fraudulent intent must be proven to rescind the contract. This was incumbent  Upon denial, she opted to surrender her insurance policy in exchange of
upon the provider. its cash surrender value of P520 but it was also denied on the same
“Having assumed a responsibility under the agreement, petitioner is bound to ground
answer the same to the extent agreed upon.  In the end, the liability of the health  September 10, 1963: Mrs. Alejandra Santos-Nario and her husband, Delfin
care provider attaches once the member is hospitalized for the disease or injury Nario, brought suit against the Philippine American Life Insurance Co
covered by the agreement or whenever he avails of the covered benefits which he  RTC: favored the insurance company
has prepaid.”  CA: vested interest or right of the beneficiaries in the policy should be
Section 27 of the Insurance Code- “a concealment entitles the injured party to measured on its full face value and not on its cash surrender value, for in
rescind a contract of insurance.” case of death of the insured, said beneficiaries are paid on the basis of its
As to cancellation procedure- Cancellation requires certain conditions: face value and in case the insured should discontinue paying premiums,
1.       Prior notice of cancellation to insured; the beneficiaries may continue paying it and are entitled to automatic
2.       Notice must be based on the occurrence after effective date of the policy of extended term or paid-up insurance options, etc. and that said vested
one or more of the grounds mentioned; right under the policy cannot be divisible at any given time. policy loan
3.       Must be in writing, mailed or delivered to the insured at the address shown and surrender of policy constitute acts of disposition or alienation of
in the policy; property rights and not merely of management or administration because
4.       Must state the grounds relied upon provided in Section 64 of the Insurance they involve the incurring or termination of contractual obligations
Code and upon request of insured, to furnish facts on which cancellation is based ISSUE: W/N parents as guardians can enter into transactions for the
None were fulfilled by the provider. benefit of  minor irrevocable beneficiaries.
As to incontestability- The trial court said that “under the title Claim procedures of
expenses, the defendant Philamcare Health Systems Inc. had twelve months from HELD: NO. Affirmed.
the date of issuance of the Agreement within which to contest the membershipof  SEC. 7. Parents as guardians. — When the property of the child
the patient if he had previous ailment of asthma, and six months from the under parental authority is worth two thousand pesos or less, the
issuance of the agreement if the patient was sick of diabetes or hypertension. The father or the mother, without the necessity of court appointment,
periods having expired, the defense of concealment or misrepresentation no shall be his legal guardian. When the property of the child is
longer lie. worth more than two thousand pesos, the father or the mother
3 – EXECUTIVE | Ab Initio Group

37
INSURANCE MIDTERMS (Weeks 1 to 8)
shall be considered guardian of the child's property, with the exercised, or agreement made with the Company to any change in or amendment
duties and obligations of guardians under these rules, and shall to the Policy, without the consent of the said beneficiary/beneficiaries.”
file the petition required by Section 2 hereof. For good reasons The alleged acquiescence of the six (6) children beneficiaries of the policy cannot
the court may, however, appoint another suitable person. be considered an effective ratification due to the fact that they were minors.
 even if worth less than P2,000 parent's authority over the estate Neither could they act through their father insured since their interests are
of the ward as a legal-guardian would not extend to acts of quite divergentfrom one another.
encumbrance or disposition, as distinguished from acts of Therefore, the parent-insured cannot exercise rights and/or privileges pertaining to
management or administration. the insurance contract, for otherwise, the vested rights of the
irrevocable beneficiaries would be rendered inconsequential.  
Phil Am Life v Pineda, GR L-54216, 19 July 1989 Of equal importance is the well-settled rule that the contract between
the parties is the law binding on both of them and for so many times, this court
Facts: has consistently issued pronouncements upholding the validity and effectivity of
Pineda  procured an ordinary life insurance policy from the petitioner company and contracts. Likewise, contracts which are the private laws of the
designated his wife and children as irrevocable beneficiaries. contracting parties should be fulfilled according to the literal sense of their
He then filed a petition to amend the designation of the beneficiaries in his life stipulations, for contracts are obligatory, no matter in what form they may be,
policy from irrevocable to revocable. whenever the essential requisites for their validity are present
The judge granted the request. The change in the designation of was not within the contemplation of the parties.
Petitioner promptly filed a motion but was denied. Hence, this petition. The lower court instead made a new contract for them. It acted in excess of its
authority when it did so.

Issues:
1. WON the designation of the irrevocable beneficiaries could be Villanueva v Oro, GR L-2227, 31 August 1948
changed or amended without the consent of all the
irrevocable beneficiaries. FACTS:
2. WON the irrevocable minor beneficiaries could give consent to the
change in designation  West Coast Life Insurance Company issued 2 policies of insurance on the
life of Esperanza J. Villanueva:
Held: No to both. Petition dismissed.  2,000 php - maturing on April 1, 1943
 if living, on the 1st day of April 1943 - to insured
Ratio:  upon death during the continuance of this policy - to the
Under the Insurance Act, the beneficiary designated in a life insurance contract beneficiary Bartolome Villanueva, father of the
cannot be changed without the consent of the beneficiary because he has a vested insured, with right on the part of the insured to change
interest in the policy. the beneficiary
There was an express stipulation to this effect: “It is hereby understood and  1940: Bartolome Villanueva died, Mariano J.
agreed that, notwithstanding the provisions of this policy to the contrary, Villanueva duly substituted as beneficiary, a
inasmuch as the designation of the primary/contingent beneficiary/beneficiaries in brother of the insured
this Policy has been made without reserving the right to change  3,000 php - maturing on March 31, 1943
said beneficiary/ beneficiaries, such designation may not be surrendered to the  Esperanza J. Villanueva survived the insurance period, for she died only on
Company, released or assigned; and no right or privilege under the Policy may be October 15, 1944, without, however, collecting the insurance proceeds.
 CFI: estate of the insured Esperanza is entitled to the insurance proceeds
3 – EXECUTIVE | Ab Initio Group

38
INSURANCE MIDTERMS (Weeks 1 to 8)
ISSUE: W/N the estate of insured Esperanza should be entitled to the Sec. 16. A mere contingent or expectant interest in anything, not founded on an
insurance proceeds since she outlived the insurance policy actual right to the thing, nor upon any valid contract for it, is not insurable.

HELD: YES. appealed order is, therefore, hereby affirmed Sec. 17. The measure of an insurable interest in property is the extent to which
 To sustain the beneficiary's claim would be altogether eliminate from the the insured might be damnified by loss or injury thereof.
policies the condition that the insurer "agrees to pay . . . to the insured
hereunder, if living Sec. 18. No contract or policy of insurance on property shall be enforceable
 Upon the insured's death, within the period, the beneficiary will take, as except for the benefit of some person having an insurable interest in the property
against the personal representative or the assignee of the insured. Upon insured.
the other hand, if the insured survives the endowment period, the benefits
are payable to him or to his assignee, notwithstanding a beneficiary is Sec. 25. Every stipulation in a policy of insurance for the payment of loss whether
designated in the policy the person insured has or has not any interest in the property insured, or that the
policy shall be received as proof of such interest, and every policy executed by
way of gaming or wagering, is void.

Harvardian College v Gountry Bankers (CA), CA CV 03771, 6 Jan. 6,


1986, CARA 1

Facts:
WEEK 6 >  Harvardian is a family corporation, the stockholders of which are Ildefonso Yap,
Virginia King Yap and their children.
>  Prior to Aug. 9, 1979, an agent of Country Bankers proposed to Harvardian to
lnsurable interest in property insurance insure its school building.  Although at first reluctant, Harvardian agreed.
>  Country Banks sent an inspector to inspect the school building and agreed to
Sec. 13. Every interest in property, whether real or personal, or any relation insure the same for P500,000 for which Harvardian paid an annual premium of
thereto, or liability in respect thereof, of such nature that a contemplated peril P2,500.
might directly damnify the insured, is an insurable interest. >  On Aug. 9, 1979, Country Bankers issued to Harvardian a fire insurance policy. 
On March 12, 1980, (39 days before I was born… hehehehe ) during the effectivity
Sec. 14. An insurable interest in property may consist of said insurance policy, the insured property was totally burned rendering it a
in:chanroblesvirtuallawlibrary total loss.
(a) An existing interest; >  A claim was made by plaintiff upon defendant but defendant denied it
(b) An inchoate interest founded on an existing interest; or contending that plaintiff had no insurable interest over the building constructed on
(c) An expectancy, coupled with an existing interest in that out of which the the piece of land in the name of the late Ildefonso Yap as owner.
expectancy arises. >  It was contended that both the lot and the building were owned by Ildefonso
Yap and NOT by the Harvardian Colleges.
Sec. 15. A carrier or depository of any kind has an insurable interest in  a thing
held by him as such, to the extent of his liability but not to exceed the value Issue:
thereof. Whether or not Harvardian colleges has a right to the proceeds.

Held:
3 – EXECUTIVE | Ab Initio Group

39
INSURANCE MIDTERMS (Weeks 1 to 8)
Harvardian has a right to the proceeds.  RTC: IMC and LSPI retained ownership of the delivered goods until fully
Regardless of the nature of the title of the insured or even if he did not have title paid, it must bear the loss (res perit domino)
to the property insured, the contract of fire insurance should still be upheld if his  CA: Reversed - sales invoices is an exception under Article 1504 (1) of the
interest in or his relation to the property is such that he will be benefited in its Civil Code to res perit domino
continued existence or suffer a direct pecuniary loss from its destruction or injury.  ISSUE: W/N Insurance Company of North America can claim against
The test in determining insurable interest in property is whether one will derive Gaisano Cagayan for the debt that was insured
pecuniary benefit or advantage from its preservation, or will suffer pecuniary loss
or damage from its destruction, termination or injury by the happening of the HELD: YES. petition is partly GRANTED. order to pay P535,613 is
event insured against. DELETED
 insurance policy is clear that the subject of the insurance is the book debts
Here Harvardian was not only in possession of the building but was in fact using and NOT goods sold and delivered to the customers and dealers of the
the same for several years with the knowledge and consent of Ildefonso Yap.  It is insured
reasonably fair to assume that had the building not been burned, Harvardian  ART. 1504. Unless otherwise agreed, the goods remain at the seller's risk
would have been allowed the continued use of the same as the site of its until the ownership therein is transferred to the buyer, but when the
operation as an educational institution.  Harvardian therefore would have been ownership therein is transferred to the buyer the goods are at the buyer's
directly benefited by the preservation of the property, and certainly suffered a risk whether actual delivery has been made or not, except that:
pecuniary loss by its being burned.
(1) Where delivery of the goods has been made to the buyer or to a bailee for the
buyer, in pursuance of the contract and the ownership in the goods has been
retained by the seller merely to secure performance by the buyer of his obligations
Gaisan'o v lnsurance Co. of North America, GR 147839, I June 2006 under the contract, the goods are at the buyer's risk from the time of such
delivery;
FACTS:  IMC and LSPI did not lose complete interest over the goods. They have an
 Intercapitol Marketing Corporation (IMC) is the maker of Wrangler Blue insurable interest until full payment of the value of the delivered goods.
Jeans. while Levi Strauss (Phils.) Inc. (LSPI) is the local distributor of Unlike the civil law concept of res perit domino, where ownership is the
products bearing trademarks owned by Levi Strauss & Co basis for consideration of who bears the risk of loss, in property insurance,
 IMC and LSPI separately obtained from Insurance Company of North one's interest is not determined by concept of title, but whether insured
America fire insurance policies for their book debt endorsements related to has substantial economic interest in the property
their ready-made clothing materials which have been sold or delivered to  Section 13 of our Insurance Code defines insurable interest as "every
various customers and dealers of the Insured anywhere in the Philippines interest in property, whether real or personal, or any relation thereto, or
which are unpaid 45 days after the time of the loss liability in respect thereof, of such nature that a contemplated peril might
 February 25, 1991: Gaisano Superstore Complex in Cagayan de Oro City, directly damnify the insured." Parenthetically, under Section 14 of the
owned by Gaisano Cagayan, Inc., containing the ready-made clothing same Code, an insurable interest in property may consist in: (a) an
materials sold and delivered by IMC and LSPI was consumed by fire.  existing interest; (b) an inchoate interest founded on existing interest; or
 February 4, 1992: Insurance Company of North America filed a complaint (c) an expectancy, coupled with an existing interest in that out of which
for damages against Gaisano Cagayan, Inc. alleges that IMC and LSPI filed the expectancy arises. 
their claims under their respective fire insurance policies which it paid thus  Anyone has an insurable interest in property who derives a benefit from its
it was subrogated to their rights existence or would suffer loss from its destruction.
 Gaisano Cagayan, Inc: not be held liable because it was destroyed
due to fortuities event or force majeure
3 – EXECUTIVE | Ab Initio Group

40
INSURANCE MIDTERMS (Weeks 1 to 8)
 it is sufficient that the insured is so situated with reference to the insurer (pp. 8-9, Record on Appeal). American Smelting and Refining Co., Ltd.
property that he would be liable to loss should it be injured or (Asarco) was the consignee. The ore was to be discharged at the wharf of Asarco’s
destroyed by the peril against which it is insured smelter at Tacoma (pp. 75-76, 98-9, Record on Appeal).
 an insurable interest in property does not necessarily imply a
property interest in, or a lien upon, or possession of, the subject  Respondent company shipped to a consignee in the United States certain cargoes
 matter of the insurance, and neither the title nor a beneficial covered by two "all risks" marine insurance policies issued by petitioners
interest is requisite to the existence of such an interest containing express stipulations that respondent company has an interest therein.
 insurance in this case is not for loss of goods by fire but for petitioner's The shipments, which were undertaken in accordance with the instructions of the
accounts with IMC and LSPI that remained unpaid 45 days after the fire insurer’s surveyor, sustained damage in transit prompting private respondent to
- obligation is pecuniary in nature file the corresponding insurance claims which were rejected. Consequently,
 obligor should be held exempt from liability when the loss occurs respondent company filed with the Court of First Instance a complaint for recovery
thru a fortuitous event only holds true when the obligation of damages which was dismissed for lack of cause of action. On appeal, the Court
consists in the delivery of a determinate thing and there is no of Appeals reversed the Commercial Union Assurance Company Limited, Et. Al. v.
stipulation holding him liable even in case of fortuitous event Lepanto Consolidated Mining Company, Et. Al. trial court’s order of dismissal.
 Article 1263 of the Civil Code in an obligation to deliver a generic thing, Hence, this petition for certiorari (herein treated as an appeal) wherein petitioners
the loss or destruction of anything of the same kind does not extinguish contend, among others, that respondent company is not the real party in interest
the obligation (Genus nunquan perit) and has no personality to sue and that respondent’s complaint has no cause of
 The subrogation receipt, by itself, is sufficient to establish not only the action against the insurers. 
relationship of respondent as insurer and IMC as the insured, but also the
amount paid to settle the insurance claim On review, the Supreme Court, without prejudging the merits of respondent’s case
 Art. 2207. If the plaintiff's property has been insured, and he has received and petitioner’s affirmative defenses, held that there is prima facie showing in
indemnity from the insurance company for the injury or loss arising out of respondent’s complaint and pleadings that it is a real party in interest under the
the wrong or breach of contract complained of, the insurance company policies and that it has a cause of action against petitioners as insurers.
shall be subrogated to the rights of the insured against the wrongdoer or
the person who has violated the contract.  ISSUE: WON Lepanto can legally sue on the marine insurance policies.
 As to LSPI, no subrogation receipt was offered in evidence. 
 Failure to substantiate the claim of subrogation is fatal to HELD: YES. Where, based (1) on express stipulation in the two subject marine
petitioner's case for recovery of the amount of P535,613 insurance policies that respondent company has an interest therein and (2) on the
facts that it was the shipper (and presumably the owner) of the insured cargoes,
Commercial Union Assurance v. Lepanto, GR L-52027,27 April 1982 that the shipments were undertaken in accordance with the instructions of the
insurer’s marine surveyor and that it was respondent company that filed the
FACTS: This is a marine insurance case. Lepanto Consolidated Mining Company corresponding claim with the adjuster when the cargoes were damaged, the
alleged in its complaint of February 7, 1974 that on November 8 and 23, 1971 it Supreme Court, without prejudging the merits of respondent company’s case and
shipped (for smelting) copper ore concentrates on board the vessels M/S Hermosa petitioners’ affirmative defenses, ruled that there is prima facie showing in
and M/S General Aguinaldo from Poro Point, San Fernando, La Union to Tacoma, respondent’s complaint and pleadings that it is a real party in interest under the
Washington, U.S.A. policies and that it has a cause of action against the petitioners as insurers.

The first shipment is known as No. 167 and the other shipment as Nos. 168 and Harding v Commercial Union, GR L-12707,10 August 1918
168-A. The copper ore concentrates were stored on board the carrying vessels
under the supervision and approval of a marine surveying firm designated by the Facts:
3 – EXECUTIVE | Ab Initio Group

41
INSURANCE MIDTERMS (Weeks 1 to 8)
 Henry Harding bought a car for 2T in 1915.  He then gave the car to his  RTC: favored Jose ordering Barreto to pay him P1,298.50 and offsetting
wife Mrs. Harding. the P2,000 
 While Mrs. Harding was having the car repaired at the Luneta Garage  Barreto alone appealed
(Luneta was an agent of Smith Bell and Co., which in turn is Commercial
Union’s agent), the latter induced Mrs. Harding to insure the care with Issue: W/N Barreto had insurable interest in the house and could insure
Commercial. it for his it for his own protection
 Mrs. Harding agreed, and Smith Bell sent an agent to Luneta Garage, who
together with the manager of LUneta, appraised the car and declared that Held:
its present value was P3T.  This amt was written in the proposal form YES. Reversed and Barretto is absolved
which Mrs. Harding signed.  Where different persons have different interests in the same property, the
 Subsequently, the car was damaged by fire.  Commercial refused to pay insurance taken by one in his own right and in his own interest does not in
because the car’s present value was only 2.8T and not 3T. any way insure to the benefit of another
   A contract of insurance made for the insurer's (insured) indemnity only, as
Issue: Whether or not Commercial is liable. where there is no agreement, express or implied, that it shall be for the
  benefit of a third person, does not attach to or run with the title to the
Held: insured property on a transfer thereof personal as between the insurer
Commercial is liable. and the insured. 
Where it appears that the proposal form, while signed by the insured was made  Barretto had an insurable interest in the house. He construed the building,
out by the person authorized to solicit the insurance (Luneta and Smith Bell) the furnishing all the materials and supplies, and insured it after it had been
facts stated in the proposal, even if incorrect, will not be regarded as warranted by completed 
the insured, in the absence of willful misstatement.  Under such circumstances,
the proposal is to be regarded as the act of the insurer. Ong Lim Sing, Jr. v FEB Leasing, GR 168115, 8 June 2007

Lampano v Jose, GR L-9401, 30 March 1915 Facts:


 FEB Leasing and Finance Corporation (FEB) leased equipment and motor
FACTS: vehicles to JVL Food Products with a monthly rental of P170,494
 Mariano R. Barretto, constructed a house for Placida A. Jose sold the  At the same date, Vicente Ong Lim Sing, Jr. (Lim) an executed an
house to Antonina Lampano for P6,000  Individual Guaranty Agreement with FEB to guarantee the prompt and
 The house was destroyed by fire during which Lampano still owed Jose faithful performance of the terms and conditions of the lease agreement
P2, 000 as evidenced by a promissory note.  Jose also owed Barretto P2,  JVL defaulted in the payment of the monthly rentals resulting to arrears of
000 for the construction. P3,414,468.75 and refused to pay despite demands
 After the completion of the house and before it was destroyed, Mariano R.  FEB filed a complaint for damages and replevin against JVL, Lim and John
Barretto took out an insurance policy upon it in his own name, with the Doe
consent of Placida A. Jose, for the sum of P4,000. After its destruction, he o JVL and Lim admitted the existence of the lease agreement but
collected P3,600 from the insurance company, having paid in premiums asserted that it is in reality a sale of equipment on installment
the sum of P301.50 basis, with FEB acting as the financier
 Lampano filed a complaint against Barreto and Jose alleging that Jose in a  RTC: Sale on installment and the FEB elected full payment of the
verbal agreement told her that the policy will be delivered to her so she obligation so for the unreturned units and machineries the JVL and Lim
should collected P3,600 from each of them are jointly and severally liable to pay

3 – EXECUTIVE | Ab Initio Group

42
INSURANCE MIDTERMS (Weeks 1 to 8)
 CA: granted FEB appeal that it is a financial lease agreement under
Republic Act  (R.A.) No. 8556 and ordered JVL and Lim jointly and FACTS:
severally to pay P3,414,468.75  Tomas Lianco and the Archbishop entered into a contract of lease on a
parcel of land owned by church 
Issue: W/N JVL and Lim should jointly and severally be liable for the  As lessee, Lianco erected a building on the leased portion of the church’s
insured financial lease land. 
 Lianco transferred ownership of this building to Kaw Eng Si,
Held: who later transferred the same to Golangco.
YES. CA affirmed.  Transfers were made without the consent of the Archbishop
 contract of adhesion is as binding as any ordinary contract  The Archbishop filed an ejectment case against Lianco, who appears to be
 The Lease Contract with corresponding Lease Schedules with Delivery and occupants of the premises building with others paying rent to Golangco.
Acceptance Certificates is, in point of fact, a financial lease within the  The right of Golangco to receive rent on the building was judicially
purview of R.A. No. 8556 recognized in a case decided between Lianco and
o FEB leased the subject equipment and motor vehicles to JVL in others occupying the premises pursuant to a compromise
consideration of a monthly periodic payment of P170, 494.00. agreement.
The periodic payment by petitioner is sufficient to amortize at  The Archbishop did not exercise his option to question Golangco’s rights as
least 70% of the purchase price or acquisition cost of the said lessee
movables in accordance with the Lease Schedules with Delivery  April 7,1949: Golangco applied for fire insurance with Trader’s
and Acceptance Certificates. Insurance and Surety Co. 
 JVL entered into the lease contract with full knowledge of its terms and  fire insurance policy states: "that all insurance covered under said
conditions.   policy, includes the 'rent or other
 Lim, as a lessee, has an insurable interest in the equipment and motor subject matter of insurance in respect of or in connection with any
vehicles leased.   building or any property contained in any building"
 In the financial lease agreement, FEB did not assume responsibility as to  June 5, 1949: the building premises was burned so Golangco requested
the quality, merchantability, or capacity of the equipment.  This stipulation Trader’s Insurance to pay the insurance amount of 10,000 including the
provides that, in case of defect of any kind that will be found by the lessee amount of rent P1,100 monthly. 
in any of the equipment, recourse should be made to the manufacturer.  Trader’s insurance refused to pay the insurance for the
“The financial lessor, being a financing company, i.e., an extender of rent averring that Golangco has no insurable interest 
credit rather than an ordinary equipment rental company, does not extend
a warranty of the fitness of the equipment for any particular use.  Thus, ISSUE: W/N Golangco has insurable interest
the financial lessee was precisely in a position to enforce such warranty on the rent of the building premises which may lawfully/validly be
directly against the supplier of the equipment and not against the financial subject of insurance?
lessor.  We find nothing contra legem or contrary to public policy in such a
contractual arrangement
HELD: YES.
Traders lnsurance v Golangco, GR L-6442,21 September 1954  Sec. 13 of the Insurance Code 
Every interest in the property, whether real or personal, or any relation thereto,
Lessons Applicable: Existing Interest (Insurance) or liability in respect thereof of such nature that a contemplated peril might
Laws Applicable: Sec. 13 of the Insurance Code  directly damnify the insured, is an insurable interest.

3 – EXECUTIVE | Ab Initio Group

43
INSURANCE MIDTERMS (Weeks 1 to 8)
 Both at the time of the issuance of the policy and at the time of the fire, Co., Ltd., and the British Traders Insurance Co., Ltd., in the amount of
Golangco was in legal possession of the premises, collecting rentals from P404,800.
its occupant.
 The argument of Trader’s Insurance that a policy of insurance must 4. All the policies were in the name of Sra. Benita Quiogue de V. del Rosario,
specify the interest of the insured in the property insured, if he is not the with the exception of one of the National Insurance Company, Inc., for
absolute owner thereof, is not meritorious because it was the Trader’s, not P40,000, in favor of the Compañia Coprera de Tayabas.
Golangco, who prepared that policy, and it cannot take advantage of its
own acts to plaintiff's detriment; and, in any case, this provision was 5. The warehouse of Mrs. Del Rosario and its contents were destroyed by fire
substantially complied with by Golangco when he made a full and clear on June 6, 1920. The warehouse was a total loss, while of the copra
statement of his interests to Trader's manager.   stored therein, only an amount equal to P49,985 was salvaged.
 The contract between Lianco and the Archbishop only forbade
Lianco from transferring 'his rights as LESSEE but the contracts 6. When Bayne, a fire loss adjuster, failed to effect a settlement between the
Lianco made in favor of Kaw Eng Si and plaintiff Golangco did not Insurance companies and Del Rosario, the latter authorized Atty. Fisher to
transfer such rights; hence no written consent thereto was negotiate with the Companies. An agreement was reached to submit the
necessary. At worst, the contract would be voidable, but not a matter to arbitration. The arbitrators in their report allowed Mrs. Del
void contract, at the option of the Archbishop and it does not Rosario P363,610, which, with the addition of the money received from
appear that it was ever exercised the salvaged copra amounting to P49,985, and interest, made a total of
Lopez v del Rosario, GR L-19189,27 November 1922 P414,258, collected by her from the companies.

DOCTRINE: The law is that a policy effected by bailee and covering by its 7. The claims by different people who had stored copra in the warehouse
terms his own property and property held in trust; inures, in the event of were settled with the exception of Froilan Lopez. Ineffectual attempts by
a loss, equally and proportionately to the benefit of all the owners of the Mrs. Del Rosario to effect a compromise with Lopez first for P71,994, later
property insured. raised to P72,724, and finally reduced to P17,000, were made. Lopez
stubbornly contended that he should receive not a centavo less than
FACTS: P88,595.43.

1. Mrs. Del Rosario, was the owner of a bonded warehouse situated in the ISSUE: Whether the plaintiff Lopez is entitled to receive insurance
City of Manila. She was engaged in the business of a warehouse keeper, benefits from the defendant.
and stored copra and other merchandise in the said building. Among the
persons who had copra deposited in the Del Rosario warehouse was HELD: YES. The SC noted in two documents, one the agreement for
Froilan Lopez, the holder of fourteen warehouse receipts in his own name, arbitration, and the other the statement of claim of Mrs. Del Rosario
and the name of Elias T. Zamora. against the insurance companies, she acknowledged her responsibility
to the owners of the stored merchandise, against risk of loss by fire.
2. The warehouse receipts, or negotiable warrants, or quedans (as they are
variously termed) of Lopez named a declared value of P107,990.40. The award of the arbitrators covered not Mrs. Del Rosario's warehouse
alone but the products stored in the warehouse by Lopez and others.
3. Mrs. Del Rosario secured insurance on the warehouse and its contents
with the National Insurance Co., Inc., the Commercial Union Insurance RATIO:
Company, the Alliance Insurance Company, the South British Insurance

3 – EXECUTIVE | Ab Initio Group

44
INSURANCE MIDTERMS (Weeks 1 to 8)
 The law is that a policy effected by bailee and covering by its terms his Lessons Applicable: 
own property and property held in trust; inures, in the event of a loss,  Mortgagor (Insurance)
equally and proportionately to the benefit of all the owners of the property  Measure of Insurable Interest (Insurance) 
insured.  Effect of Change of Interest in Thing Insured (Insurance)
 Effect of transfer of thing insured (Insurance)
 Even if one secured insurance covering his own goods and goods stored Laws Applicable: sec. 16,sec. 19 (now sec. 20),sec. 50,sec.55 (now sec.
with him, and even if the owner of the stored goods did not request or 58) of the Insurance Code (all old law)
know of the insurance, and did not ratify it before the payment of the loss,
yet it has been held by a reputable court that the warehouseman is liable FACTS:
to the owner of such stored goods for his share.  In the contract of mortgage, the owner P.D. Dunn had agreed, at his own
expense, to insure the mortgaged property for its full value and to indorse
 Plaintiff's rights to the insurance money have not been forfeited by failure the policies in such manner as to authorize the Brewery Company to
to pay the insurance provided for in the warehouse receipts. receive the proceeds in case of loss and to retain such part thereof as
might be necessary to satisfy the remainder then due upon the mortgage
 A preponderance of the proof does not demonstrate that the plaintiff ever debt. Instead, however, of effecting the insurance himself Dunn
ordered the cancellation of his insurance with the defendant. Nor is it authorized and requested the Brewery Company to procure insurance on
shown that the plaintiff ever refused to pay the insurance when the bills the property in the amount of P15,000 at Dunn's expense.  
were presented to him, and that notice of an intention to cancel the  San Miguel insured the property only as mortgagee.
insurance was ever given the plaintiff.  Dunn sold the property to Henry Harding.  The insurance was not
assigned by Dunn to Harding.
 The remaining contention of the defendant that the plaintiff cannot claim  When it was destroyed by fire, the two companies settled with San Miguel
the benefits of the agency without sharing in the expenses, is well taken. to the extent of the mortgage credit.  
 RTC: Absolved the 2 companies from the difference.  Henry Harding is
 Although the plaintiff did not expressly authorize the agreement to submit not entitled to the difference between the mortgage credit and the face
the matter to arbitration, yet on his own theory of the case, Mrs. Del value of the policies.
Rosario was acting as his agent in securing insurance, while he benefits  Henry Harding appealed.
from the amicable adjustment of the insurance claims.
ISSUE: 
 The amounts can be figured in several different ways, plaintiff is entitled 1. W/N San Miguel has insurable interest as mortgagor only to the
to P88,595.43 minus P7,185.88, his share of the expenses, minus extent of the mortgage credit - YES
P315.90, due for insurance and storage, or approximately a net amount of 2. W/N Harding has insurable interest as owner - NO
P81,093.65, with legal interest.

 Wherefore, judgment is modified and the plaintiff shall have and recover HELD: affirmed
from the defendants the sum of P81,093.65, with interest at 6 per cent  section 19 of the Insurance Act:
per annum from May 13, 1921, until paid. Without special finding as to  a change of interest in any part of a thing insured unaccompanied
costs in either instance, it is so ordered. by a corresponding change of interest in the insurance, suspends
the insurance to an equivalent extent, until the interest in the
San Miguel v Law Union Rock lnsurance, GR L-14300, 19 Jan.1920 thing and the interest in the insurance are vested in the same
person
3 – EXECUTIVE | Ab Initio Group

45
INSURANCE MIDTERMS (Weeks 1 to 8)
 section 55: RTC: United to pay CKS the amount of P335, 063.11 and Spouses Cha P50,000 as
 the mere transfer of a thing insured does not transfer the policy, exemplary damages, P20,000 as attorney’s fees and costs of suit.
but suspends it until the same person becomes the owner of both CA; deleted exemplary damages and attorney’s fees.
the policy and the thing insured
 Undoubtedly these policies of insurance might have been so framed as to Issue: W/N the CKS has insurable interest because the spouses Cha
have been "payable to the San Miguel Brewery, mortgagee, as its interest violated the stipulation.
may appear, remainder to whomsoever, during the continuance of the
risk, may become the owner of the interest insured." (Sec 54, Act No. Held: No, CA set aside. Awarding the proceeds to spouses Cha.
2427.) Such a clause would have proved an intention to insure the entire Sec. 18. No contract or policy of insurance on property shall be enforceable except
interest in the property, not merely the insurable interest of the San for the benefit of some person having an insurable interest in the property insured.
Miguel Brewery, and would have shown exactly to whom the money, in A non-life insurance policy taken by the petitioner-spouses over their merchandise
case of loss, should be paid. But the policies are not so written. is primarily a contract of indemnity. Insurable interest in the property insured must
 The blame for the situation thus created rests, however, with the Brewery exist at the time the loss occurs. The basis of such requirements of insurable
rather than with the insurance companies, and there is nothing in the interest in property insured is based on sound public policy: to prevent a person
record to indicate that the insurance companies were requested to write from taking out an insurance policy on property upon which he has no insurable
insurance upon the insurable interest of the owner or intended to make interest and collecting the proceeds of said policy in case of loss of the property.
themselves liable to that extent In such a case, the contract of insurance is a mere wager which is void under
 If by inadvertence, accident, or mistake the terms of the contract were not Section 25 of the Insurance Code.
fully set forth in the policy, the parties are entitled to have it reformed. Section 25. Every stipulation in a policy of Insurance for the payment of loss,
But to justify the reformation of a contract, the proof must be of the most whether the person insured has or has not any interest in the property insured, or
satisfactory character, and it must clearly appear that the contract failed to the policy shall be received as proof of such interest, and every policy executed by
express the real agreement between the parties way of gaming or wagering is void.
In the case now before us the proof is entirely insufficient to authorize Section 17. The measure of an insurable interest in the property is the extent to
reformation.  which the insured might be damnified by loss of injury thereof.
The automatic assignment of the policy to CKS under the provision of the lease
Spouses Cha v Court of Appeals, GR 124520, 18 August 1997 contract previously quoted is void for being contrary to law/ or public policy. The
proceeds of the fire insurance policy thus rightfully belong to the spouses. The
Facts: liability of the Cha spouses to CKS for violating their lease contract in that Cha
Spouses Nilo Cha and Stella Uy-Cha and CKS Development Corporation entered a spouses obtained a fire insurance policy over their own merchandise, without the
1 year lease contract with a stipulation not to insure against fire the chattels, consent of CKS, is separate and distinct issue which we do not resolve in this case.
merchandise, textiles, goods and effects placed at any stall or store or space in the
leased premises without first obtaining the written consent and approval of the When insurable interest must exist
lessor. But it insured against loss by fire their merchandise inside the leased
premises for P500, 000 with the United Insurance Co., Inc. without the written Sec. 19. An interest in property insured must exist when the insurance takes
consent of CKS. effect, and when the loss occurs, but not exist in the meantime; and interest in the
On the day the lease contract was to expire, fire broke out inside the leased life or health of a person insured must exist when the insurance takes effect, but
premises and CKS learning that the spouses procured an insurance wrote to need not exist thereafter or when the loss occurs.
United to have the proceeds be paid recently to them. But United refused so CKS
filed against Spouses Cha and United. Sec. 20. Except in the cases specified in the next four sections, and in the cases
of life, accident, and health insurance, a change of interest in any part of a thing
3 – EXECUTIVE | Ab Initio Group

46
INSURANCE MIDTERMS (Weeks 1 to 8)
insured unaccompanied by a corresponding change in interest in the insurance,
suspends the insurance to an equivalent extent, until the interest in the thing and Respondents, Zenith Insurance, Phil. British Assurance and SSS Accredited Group
the interest in the insurance are vested in the same person. of Insurers, paid their corresponding shares of the loss. Demand was made from
respondent Travellers Multi-Indemnity for its share in the loss but the same was
Sec. 21. A change in interest in a thing insured, after the occurrence of an injury refused. Hence, complainants demanded from the other threethe same was also
which results in a loss, does not affect the right of the insured to indemnity for the refused. Petitioner then filed a complaint in intervention claiming the proceeds of
loss. the fire insurance policy issued by respondent Travellers Multi-Indemnity but the
latter alleged that it is not entitled to indemnity for lack of insurable interest before
Sec. 22. A change of interest in one or more several distinct things, separately the loss of the insured premises.
insured by one policy, does not avoid the insurance as to the others.
Issue: Whether or not petitioner has an insurable interest.
Sec. 23. A change on interest, by will or succession, on the death of the insured,
does not avoid an insurance; and his interest in the insurance passes to the person Ruling: Respondent insurance company did not assail the validity of the insurance
taking his interest in the thing insured. policy taken out by petitioner over the mortgaged property. Neither did it deny
that the said property was totally razed by fire within the period covered by the
Sec. 24. A transfer of interest by one of several partners, joint owners, or owners insurance. Respondent advanced an affirmative defense of lack of insurable
in common, who are jointly insured, to the others, does not avoid an insurance interest on the part of the petitioner that before the occurrence of the peril insured
even though it has been agreed that the insurance shall cease upon an alienation against the Palomos had already paid their credit due the petitioner. Respondent
of the thing insured. having admitted the material allegations in the complaint has the burden of proof
to show that petitioner has no insurable interest over the insured property at the
Sec. 25. Every stipulation in a policy of insurance for the payment of loss whether time the contingency took place. Upon that point, there is a failure of proof.
the person insured has or has not any interest in the property insured, or that the Respondent exerted no effort to present any evidence to substantiate its claim,
policy shall be received as proof of such interest, and every policy executed by while petitioner did. For said respondent's failure, the decision must be adverse to
way of gaming or wagering, is void. it.

Tai Tong Chua Che v lnsurance Com, GR L-55397 ,29 February 1988 However, respondent Insurance Commission absolved respondent insurance
company from liability on the basis of the certification issued by the then Court of
Facts: First Instance of Davao, Branch II, that in a certain civil action against the
Complainants acquired a parcel of land and a building they assumed the mortgage Palomos, Arsenio Lopez Chua stands as the complainant and not Tai Tong
of the latter in favor of SSS, which was insured with respondent SSS Accredited Chuache. From said evidence respondent commission inferred that the credit
Group of Insurers. On April 19, 1975, Azucena Palomo obtained a loan from extended by herein petitioner to the Palomos secured by the insured property
petitioner Tai Tong Chuache Inc. securing it with a mortgage was executed over must have been paid. Such is a glaring error which cannot be sanctioned.
the land and the building in favor of petitioner. On April 25, 1975, Arsenio Chua, Respondent Commission's findings are based upon a mere inference.
petitioner’s representative insured the latter's interest with Travellers Multi-
Indemnity Corporation. On June 11, 1975, Pedro Palomo secured fire insurance The record of the case shows that the petitioner to support its claim for the
covering the building with respondent Zenith Insurance Corporation and on July insurance proceeds offered as evidence the contract of mortgage which has not
16, 1975, another fire insurance was procured from respondent Philippine British been cancelled nor released. It has been held in a long line of cases that when the
Assurance Company, covering the same building and the contents thereof. creditor is in possession of the document of credit, he need not prove non-
However, on July 31, 1975, the building and the contents were totally razed by payment for it is presumed. The validity of the insurance policy taken by petitioner
fire. was not assailed by private respondent. Moreover, petitioner's claim that the loan
3 – EXECUTIVE | Ab Initio Group

47
INSURANCE MIDTERMS (Weeks 1 to 8)
extended to the Palomos has not yet been paid was corroborated by Azucena the said Macke for and on behalf of the insured. Such execution of a chattel
Palomo who testified that they are still indebted to herein petitioner. Public mortgage on the insured property without consent to the insurer violated what is
respondent argues however, that if the civil case really stemmed from the loan known as the "alienation clause,".
granted to Azucena Palomo by petitioner the same should have been brought by ISSUE:
Tai Tong Chuache or by its representative in its own behalf. From the above I. Whether the use of the building as a paint and
premise respondent concluded that the obligation secured by the insured property varnish shop annulled the policy insurance.
must have been paid. The premise is correct but the conclusion is wrong. Citing II. II. Whether the execution of the chattel
Rule 3, Sec. 2 respondent pointed out that the action must be brought in the mortgages without the knowledge and consent
name of the real party in interest. Correct! However, it should be borne in mind of the insurance company annulled the policy
that petitioner being a partnership may sue and be sued in its name or by its duly insurance.
authorized representative. The fact that Arsenio Lopez Chua is the representative HELD:
of petitioner is not questioned. Petitioner's declaration that Arsenio Lopez Chua The court ruled in negative for both issues.
acts as the managing partner of the partnership was corroborated by respondent I.
insurance company. Thus Chua as the managing partner of the partnership may The property insured consisted mainly of household furniture kept for the purpose
execute all acts of administration including the right to sue debtors of the of sale. The preservation of the furniture in a salable condition by retouching or
partnership in case of their failure to pay their obligations when it became due and otherwise was incidental to the business.
demandable. Or at the very least, Chua being a partner of petitioner Tai Tong The evidence offered by the plaintiff is to the effect that alcohol was used in
Chuache & Company is an agent of the partnership. Being an agent, it is preparing varnish for the purpose of retouching, though he also says that the
understood that he acted for and in behalf of the firm. Public respondent's alcohol was kept in store and not in the bodega where the furniture was.
allegation that the civil case filed by Arsenio Chua was in his capacity as personal It is well settled that the keeping of inflammable oils on the premises, though
creditor of spouses Palomo has no basis. prohibited by the policy, does not void it if such keeping is incidental to the
business.
The respondent insurance company having issued a policy in favor of herein Thus, where a furniture factory keeps benzine for the purposes of operation (Davis
petitioner which policy was of legal force and effect at the time of the fire, it is vs. Pioneer Furniture Company, 78 N. W. Rep., 596; Faust vs. American Fire
bound by its terms and conditions. Upon its failure to prove the allegation of lack Insurance Company, 91 Wis., 158), or where it is used for the cleaning machinery
of insurable interest on the part of the petitioner, respondent insurance company (Mears vs. Humboldt Insurance Company, 92 Pa. St., 15; 37 Am. Rep., 647), the
is and must be held liable. insurer can not on that ground avoid payment of loss, though the keeping of the
benzine on the premises is expressly prohibitedIt may be added that there was no
Bachrach v British American lnsurance, L-5715,2A December 1910 provision in the policy prohibiting the keeping of paints and varnishes upon the
premises where the insured property was stored. If the company intended to rely
FACTS: upon a condition of that character, it ought to have been plainly expressed in the
Bachrach insured his building against fire with the British-American Assurance policy.
Company. After the effectivity of the policy, the insured stored gasoline, paints and II.
varnishes within the premises insured. The building was burned and the insurer Upon reading the policy of insurance issued by the defendant to the plaintiff, it will
refused to pay the loss on the ground that the risk of fire was increased by the be noted that there is no provision in said policy prohibiting the plaintiff from
storage of gasoline, paints and varnishes. The insurer also claimed that the placing a mortgage upon the property insured, but, admitting that such a provision
plaintiff transferred his interest in and to the property covered by the policy to H. was intended, we think the lower court has completely answered this contention of
W. Peabody & Co. to secure certain indebtedness due and owing to said company, the defendant. He said, in passing upon this question as it was presented: It is
and also that the plaintiff had transferred his interest in certain of the goods claimed that the execution of a chattel mortgage on the insured property violated
covered by the said policy to one Macke, to secure certain obligations assumed by what is known as the "alienation clause," which is now found in most policies, and
3 – EXECUTIVE | Ab Initio Group

48
INSURANCE MIDTERMS (Weeks 1 to 8)
which is expressed in the policies involved in cases 6496 and 6497 by a purchase HELD: affirmed section 19 of the Insurance Act: a change of interest in any
imposing forfeiture if the interest in the property pass from the insured. (Cases part of a thing insured unaccompanied by a corresponding change of interest in
6496 and 6497, in which are involved other action against other insurance the insurance, suspends the insurance to an equivalent extent, until the interest in
companies for the same loss as in the present action.) the thing and the interest in the insurance are vested in the same person
This clause has been the subject of a vast number of judicial decisions (13 Am. &
Eng. Encyc. of Law, 2d ed., pp. 239 et seq.), and it is held by the great weight of section 55: the mere transfer of a thing insured does not transfer the policy, but
authority that the interest in property insured does not pass by the mere execution suspends it until the same person becomes the owner of both the policy and the
of a chattel mortgage and that while a chattel mortgage is a conditional sale, there thing insured
is no alienation within the meaning of the insurance law until the mortgage
acquires a right to take possession by default under the terms of the mortgage. No Undoubtedly these policies of insurance might have been so framed as to have
such right is claimed to have accrued in the case at bar, and the alienation clause been "payable to the San Miguel Brewery, mortgagee, as its interest may appear,
is therefore inapplicable. remainder to whomsoever, during the continuance of the risk, may become the
owner of the interest insured." (Sec 54, Act No. 2427.) Such a clause would have
proved an intention to insure the entire interest in the property, not merely the
insurable interest of the San Miguel Brewery, and would have shown exactly to
whom the money, in case of loss, should be paid. But the policies are not so
San Miguel v Law Union Rock Insurance, GR L-14300, 19 January 1920 written.
The blame for the situation thus created rests, however, with the Brewery rather
FACTS: than with the insurance companies, and there is nothing in the record to indicate
In the contract of mortgage, the owner P.D. Dunn had agreed, at his own that the insurance companies were requested to write insurance upon the
expense, to insure the mortgaged property for its full value and to indorse the insurable interest of the owner or intended to make themselves liable to that
policies in such manner as to authorize the Brewery Company to receive the extent
proceeds in case of loss and to retain such part thereof as might be necessary to
satisfy the remainder then due upon the mortgage debt. Instead,however, of If by inadvertence, accident, or mistake the terms of the contract were not fully
effecting the insurance himself Dunn authorized and requested the Brewery set forth in the policy, the parties are entitled to have it reformed. But to justify
Company to procure insurance on the property in the amount of P15,000 at the reformation of a contract, the proof must be of the most satisfactory
Dunn's expense. character, and it must clearly appear that the contract failed to express the real
San Miguel insured the property only as mortgagee. Dunn sold the propert to agreement between the parties
Henry Harding. The insurance was not assigned by Dunn to Harding.
When it was destroyed by fire, the two companies settled with San Miguelto the In the case now before us the proof is entirely insufficient to authorize
extent of the mortgage credit. reformation.
RTC: Absolved the 2 companies from the difference. Henry Harding is
not entitled to the difference between the mortgage credit and the face value of
the policies.
WEEK 7
Henry Harding appealed.
ISSUE: CONCEALMENT
1. W/N San Miguel has insurable interest as mortgagor only to the
extent of the mortgage credit - YES 2. W/N Harding has insurable Sec. 26. A neglect to communicate that which a party knows and ought to
interest as owner - NO communicate, is called a concealment.

3 – EXECUTIVE | Ab Initio Group

49
INSURANCE MIDTERMS (Weeks 1 to 8)
Sec. 27. A concealment whether intentional or unintentional entitles the injured
party to rescind a contract of insurance. (As amended by Batasang Pambansa Blg.  Facts:
874) A joint life insurance policy was issued to Bernardo Argente and his wife Vicenta
Sec. 28. Each party to a contract of insurance must communicated to the other, upon payment of premium, by West Coast. On Nov. 18, 1925, during the
in good faith, all facts within his knowledge which are material to the contract and effectivity of the policy, Vicenta died of cerebral apoplexy. The representations
as to which he makes no warranty, and which the other has not the means of made by Bernardo Argente and his wife in their applications to the defendant for
ascertaining. life insurance were false with respect to their estate of health during the period of
Sec. 29. An intentional and fraudulent omission, on the part of one insured, to five years preceding the date of such applications, and that they knew the
communicate information of matters proving or tending to prove the falsity of a representations made by them in their applications were false. Also, it was further
warranty, entitles the insurer to rescind. found from the evidence that the answers given by Bernardo Argente and his wife
Sec. 30. Neither party to a contract of insurance is bound to communicate at the time of the medical examination by Doctor Sta. Ana were false with respect
information of the matters following, except in answer to the inquiries of the to the condition of their health at that time and for a period of several years prior
other: thereto. Thereafter, Bernardo claimed payment but was refused.
(a) Those which the other knows;
(b) Those which, in the exercise of ordinary care, the other ought to know, and of Issue: Whether or not on the basis of the misrepresentations of Vicenta,
which the former has no reason to suppose him ignorant; Bernardo is barred from recovery.
(c) Those of which the other waives communication;
(d) Those which prove or tend to prove the existence of a risk excluded by a
warranty, and which are not otherwise material; and
(e) Those which relate to a risk excepted from the policy and which are not Ruling
otherwise material. YES. One ground for the rescission of a contract of insurance under the
Insurance Act is "a concealment," which in section 25 is defined as "A neglect to
Sec. 31. Materiality is to be determined not by the event, but solely by the communicate that which a party knows and ought to communicate." The basis of
probable and reasonable influence of the facts upon the party to whom the the rule vitiating the contract in case of concealment is that it misleads or deceives
communication is due, in forming his estimate of the disadvantages of the the insurer into accepting the risk, or accepting it at the rate of premium agreed
proposed contract, or in making his inquiries. upon. The insurer, relying upon the belief that the assured will disclose every
Sec. 32. Each party to a contract of insurance is bound to know all the general material within his actual or presumed knowledge, is misled into a belief that the
causes which are open to his inquiry, equally with that of the other, and which circumstance withheld does not exist, and he is thereby induced to estimate the
may affect the political or material perils contemplated; and all general usages of risk upon a false basis that it does not exist.
trade. The court found that the representations made by Vicenta in his application for life
Sec. 33. The right to information of material facts may be waived, either by the insurance were false with respect to her state of health and that she knew and
terms of the insurance or by neglect to make inquiry as to such facts, where they was aware that the representations so made by her were false.  In an action on a
are distinctly implied in other facts of which information is communicated. life insurance policy where the evidence conclusively shows that the answers to
Sec. 34. Information of the nature or amount of the interest of one insured need questions concerning diseases were untrue, the truth or falsity of the answer
not be communicated unless in answer to an inquiry, except as prescribed by becomes the determining factor.
section fifty-one. If the policy was procured by fraudulent misrepresentations, the contract of
Sec. 35. Neither party to a contract of insurance is bound to communicate, even insurance apparently set forth therein was never legally existent.  It can be fairly
upon inquiry, information of his own judgment upon the matters in question. assumed that had the true facts been disclosed by the insured, the insurance
would never have been granted.
Argente v West Coast Life, GR L-24899, 9 March 1928
3 – EXECUTIVE | Ab Initio Group

50
INSURANCE MIDTERMS (Weeks 1 to 8)
Great Pacific Life v CA, GR L-31845, 30 April 1979
1. No.
Facts:
The receipt was intended to be merely a provisional insurance contract. Its
Ngo Hing filed an application with the Great Pacific for a twenty-year endowment perfection was subject to compliance of the following conditions: (1) that the
policy in the amount of P50,000.00 on the life of his one-year old daughter Helen. company shall be satisfied that the applicant was insurable on standard rates; (2)
He supplied the essential data which petitioner Mondragon, the Branch Manager, that if the company does not accept the application and offers to issue a policy for
wrote on the form. The latter paid the annual premium the sum of P1,077.75 a different plan, the insurance contract shall not be binding until the applicant
going over to the Company, but he retained the amount of P1,317.00 as his accepts the policy offered; otherwise, the deposit shall be refunded; and (3) that if
commission for being a duly authorized agent of Pacific Life. the company disapproves the application, the insurance applied for shall not be in
force at any time, and the premium paid shall be returned to the applicant.
Upon the payment of the insurance premium, the binding deposit receipt was
issued Ngo Hing. Likewise, petitioner Mondragon handwrote at the bottom of the The receipt is merely an acknowledgment that the latter's branch office had
back page of the application form his strong recommendation for the approval of received from the applicant the insurance premium and had accepted the
the insurance application. Then Mondragon received a letter from Pacific Life application subject for processing by the insurance company. There was still
disapproving the insurance application. The letter stated that the said life approval or rejection the same on the basis of whether or not the applicant is
insurance application for 20-year endowment plan is not available for minors "insurable on standard rates." Since Pacific Life disapproved the insurance
below seven years old, but Pacific Life can consider the same under the Juvenile application of respondent Ngo Hing, the binding deposit receipt in question had
Triple Action Plan, and advised that if the offer is acceptable, the Juvenile Non- never become in force at any time. The binding deposit receipt is conditional and
Medical Declaration be sent to the company. does not insure outright. This was held in Lim v Sun. The deposit paid by private
respondent shall have to be refunded by Pacific Life.
The non-acceptance of the insurance plan by Pacific Life was allegedly not
communicated by petitioner Mondragon to private respondent Ngo Hing. Instead,
on May 6, 1957, Mondragon wrote back Pacific Life again strongly recommending 2. Yes.  
the approval of the 20-year endowment insurance plan to children, pointing out
that since the customers were asking for such coverage. Ngo Hing had deliberately concealed the state of health of his daughter Helen Go.
Helen Go died of influenza. Ngo Hing sought the payment of the proceeds of the When he supplied data, he was fully aware that his one-year old daughter is
insurance, but having failed in his effort, he filed the action for the recovery before typically a mongoloid child. He withheld the fact material to the risk insured.
the Court of First Instance of Cebu, which ruled against him.
“The contract of insurance is one of perfect good faith uberrima fides meaning
good faith, absolute and perfect candor or openness and honesty; the absence of
Issues: any concealment or demotion, however slight.” The concealment entitles the
insurer to rescind the contract of insurance.
1. Whether the binding deposit receipt constituted a temporary contract
of the life insurance in question
2. Whether Ngo Hing concealed the state of health and physical Saturnino v Philippine American Life, GR L-16163, 28 February 1963
condition of Helen Go, which rendered void the policy
Facts:

Ruling:
3 – EXECUTIVE | Ab Initio Group

51
INSURANCE MIDTERMS (Weeks 1 to 8)
>  2 months prior to the insurance of the policy, Saturnino was operated on for Appellants also contend that there was no fraudulent concealment of the truth
cancer, involving complete removal of the right breast, including the pectoral inasmuch as the insured herself did not know, since her doctor never told her, that
muscles and the glands, found in the right armpit. the disease for which she had been operated on was cancer.  In the first place,
>  Notwithstanding the fact of her operation, Saturnino did not make a disclosure concealment of the fact of the operation itself was fraudulent, as there could not
thereof in her application for insurance. have been any mistake about it, no matter what the ailment.
>  She stated therein that she did not have, nor had she ever had, among others
listed in the application, cancer or other tumors; that she had not consulted any Secondly, in order to avoid a policy, it is not necessary to show actual fraud on the
physician, undergone any operation or suffered any injury within the preceding 5 part of the insured. In this jurisdiction, concealment, whether intentional or
years. unintentional entitled the insurer to rescind the contract of insurance, concealment
>  She also stated that she had never been treated for, nor did she ever have any being defined as “negligence to communicate that which a party knows and ought
illness or disease peculiar to her sex, particularly of the breast, ovaries, uterus and to communicate.”  The basis of the rule vitiating the contract in cases of
menstrual disorders. concealment is that it misleads or deceives the insurer into accepting the risk, or
>  The application also recited that the declarations of Saturnino constituted a accepting it at a rate of premium agreed upon.  The insurer, relying upon the
further basis for the issuance of the policy. belief that the insured will disclose every material fact within his actual or
presumed knowledge, is misled into a belief that the circumstances withheld does
Issue: not exist, and he is thereby induced to estimate the risk upon a false basis that it
Whether or not the insured made such false representation of material does not exist.
facts as to avoid the policy.
Canilang v CA and Great Pacific Life, GR 92492,17 June 1993
Held:
YES. Facts:
There can be no dispute that the information given by her in the application for Canilang was found to have suffered from sinus tachycardia then bronchitis after a
insurance was false, namely, that she never had cancer or tumors or consulted check-up from his doctor. The next day, he applied for a "non-medical" insurance
any physician or undergone any operation within the preceding period of 5 years. policy with respondent Grepalife naming his wife, Thelma Canilang, as
his beneficiary. This was to the value of P19,700.
The question to determine is: Are the facts then falsely represented material?   The He died of "congestive heart failure," "anemia," and "chronic anemia."  The widow
Insurance Law provides that “materiality is to be determined not by the event, but filed a claim with Great Pacific which the insurer denied on the ground that the
solely by the probable and reasonable influence of the facts upon the party to insured had concealed material information from it.
whom the communication is due, in forming his estimate of the proposed contract, Petitioner then filed a complaint against Great Pacific for recovery of the insurance
or making his inquiries. proceeds. Petitioner testified that she was not aware of any serious illness suffered
by her late husband and her husband had died because of a kidney disorder. The
The contention of appellants is that the facts subject of the representation were doctor who gave the check up stated that he treated the deceased for “sinus
not material in view of the non-medical nature of the insurance applied for, which tachycardia” and "acute bronchitis."
does away with the usual requirement of medical examination before the policy is Great Pacific presented a physician who testified that the deceased's
issued.  The contention is without merit.  If anything, the waiver of medical insurance application had been approved on the basis of his medical declaration.
examination renders even more material the information required of the applicant She explained that as a rule, medical examinations are required only in cases
concerning previous condition of health and diseases suffered, for such where the applicant has indicated in his application for insurance coverage that he
information necessarily constitutes an important factor which the insurer takes into has previously undergone medical consultation and hospitalization.
consideration in deciding whether to issue the policy or not. The Insurance Commissioner  ordered Great Pacific to pay P19,700 plus legal
interest and P2,000.00 as attorney's fees. On appeal by Great Pacific, the Court
3 – EXECUTIVE | Ab Initio Group

52
INSURANCE MIDTERMS (Weeks 1 to 8)
of Appeals reversed. It found that the failure of Jaime Canilang to disclose the application for insurance; that "probable and reasonable influence of the facts"
previous medical consultation and treatment constituted material information concealed must, of course, be determined objectively, by the judge ultimately.
which should have been communicated to Great Pacific to enable the latter to The Insurance Commissioner had also ruled that the failure of Great Pacific to
make proper inquiries. convey certain information to the insurer was not "intentional" in nature, for the
Hence this petition by the widow. reason that Canilang believed that he was suffering from minor ailment like a
common cold. Section 27 stated that:
Issue: Won Canilang was guilty of misrepresentation Sec. 27. A concealment whether intentional or unintentional entitles the injured
party to rescind a contract of insurance.
Held: Yes. Petition denied. The failure to communicate must have been intentional rather than inadvertent.
Canilang could not have been unaware that his heart beat would at times rise to
Ratio: high and alarming levels and that he had consulted a doctor twice in the two (2)
There was a right of the insurance company to rescind the contract if it was months before applying for non-medical insurance. Indeed, the last medical
proven that the insured committed fraud in not affirming that he was treated for consultation took place just the day before the insurance application was filed. In
heart condition and other ailments stipulated. all probability, Jaime Canilang went to visit his doctor precisely because of
Apart from certifying that he didn’t suffer from such a condition, Canilang also the ailment.
failed to disclose in the that he had twice consulted a doctor who had found him to Canilang's failure to set out answers to some of the questions in the
be suffering from "sinus tachycardia" and "acute bronchitis." insurance application constituted concealment. 
Under the Insurance Code:
Sec. 26. A neglect to communicate that which a party knows and ought to Sun Life v Bacani, GR 105135, 22 June 1995
communicate, is called a concealment.
Sec. 28. Each party to a contract of insurance must communicate to the other, in FACTS:
good faith, all factors within his knowledge which are material to the contract and  
as to which he makes no warranty, and which the other has not the means of Robert John Bacani procured a life insurance contract for himself from petitioner-
ascertaining. company, designating his mother Bernarda Bacani, herein private respondent, as
The information concealed must be information which the concealing party knew the beneficiary. He was issued a policy valued at P100,000.00 with double
and should have communicated. The test of materiality of such information indemnity in case of accidental death. Sometime after, the insured died in a plane
is contained in Section 31: crash. Bernarda filed a claim with petitioner, seeking the benefits of the insurance
Sec. 31. Materiality is to be determined not by the event, but solely by the policy taken by her son. However, said insurance company rejected the claim on
probable and reasonable influence of the facts upon the party to whom the the ground that the insured did not disclose material facts relevant to the issuance
communication is due, in forming his estimate of the disadvantages of the of the policy, thus rendering the contract of insurance voidable. Petitioner
proposed contract, or in making his inquiries. discovered that two weeks prior to his application for insurance, the insured was
The information which Jaime Canilang failed to disclose was material to the ability examined and confined at the Lung Center of the Philippines, where he was
of Great Pacific to estimate the probable risk he presented as a subject of life diagnosed for renal failure. The RTC, as affirmed by the CA, this fact was
insurance. Had he disclosed his visits to his doctor, the diagnosis made and concealed, as alleged by the petitioner. But the fact that was concealed was not
medicines prescribed by such doctor, in the insurance application, it may be the cause of death of the insured and that matters relating to the medical history
reasonably assumed that Great Pacific would have made further inquiries and of the insured is deemed to be irrelevant since petitioner waived the medical
would have probably refused to issue a non-medical insurance policy. examination prior to the approval and issuance of the insurance policy.
Materiality relates rather to the "probable and reasonable influence of the facts"  
upon the party to whom the communication should have been made, in assessing
the risk involved in making or omitting to make further inquiries and in accepting
3 – EXECUTIVE | Ab Initio Group

53
INSURANCE MIDTERMS (Weeks 1 to 8)
ISSUE: Whether or not the concealment of such material fact, despite it The company set up the defense that the insured was guilty of misrepresentation
not being the cause of death of the insured, is sufficient to render the and concealment of material facts. They subsequently refused to give the
insurance contract voidable indemnity.
  The trial court rendered judgment ordering defendant to pay plaintiff the sum of
HELD: P10,000.00, plus P2,000.00 as attorney's fees. The Court of Appeals reversed the
  decision of the trial court, holding that the insured was guilty of concealment of
YES. Section 26 of the Insurance Code is explicit in requiring a party to a contract material facts. Hence the present petition.
of insurance to communicate to the other, in good faith, all facts within his
knowledge which are material to the contract and as to which he makes no Issue: Whether or not the insured is guilty of concealment of some facts
warranty, and which the other has no means of ascertaining. Anent the finding material to the risk insured that consequently avoids the policy.
that the facts concealed had no bearing to the cause of death of the insured, it is
well settled that the insured need not die of the disease he had failed to disclose Held: Yes. Petition dismissed.
to the insurer. It is sufficient that his non-disclosure misled the insurer in forming
his estimates of the risks of the proposed insurance policy or in making inquiries. Ratio:
The SC, therefore, ruled that petitioner properly exercised its right to rescind the The first confinement took place from January 29, 1950 to February 11, while
contract of insurance by reason of the concealment employed by the insured. It his application was submitted on September 5, 1950. When he gave
must be emphasized that rescission was exercised within the two-year his answers to the policy, he concealed the ailment of which he was treated in the
contestability period as recognized in Section 48 of The Insurance Code. hospital.
WHEREFORE, the petition is GRANTED and the Decision of the Court of Appeals is The negative answers given by the insured regarding his
REVERSED and SET ASIDE. previous ailment deprived defendant of the opportunity to make the necessary
inquiry as to the nature of his past illness so that as it may form its estimate
Yu Pang Cheng v CA, GR L-12465, 29 May 1959 relative to the approval of his application. Had defendant been given such
opportunity, the company would probably had never consented to the issuance of
Facts: the policy in question. In fact, according to the death certificate, the insured’s
Yu Pang Eng submitted application for insurance consisting of the medical death may have direct connection with his previous illness.
declaration made by him to the medical examiner and the report. Yu then paid the Under the law, a neglect to communicate that which a party knows and ought to
premium in the sum of P591.70. communicate, is called concealment. This entitles the insurer to rescind the
The insured, in his application for insurance, said “no” to ever having stomach contract. The insured is required to communicate to the insurer all facts within his
disease, cancer, and fainting-spells. He also claimed to not have consulted a knowledge which are material to the contract and which the other party has not
physician regarding such diseases. the means of ascertaining. The materiality is to be determined not by the event
After submitting the form, he entered the hospital where he complained of but solely by the probable and reasonable influence of the facts upon the party to
dizziness, anemia, abdominal pains and tarry stools. He was found to have peptic whom the communication is due.
ulcer. Argente vs. West Coast- “One ground for the rescission of a contract of insurance
The insured entered another hospital for medical treatment but he died of under the insurance Act is "a concealment", which in section 25 is defined "A
"infiltrating medullary carcinoma, Grade 4, advanced cardiac and of neglect to communicate that which a party knows and ought to communicate."
lesser curvature, stomach metastases spleen." “In an action on a life insurance policy where the evidence conclusively shows that
Yu Pang Cheng aimed to collect P10,000.00 on  life of one Yu Pang Eng from an the answers to questions concerning diseases were untrue, the truth or falsity of
insurance company. the answers become the determining factor. If the policy was procured by
fraudulent representations, the contract of insurance was never legally existent. It

3 – EXECUTIVE | Ab Initio Group

54
INSURANCE MIDTERMS (Weeks 1 to 8)
can fairly be assumed that had the true facts been disclosed by the assured, the Section 27 of the Insurance Law:
insurance would never have been granted.” Sec. 27. Such party a contract of insurance must communicate to the other, in
good faith, all facts within his knowledge which are material to the contract, and
Ng Zee v Asian Crusader Life lnsurance, GR L-30685, 30 May 1983 which the other has not the means of ascertaining, and as to which he makes no
warranty.
Facts:  "Concealment exists where the assured had knowledge of a fact material to the
Kwong Nam applied for a 20-year endowment insurance on his life for the sum of risk, and honesty, good faith, and fair dealing requires that he should
P20,000.00, with his wife, appellee Ng Gan Zee as beneficiary. On the same date, communicate it to the assurer, but he designedly and intentionally withholds the
Asian Crusader, upon receipt of the required premium from the insured, approved same."
the application and issued the corresponding policy. Kwong Nam died of cancer of It has also been held "that the concealment must, in the absence of inquiries, be
the liver with metastasis. All premiums had been paid at the time of his death. not only material, but fraudulent, or the fact must have been intentionally
Ng Gan Zee presented a claim for payment of the face value of the policy. On the withheld."
same date, she submitted the required proof of death of the Fraudulent intent on the part of the insured must be established to entitle the
insured. Appellant denied the claim on the ground that the answers given by the insurer to rescind the contract. And as correctly observed by the lower court,
insured to the questions in his application for life insurance were untrue. "misrepresentation as a defense of the insurer to avoid liability is an 'affirmative'
Appellee brought the matter to the attention of the Insurance Commissioner. The defense. The duty to establish such a defense by satisfactory and convincing
latter, after conducting an investigation, wrote the appellant that he had found no evidence rests upon the defendant. The evidence before the Court does not clearly
material concealment on the part of the insured and that, therefore, appellee and satisfactorily establish that defense."
should be paid the full face value of the policy. The company refused to settle its It bears emphasis that Kwong Nam had informed the appellant's medical examiner
obligation. of the tumor. His statement that said tumor was "associated with ulcer of the
Appellant alleged that the insured was guilty of misrepresentation when he stomach" should be construed as an expression made in good faith of his belief as
answered "No" to the following question appearing in the application for life to the nature of his ailment and operation.
insurance- While the information communicated was imperfect, the same was sufficient to
Has any life insurance company ever refused your application for insurance or for have induced appellant to make further inquiries about the ailment and operation
reinstatement of a lapsed policy or offered you a policy different from that applied of the insured.
for? If, so, name company and date. Section 32 of Insurance Law:
The lower court ruled against the company on lack of evidence. Section 32. The right to information of material facts maybe waived either by the
Appellant further maintains that when the insured was examined in connection terms of insurance or by neglect to make inquiries as to such facts where they are
with his application for life insurance, he gave the appellant's medical examiner distinctly implied in other facts of which information is communicated.
false and misleading information as to his ailment and previous operation. The Where a question appears to be not answered at all or to be imperfectly
company contended that he was operated on for peptic ulcer  2 years before the answered, and the insurers issue a policy without any further inquiry, they waive
policy was applied for and that he never disclosed such an operation. the imperfection of the answer and render the omission to answer more fully
immaterial.
Issue: WON Asian Crusader was deceived into entering the contract or in The company or its medical examiner did not make any further inquiries on such
accepting the risk at the rate of premium agreed upon because of matters from the hospital before acting on the application for insurance. The fact
insured's representation? of the matter is that the defendant was too eager to accept the application and
receive the insured's premium. It would be inequitable now to allow
Held: No. Petition dismissed. the defendant to avoid liability under the circumstances."

Ratio: Colado v lnsular Life, 51 O.G. (No. 12), 1955


3 – EXECUTIVE | Ab Initio Group

55
INSURANCE MIDTERMS (Weeks 1 to 8)

Facts: The acceptance of Insular life of the overdue premiums did not necessarily deprive
>  Vivencio Collado applied for an insurance contract with Insular life in 1948.  His it of the right to cancel the policy in case of default incurred by the Insured in the
application was approved and he began started making premium payments.  payment of future premiums.  The case would be different had the insured died at
However, he defaulted and the insurance was cancelled. any time after the payment of overdue premiums but previous to the
>  He then applied for the reinstatement of his insurance policy in Nov. of 1951 reinstatement of the policy, for the, Insular, by its acceptance of its overdue
and tendered the amount of premium for the years 1950-1951. premiums is deemed to have waived its right to rescind the policy.
>  He stated that he was as of Nov. 1951 of good health, and that he had no
injuries, ailments or illnesses and had not been sick for any case since 1948 (his The evidence at hand shows that insofar as the payment of the last quarterly
medical check up when he applied for insurance) and that he had not consulted premium for 1951 was concerned, Insular had availed of the right to rescind the
any physician or practitioner for any case since the date of such latest medical policy by notifying the Insured that the policy had lapsed.
exam.
>  However, when Vivencio applied for the reinstatement, he was already sick of a REPRESENTATION
fatal disease known as carcinoma of the liver and that 4 days prior to his
application for insurance, he consulted a doctor regarding his condition. Sec. 37. A representation may be made at the time of, or before, issuance of the
>  The reinstatement was approved.  Vivencio again failed to pay the premiums policy.
for the last quarter of Nov. 1951 and as such, Insular life sent him a notice
canceling the policy. Sec. 38. The language of a representation is to be interpreted by the same rules
>  Vivencio then died.  The beneificiaries instituted the present action to recover as the language of contracts in general.
from Insular life the death benefits of a life insurance policy valued at 2T.   Insular
refused to pay claiming concealment on the part of Vivencio. Sec. 39. A representation as to the future is to be deemed a promise, unless it
>  Collado contends that Insular life had waived the right to rescine the policy in appears that it was merely a statement of belief or expectation.
view of its repeated acceptance of the overdue premiums for the second and third Sec. 40. A representation cannot qualify an express provision in a contract of
years. insurance, but it may qualify an implied warranty.
>  Municipal court of Manila found for Collado and Insular filed an appeal with CFI
of Manila. CFI rendered judgment in favor of Insular and dismissed Collado’s Sec. 41. A representation may be altered or withdrawn before the insurance is
complaint. effected, but not afterwards.

Issue: Whether or nor Insular life was estopped and could no longer Sec. 42. A representation must be presumed to refer to the date on which the
cancel the contract due to the fact that it accepted the tender of overdue contract goes into effect.
payments from Vivencio.
Sec. 43. When a person insured has no personal knowledge of a fact, he may
Held: nevertheless repeat information which he has upon the subject, and which he
NO. believes to be true, with the explanation that he does so on the information of
It is enormously clear that  when the deceased applied for a reinstatement of his others; or he may submit the information, in its whole extent, to the insurer; and
policy in Nov. 1951, he had already been afflicted with the fatal ailment for a in neither case is he responsible for its truth, unless it proceeds from an agent of
period of about four months.  Furthermore, in submitting together with his the insured, whose duty it is to give the information.
application for reinstatement, a health statement to the effect that he was in good
health, Vivencio concealed the material fact that he had consulted a doctor and Sec. 44. A representation is to be deemed false when the facts fail to correspond
was then found to be afflicted with the malady. with its assertions or stipulations.
3 – EXECUTIVE | Ab Initio Group

56
INSURANCE MIDTERMS (Weeks 1 to 8)
The trial court dismissed the complaint and ordered edillon to pay P1000. The
Sec. 45. If a representation is false in a material point, whether affirmative or reason was that a policy of insurance being a contract of adhesion, it was the duty
promissory, the injured party is entitled to rescind the contract from the time when of the insured to know the terms of the contract he or she is entering into.
the representation becomes false. The right to rescind granted by this Code to the The insured could not have been qualified under the conditions stated in said
insurer is waived by the acceptance of premium payments despite knowledge of contract and should have asked for a refund of the premium.
the ground for rescission. (As amended by Batasang Pambansa Blg. 874).
Issue: Whether or not the acceptance by the insurance corporation of
Sec. 46. The materiality of a representation is determined by the same rules as the premium and the issuance of the corresponding certificate of
the materiality of a concealment. insurance should be deemed a waiver of the exclusionary condition of
coverage stated in the policy.
Sec. 47. The provisions of this chapter apply as well to a modification of a
contract of insurance as to its original formation. Held: Yes. Petition granted.

Sec. 48. Whenever a right to rescind a contract of insurance is given to the Ratio:
insurer by any provision of this chapter, such right must be exercised previous to The age of Lapuz was not concealed to the insurance company.
the commencement of an action on the contract. Her application clearly indicated her age of the time of filing the same to be almost
65 years of age. Despite such information which could hardly be overlooked, the
After a policy of life insurance made payable on the death of the insured shall have insurance corporation received her payment of premium and issued the
been in force during the lifetime of the insured for a period of two years from the corresponding certificate of insurance without question.
date of its issue or of its last reinstatement, the insurer cannot prove that the There was sufficient time for the private respondent to process the application and
policy is void ab initio or is rescindible by reason of the fraudulent concealment or to notice that the applicant was over 60 years of age and cancel the policy.
misrepresentation of the insured or his agent. Under the circumstances, the insurance corporation is already deemed in estoppel.
It inaction to revoke the policy despite a departure from the exclusionary
Edillon v Manila Bankers Life, GR L-34200,30 September 1982 condition contained in the said policy constituted a waiver of such condition,
similar to Que Chee Gan vs. Law Union Insurance.
Facts: The insurance company was aware, even before the policies were issued, that in
Carmen O, Lapuz applied with Manila Bankers for insurance coverage against the premises insured there were only two fire hydrants contrary to
accident and injuries. She gave the date of her birth as July 11, 1904. She paid the the requirements of the warranty in question.
sum of P20.00 representing the premium for which she was issued the It is usually held that where the insurer, at the time of the issuance of a policy of
corresponding receipt. The policy was to be effective for 90 days. insurance, has knowledge of existing facts which, if insisted on, would invalidate
During the effectivity, Carmen O. Lapuz died in a vehicular accident in the North the contract from its very inception, such knowledge constitutes a waiver of
Diversion Road. conditions in the contract inconsistent with the known facts, and the insurer is
Petitioner Regina L. Edillon, a sister of the insured and the beneficiary in the stopped thereafter from asserting the breach of such conditions.
policy, filed her claim for the proceeds of the insurance. Her claim having been To allow a company to accept one's money for a policy of insurance which it then
denied, Regina L. Edillon instituted this action in the trial court. knows to be void and of no effect, though it knows as it must, that the
The insurance corporation relies on a provision contained in the contract excluding assured believes it to be valid and binding, is so contrary to the dictates
its liability to pay claims under the policy in behalf of "persons who are under the of honesty and fair dealing.
age of sixteen (16) years of age or over the age of sixty (60) years" They pointed Capital Insurance & Surety Co., Inc. vs. - involved a violation of the provision of
out that the insured was over sixty (60) years of age when she applied for the the policy requiring the payment of premiums before the insurance shall become
insurance coverage, hence the policy became void. effective. The company issued the policy upon the execution of a promissory note
3 – EXECUTIVE | Ab Initio Group

57
INSURANCE MIDTERMS (Weeks 1 to 8)
for the payment of the premium. A check given subsequent by the insured as The fact that the insured did not read the application which he signed, is not
partial payment of the premium was dishonored for lack of funds. Despite indicative of bad faith. It has been held that it is not negligence for the insured to
such deviation from the terms of the policy, the insurer was held liable. sign an application without first reading it if the insurer by its conduct in
“... is that although one of conditions of an insurance policy is that "it shall not be appointing the agent influenced the insured to place trust and confidence in the
valid or binding until the first premium is paid", if it is silent as to the mode of agent.
payment, promissory notes received by the company must be deemed to have
been accepted in payment of the premium. In other words, a requirement for the Tan Chay Heng v West Coast Life, GR L-27541,21 Nov. 1927
payment of the first or initial premium in advance or actual cash may be waived by
acceptance of a promissory note...” Facts:
Tan Chay applied for a life insurance policy of for the sum of P10,000 where he
lnsular Life v Feliciano, GR L-47593, 29 December 1943 was the sole beneficiary. The company approved this.  The policy was issued upon
the payment by Tan Ceang of the first year's premium worth P936. The company
FACTS: agreed to pay the beneficiay the amount of the policy upon the receipt of the
Evaristo Feliciano filed an application for insurance with the Insular Life upon the proofs of the death of the insured while the policy was in force. Without any
solicitation in one of its agent. He was issued two insurance policies with an premium due or unpaid, Ceang died. Tan Chay plaintiff submitted the proofs of the
aggregate amount of Php25,000. In September 1935, he died. His heirs (Serafin death of Tan Ceang with a claim for the payment. The company refused to pay.
Feliciano et al) filed an insurance claim but Insular Life refused to pay as it averred The company alleged that Tan Ceang obtained the policy by means of deceit to
that Feliciano’s application was fraudulenly obtained. It was later found in court the effect that the medical certificate had false statements about his health. They
that the insurance agent and the medical examiner of Insular Life who assisted also claimed that he didn’t pay the premium. The court ruled for Tan Chay and
Feliciano in signing the application knew that Feliciano was already suffering from commanded the company to pay 10,000 pesos.
tuberculosis; that they were aware of the true medical condition of Feliciano yet
they still made it appear that he was healthy in the insurance application form; Issue: WON Section 47 of the Insurance Code applies to this case.
that Feliciano signed the application in blank and the agent filled the information
for him. Held: No. Petition dismissed.
ISSUE:
Whether or not Insular Life can avoid the insurance policy by reason of Ratio:
the fact that its agent knowingly and intentionally wrote down the The plaintiff contends that section 47 of the Insurance Act should be applied, and
answers in the application differing from those made by Feliciano hence that when so applied, the company is barred and estopped to plead the matters
instead of serving the interests of his principal, acts in his own or alleged in its special defense. That section states:
another’s interest and adversely to that of his principal. Whenever a right to rescind a contract of insurance is given to the insurer by any
HELD: provision of this chapter, such right must be exercised previous to
No. Insular Life must pay the insurance policy. The weight of authority is that if an the commencement of an action on the contract.
agent of the insurer, after obtaining from an applicant for insurance a correct and
truthful answer to interrogatories contained in the application for insurance, The defendant contends that section 47 does not apply to this special defense. If
without knowledge of the applicant fills in false answers, either fraudulently or in legal effect defendant's special defense is in the nature of an act to rescind "a
otherwise, the insurer cannot assert the falsity of such answers as a defense to contract of insurance," then such right must be exercised prior to an
liability on the policy, and this is true generally without regard to the subject action enforce the contract.
matter of the answers or the nature of the agent’s duties or limitations on his
authority, at least if not brought to the attention of the applicant. Defendant denied that if ever issued the policy in question.

3 – EXECUTIVE | Ab Initio Group

58
INSURANCE MIDTERMS (Weeks 1 to 8)
The word "rescind" has a well defined legal meaning, and as applied to contracts, The Court of Appeals dismissed ' the petitioners' appeal from the Insurance
it presupposes the existence of a contract to rescind. The rescission relates only to Commissioner's decision for lack of merit. Hence, this petition.
the unfulfilled part, and not to the entire agreement, making the party rescinding
liable on notes executed pursuant to the contract which matured before the Issue:
rescission. The rescission is the unmaking of a contract, requiring the WON Philam didn’t have the right to rescind the contract of insurance as
same concurrence of wills as that which made it, and nothing short of this will rescission must allegedly be done during the lifetime of
suffice. the insured within two years and prior to the commencement of action.

After a contract has been broken, whether by an inability to perform it, or by Held: No. Petition dismissed.
rescinding against right or otherwise, the party not in fault may sue the other for
the damages suffered, or, if the parties can be placed in status quo, he may, Ratio:
should he prefer, return what he has received and recover in a suit value of what The Insurance Code states in Section 48:
he has paid or done. The latter remedy is termed "rescission." “Whenever a right to rescind a contract of insurance is given to the insurer by any
provision of this chapter, such right must be exercised previous to
In the instant case, the defendant does not seek to have the alleged insurance the commencement of an action on the contract.
contract rescinded. It only denies that it ever made any contract of insurance on After a policy of life insurance made payable on the death of the insured shall
the life of Tan Ceang or that any such a contract ever existed. If have been in force during the lifetime of the insured for a period of two years from
the defendant never made or entered into the contract in question, there is no the date of its issue or of its last reinstatement, the insurer cannot prove that the
contract to rescind, and, hence, section 47 doesn’t apply. As stated, an action to policy is void ab initio or is rescindable by reason of the fraudulent concealment or
rescind a contract is founded upon and presupposes the existence of the contract misrepresentation of the insured or his agent.”
which is sought to be rescinded.
If all of the material matters set forth and alleged in the defendant's special plea The so-called "incontestability clause"  in the second paragraph prevents the
are true, there was no valid contract of insurance, for the simple reason that the insurer from raising the defenses of false representations insofar as health and
minds of the parties never met and never agreed upon the terms and conditions of previous diseases are concerned if the insurance has been in force for at least two
the contract. If such matters are known to exist by a preponderance of the years during the insured's lifetime.
evidence, they would constitute a valid defense to plaintiff's cause of action. Upon The policy was in force for a period of only one year and five months. Considering
the question as to whether or not they or are not true, the court couldn’t say, but that the insured died before the two-year period had lapsed, respondent company
they were sure that section 47 does not apply to the allegations made in the is not, therefore, barred from proving that the policy is void ab initio by reason of
answer. the insured's fraudulent concealment or misrepresentation.

Tan v Court of Appeals, GR 48049,29 June 1989 The "incontestability clause" added by the second paragraph of Section 48 is in
force for two years. After this, the defenses of concealment or misrepresentation
Facts: no longer lie.
Tan Lee Siong, father of the petitioners, applied for life insurance in the amount of
P 80,000.00 with Philamlife. It was approved. Tan Lee Siong died of hepatoma. The petitioners argue that no evidence was presented to show that the medical
Petitioners then filed a claim for the proceeds. The company denied petitioners' terms were explained in a layman's language to the insured. They also argue that
claim and rescinded the policy by reason of the alleged misrepresentation and no evidence was presented by respondent company to show that the questions
concealment of material facts. The premiums paid on the policy were refunded. appearing in Part II of the application for insurance were asked, explained to and
The petitioners filed a complaint in the Insurance Commission. The latter understood by the deceased so as to prove concealment on his part. This couldn’t
dismissed the complaint.
3 – EXECUTIVE | Ab Initio Group

59
INSURANCE MIDTERMS (Weeks 1 to 8)
be accepted because the insured signed the form. He affirmed the correctness of (c) The premium, or if the insurance is of a character where the exact premium is
all the entries. only determinable upon the termination of the contract, a statement of the basis
and rates upon which the final premium is to be determined;
The company records show that the deceased was examined by Dr. Victoriano Lim (d) The property or life insured;
and was found to be diabetic and hypertensive. He was also found to have (e) The interest of the insured in property insured, if he is not the absolute owner
suffered from hepatoma. Because of the concealment made by the deceased, the thereof;
company was thus misled into accepting the risk and approving his application as (f) The risks insured against; and
medically fit.  (g) The period during which the insurance is to continue.

WEEK 8 Sec. 52. Cover notes may be issued to bind insurance temporarily pending the
issuance of the policy.  Within sixty days after the issue of the cover note, a policy
shall be issued in lieu thereof, including within its terms the identical insurance
THE POLICY bound under the cover note and the premium therefor. 
Cover notes may be extended or renewed beyond such sixty days with the written
approval of the Commissioner if he determines that such extension is not contrary
Sec. 49. The written instrument in which a contract of insurance is set forth, is to and is not for the purpose of violating any provisions of this Code.  The
called a policy of insurance. Commissioner may promulgate rules and regulations governing such extensions
for the purpose of preventing such violations and may by such rules and
Sec. 50. The policy shall be in printed form which may contain blank spaces; and regulations dispense with the requirement of written approval by him in the case
any word, phrase, clause, mark, sign, symbol, signature, number, or word of extension in compliance with such rules and regulations.
necessary to complete the contract of insurance shall be written on the blank
spaces provided therein. Sec. 232. (1)  No insurance company authorized to transact business in the
Philippines shall issue, deliver, sell or use any variable contract in the Philippines,
Any rider, clause, warranty or endorsement purporting to be part of the contract of unless and until such company shall have satisfied the Commissioner that its
insurance and which is pasted or attached to said policy is not binding on the financial and general condition and its methods of operations, including the issue
insured, unless the descriptive title or name of the rider, clause, warranty or and sale of variable contracts, are not and will not be hazardous to the public or to
endorsement is also mentioned and written on the blank spaces provided in the its policy and contract owners. No foreign insurance company shall be authorized
policy. to issue, deliver or sell any variable contract in the Philippines, unless it is likewise
authorized to do so by the laws of its domicile.
Unless applied for by the insured or owner, any rider, clause, warranty or
endorsement issued after the original policy shall be countersigned by the insured (2) The term "variable contract" shall mean any policy or contract on
or owner, which countersignature shall be taken as his agreement to the contents either a group or on an individual basis issued by an insurance company providing
of such rider, clause, warranty or endorsement. for benefits or other contractual payments or values thereunder to vary so as to
Group insurance and group annuity policies, however, may be typewritten and reflect investment results of any segregated portfolio of investments or of a
need not be in printed form. designated separate account in which amounts received in connection with such
contracts shall have been placed and accounted for separately and apart from
Sec. 51. A policy of insurance must specify:chanroblesvirtuallawlibrary other investments and accounts. This contract may also provide benefits or values
incidental thereto payable in fixed or variable amounts, or both. It shall not be
(a) The parties between whom the contract is made; deemed to be a "security"  or"securities" as defined in The Securities Act, as
(b) The amount to be insured except in the cases of open or running policies;

3 – EXECUTIVE | Ab Initio Group

60
INSURANCE MIDTERMS (Weeks 1 to 8)
amended, or in the The Investment Company Act, as amended, nor subject to contracts which are required by the terms thereof, to be collected or applied to
regulation under said Acts. one or more designated separate variable accounts shall be placed in such
designated account or accounts. The assets and liabilities of each such separate
(3) In determining the qualifications of a company requesting authority to variable account shall at all times be clearly identifiable and distinguishable from
issue, deliver, sell or use variable contracts, the Commissioner shall always the assets and liabilities in all other accounts of the company. Notwithstanding any
consider the following: (a) the history, financial and general condition of the provision of law to the contrary, the assets held in any such separate variable
company: Provided, That such company, if a foreign company, must have account shall not be chargeable with liabilities arising out of any other business the
deposited with the Commissioner for the benefit and security of its variable company conduct but shall be held and applied exclusively for the benefit of the
contract owners in the Philippines, securities satisfactory to the Commissioner owners or beneficiaries of the variable contracts applicable thereto. In the event of
consisting of bonds of the Government of the Philippines or its instrumentalities the insolvency of the company, the assets of each such separate variable account
with an actual market value of two million pesos; (b) the character, responsibility shall be applied to the contractual claims of the owners or beneficiaries of the
and fitness of the officers and directors of the company; and (c) the law and variable contracts applicable thereto. Except as otherwise specifically provided by
regulation under which the company is authorized in the state of domicile to issue the contract, no sale, exchange or other transfer of assets may be made by a
such contracts. company, between any of its separate accounts or between any other investment
(4) If after notice and hearing, the Commissioner shall find that the account and one or more of its separate accounts, unless in the case of a transfer
company is qualified to issue, deliver, sell or use variable contracts in accordance into a separate account, such transfer is made solely to establish the account or to
with this Code and the regulations and rules issued thereunder, the corresponding support the operation of the contracts with respect to the separate account to
order of authorization shall be issued. Any decision or order denying authority to which the transfer is made, or in case of a transfer from a separate account, such
issue, deliver, sell or use variable contracts shall clearly and distinctly state the transfer would not cause the remaining assets of the account to become less than
reasons and grounds on which it is based. the reserves and other contract liabilities with respect to such separate account.
Such transfer, whether into or from a separate account, shall be made by a
Sec. 233. Any insurance company issuing variable contracts pursuant to this Code transfer of cash, or by a transfer of securities having a valuation which could be
may in its discretion issue contracts providing a combination of fixed amount and readily determined in the market place, provided that such transfer of securities is
variable amount of benefits and for option lump-sum payment of benefits. approved by the Commissioner. The Commissioner may authorize other transfers
among such accounts, if, in his opinion, such transfer would not be inequitable. All
Sec. 234. Every variable contract form delivered or issued for delivery in the amounts and assets allocated to any such separate variable account shall be
Philippines, and every certified form evidencing variable benefits issued pursuant owned by the company and with respect to same the company shall not be nor
to any such contract on a group basis, and the application, rider and endorsement hold itself out to be a trustee.
forms applicable thereto and used in connection therewith, shall be subject to the
prior approval of the Commissioner. Pacific Banking v CA, GR L-41014,28 November 1988

Sec. 236. Variable contracts may be issued on the industrial life basis, provided Facts: An open fire insurance policy was issued to Paramount Shirt Manufacturing
that the pertinent provisions of this Code and of the rules and regulations of the by Oriental Assurance Corporation to indemnify P61,000, caused by fire to the
Commissioner governing variable contracts are complied with in connection with factory’s stocks, materials and supplies. The insured was a debtor of Pacific
such contracts. Banking in the amount of (P800,000) and the goods described in the policy were
held in trust by the insured for Pacific Banking under trust receipts. The policy was
Sec. 237. Every life insurance company authorized under the provisions of this endorsed to Pacific Banking as mortgagee/trustor of the properties insured, with
Code to issue, deliver, sell or use variable contracts shall, in connection with same, the knowledge and consent of private respondent to the effect that “loss if any
establish one or more separate accounts to be known as separate variable under this policy is payable to the Pacific Banking Corporation”. A fire broke out on
accounts. All amounts received by the company in connection with any such the premises destroying the goods contained in the building. The bank sent a
3 – EXECUTIVE | Ab Initio Group

61
INSURANCE MIDTERMS (Weeks 1 to 8)
letter of demand to Oriental for indemnity. The company wasn’t ready to give insured of other insurance upon the same property, the total absence of such
since it was awaiting the adjuster’s report. The company then made an excuse notice nullifies the policy.
that the insured had not filed any claim with it, nor submitted proof of loss which 2) Generally, the cause of action on the policy accrues when the loss occurs,
is a clear violation of Policy Condition no. 11, as a result, determination of the but when the policy provides that no action shall be brought unless the claim is
liability of private respondent could not be made. Pacific Banking filed in the trial first presented extrajudicially in the manner provided in the policy, the cause of
court an action for a sum of money for P61,000 against Oriental Assurance. At the action will accrue from the time the insurer finally rejects the claim for payment. In
trial, petitioner presented communications of the insurance adjuster to Asian the case at bar, policy condition no. 11 specifically provides that the insured shall
Surety revealing undeclared co-insurances with the following: P30,000 with on the happening of any loss or damage give notice to the company and shall
Wellington Insurance; P25,000 with empire surety and P250,000 with Asian Surety within 15 days after such loss or damage deliver to the private respondent (a) a
undertaken by insured Paramount on the same property covered by its policy with claim in writing giving particular account as to the articles or goods destroyed and
Oriental whereas the only co-insurances declared in the subject policy are those of the amount of the loss or damage and (b) particulars of all other insurances, if
P30,000 with Malayan P50,000 with South Sea and P25,000 with Victory. The any.
defense of fraud, in the form of non-declaration of co-insurances which was not Twenty-four days after the fire did petitioner merely wrote letters to private
pleaded in the answer, was also not pleaded in the motion to dismiss. The trial respondent to serve a notice of loss. It didn’t even furnish other documents.
court denied the respondent’s motion. Oriental filed another motion to include Instead, petitioner shifted upon private respondent the burden of fishing out the
additional evidence of the co-insurance which could amount to fraud. The trial necessary information to ascertain particular account of the articles destroyed by
court still made Oriental liable for P61,000. The CA reversed the trial court fire as well as the amount of loss. Since the required claim by insured, together
decision. Pacific Banking filed a motion for reconsideration of the said decision of with the preliminary submittal of relevant documents had not been complied with,
the respondent Court of Appeals, but this was denied for lack of merit. it follows that private respondent could not be deemed to have finally rejected
Issues: petitioner’s claim and therefore there was no cause of action.
1. WON unrevealed co-insurances violated policy condition no. 3 It appearing that insured has violated or failed to perform the conditions under
2. WON the insured failed to file the required proof of loss prior to no.3 and 11 of the contract, and such violation or want of performance has not
court action been waived by the insurer, the insured cannot recover, much less the herein
Held: petitioner.
Yes, petition dismissed.
Ratio: 1) Policy condition no.3 explicitly provides: Oriental Assurance v Court of Appeals, GR 94052, 9 August 1991
“The insured shall give notice to the Company of any insurance already effected,
or which may subsequently be effected, covering any of the property hereby Facts: An open fire insurance policy was issued to Paramount Shirt Manufacturing
insured, and unless such notice be given and the particulars of such insurance or by Oriental Assurance Corporation to indemnify P61,000, caused by fire to the
insurances be stated in or endorsed on this Policy by or on behalf of the company factory’s stocks, materials and supplies. The insured was a debtor of Pacific
before the occurrence of any loss or damage, all benefit under this policy shall be Banking in the amount of (P800,000) and the goods described in the policy were
forfeited”. held in trust by the insured for Pacific Banking under trust receipts. The policy was
The insured failed to reveal, before the loss, three other insurances. Had the endorsed to Pacific Banking as mortgagee/trustor of the properties insured, with
insurer known that there were many co-insurances, it could have hesitated or the knowledge and consent of private respondent to the effect that “loss if any
plainly desisted from entering such contract. Hence, the insured was guilty of clear under this policy is payable to the Pacific Banking Corporation”. A fire broke out on
fraud. Concrete evidence of fraud or false declaration by the insured was furnished the premises destroying the goods contained in the building. The bank sent a
by the petitioner itself when the facts alleged in the policy under clauses “Co- letter of demand to Oriental for indemnity. The company wasn’t ready to give
insurances Declared” and “Other Insurance Clause” are materially different from since it was awaiting the adjuster’s report. The company then made an excuse
the actual number of con-insurances taken over the subject property. As the that the insured had not filed any claim with it, nor submitted proof of loss which
insurance policy against fire expressly required that notice should be given by the is a clear violation of Policy Condition no. 11, as a result, determination of the
3 – EXECUTIVE | Ab Initio Group

62
INSURANCE MIDTERMS (Weeks 1 to 8)
liability of private respondent could not be made. Pacific Banking filed in the trial 2) Generally, the cause of action on the policy accrues when the loss occurs,
court an action for a sum of money for P61,000 against Oriental Assurance. At the but when the policy provides that no action shall be brought unless the claim is
trial, petitioner presented communications of the insurance adjuster to Asian first presented extrajudicially in the manner provided in the policy, the cause of
Surety revealing undeclared co-insurances with the following: P30,000 with action will accrue from the time the insurer finally rejects the claim for payment. In
Wellington Insurance; P25,000 with empire surety and P250,000 with Asian Surety the case at bar, policy condition no. 11 specifically provides that the insured shall
undertaken by insured Paramount on the same property covered by its policy with on the happening of any loss or damage give notice to the company and shall
Oriental whereas the only co-insurances declared in the subject policy are those of within 15 days after such loss or damage deliver to the private respondent (a) a
P30,000 with Malayan P50,000 with South Sea and P25,000 with Victory. The claim in writing giving particular account as to the articles or goods destroyed and
defense of fraud, in the form of non-declaration of co-insurances which was not the amount of the loss or damage and (b) particulars of all other insurances, if
pleaded in the answer, was also not pleaded in the motion to dismiss. The trial any.
court denied the respondent’s motion. Oriental filed another motion to include Twenty-four days after the fire did petitioner merely wrote letters to private
additional evidence of the co-insurance which could amount to fraud. The trial respondent to serve a notice of loss. It didn’t even furnish other documents.
court still made Oriental liable for P61,000. The CA reversed the trial court Instead, petitioner shifted upon private respondent the burden of fishing out the
decision. Pacific Banking filed a motion for reconsideration of the said decision of necessary information to ascertain particular account of the articles destroyed by
the respondent Court of Appeals, but this was denied for lack of merit. fire as well as the amount of loss. Since the required claim by insured, together
Issues: with the preliminary submittal of relevant documents had not been complied with,
1. WON unrevealed co-insurances violated policy condition no. 3 it follows that private respondent could not be deemed to have finally rejected
2. WON the insured failed to file the required proof of loss prior to petitioner’s claim and therefore there was no cause of action.
court action It appearing that insured has violated or failed to perform the conditions under
no.3 and 11 of the contract, and such violation or want of performance has not
Held: been waived by the insurer, the insured cannot recover, much less the herein
Yes, petition dismissed. petitioner.

Ratio: 1) Policy condition no.3 explicitly provides: Perez v CA, GR 112329,28 January 2000
“The insured shall give notice to the Company of any insurance already effected,
or which may subsequently be effected, covering any of the property hereby
insured, and unless such notice be given and the particulars of such insurance or Facts:
insurances be stated in or endorsed on this Policy by or on behalf of the company Primitivo B. Perez had been insured with the BF Lifeman Insurance Corporation for
before the occurrence of any loss or damage, all benefit under this policy shall be P20,000.00. Sometime in October 1987, an agent of the insurance corporation,
forfeited”. visited Perez in Quezon and convinced him to apply for additional insurance
The insured failed to reveal, before the loss, three other insurances. Had the coverage of P50,000.00. Virginia A. Perez, Primitivo’s wife, paid P2,075.00 to
insurer known that there were many co-insurances, it could have hesitated or the agent. The receipt issued indicated the amount received was a "deposit."
plainly desisted from entering such contract. Hence, the insured was guilty of clear Unfortunately, the agent lost the application form accomplished by Perez and he
fraud. Concrete evidence of fraud or false declaration by the insured was furnished asked the latter to fill up another application form. The agent sent
by the petitioner itself when the facts alleged in the policy under clauses “Co- the application for additional insurance of Perez to the Quezon office. Such was
insurances Declared” and “Other Insurance Clause” are materially different from supposed to forwarded to the Manila office.
the actual number of con-insurances taken over the subject property. As the Perez drowned. His application papers for the additional insurance of P50,000.00
insurance policy against fire expressly required that notice should be given by the were still with the Quezon. It was only after some time that the papers were
insured of other insurance upon the same property, the total absence of such brought to Manila. Without knowing that Perez died, BF Lifeman Insurance
notice nullifies the policy.
3 – EXECUTIVE | Ab Initio Group

63
INSURANCE MIDTERMS (Weeks 1 to 8)
Corporation approved the application and issued the corresponding policy for the BF Lifeman didn’t give its assent when it merely received the application form and
P50,000.00. all the requisite supporting papers of the applicant. This happens only when it
Petitioner Virginia Perez went to Manila to claim the benefits under the insurance gives a policy.
policies of the deceased. She was paid P40,000.00 under the first insurance policy It is not disputed, however, that when Primitivo died on November 25, 1987,
for P20,000.00 but the insurance company refused to pay the claim under the his application papers for additional insurance coverage were still with the branch
additional policy coverage of P50,000.00, the proceeds of which amount to office of respondent corporation in Quezon. Consequently, there was absolutely no
P150,000.00. way the acceptance of the application could have been communicated to the
The insurance company maintained that the insurance for P50,000.00 had not applicant for the latter to accept inasmuch as the applicant at the time was already
been perfected at the time of the death of Primitivo Perez. Consequently, the dead.
insurance company refunded the amount paid. Petitioner insists that the condition imposed by BF that a policy must have been
BF Lifeman Insurance Corporation filed a complaint against Virginia Perez seeking delivered to and accepted by the proposed insured in good health is potestative,
the rescission and declaration of nullity of the insurance contract in question. being dependent upon the will of the corporation and is therefore void. The court
Petitioner Virginia A. Perez, on the other hand, averred that the deceased had didn’t agree. A potestative condition depends upon the exclusive will of one of
fulfilled all his prestations under the contract and all the elements of a valid the parties and is considered void. The Civil Code states: When the fulfillment of
contract are present. the condition depends upon the sole will of the debtor, the conditional obligation
On October 25, 1991, the trial court rendered a decision in favor of petitioner shall be void.
ordering respondent to pay 150,000 pesos. The Court of Appeals, however, The following conditions were imposed by the respondent company for the
reversed the decision of the trial court saying that the insurance contract for perfection of the contract of insurance: a policy must have been issued, the
P50,000.00 could not have been perfected since at the time that the policy was premiums paid, and the policy must have been delivered to and accepted by the
issued, Primitivo was already dead. applicant while he is in good health.
Petitioner’s motion for reconsideration having been denied by respondent court, The third condition isn’t potestative, because the health of the applicant at the
the instant petition for certiorari was filed on the ground that there was a time of the delivery of the policy is beyond the control or will of the insurance
consummated contract of insurance between the deceased and BF Lifeman company. Rather, the condition is a suspensive one whereby the acquisition of
Insurance Corporation. rights depends upon the happening of an event which constitutes the condition. In
this case, the suspensive condition was the policy must have been delivered and
Issue: WON the widow can receive the proceeds of the 2 nd insurance accepted by the applicant while he is in good health. There was non-fulfillment of
policy the condition, because the applicant was already dead at the time the policy was
issued.
Held: No. Petition dismissed. As stated above, a contract of insurance, like other contracts, must be assented to
by both parties either in person or by their agents. So long as an application for
Ratio: insurance has not been either accepted or rejected, it is merely an offer or
Perez’s application was subject to the acceptance of private respondent BF proposal to make a contract. The contract, to be binding from the date
Lifeman Insurance Corporation. The perfection of the contract of insurance of application, must have been a completed contract.
between the deceased and respondent corporation was further conditioned with The insurance company wasn’t negligent because delay in acting on
the following requisites stated in the application form: the application does not constitute acceptance even after payment. The
"there shall be no contract of insurance unless and until a policy is issued on corporation may not be penalized for the delay in the processing of
this application and that the said policy shall not take effect until the premium has the application papers due to the fact that process in a week wasn’t the usual
been paid and the policy delivered to and accepted by me/us in person while timeframe in fixing the application. Delay could not be deemed unreasonable so as
I/We, am/are in good health." to constitute gross negligence.

3 – EXECUTIVE | Ab Initio Group

64
INSURANCE MIDTERMS (Weeks 1 to 8)
Enriquez v Sun Life, GR L-15895, 29 November 1924 Civil Code
Art. 1319 (formerly Art.1262)
FACTS: Art. 1319. Consent is manifested by the meeting of the offer and the
 September 24, 1917: Joaquin Herrer made application to the Sun Life acceptance upon the thing and the cause which are to constitute the
Assurance Company of Canada through its office in Manila for a life contract. The offer must be certain and the acceptance absolute. A
annuity qualified acceptance constitutes a counter-offer.
 2 days later: he paid P6,000 to the manager of the company's Manila Acceptance made by letter or telegram does not bind the offerer
office and was given a receipt except from the time it came to his knowledge. The contract, in such a
 according to the provisional receipt, 3 things had to be accomplished by case, is presumed to have been entered into in the place where the
the insurance company before there was a contract:  offer was made.
 (1) There had to be a medical examination of the applicant;  not perfected because it has not been proved satisfactorily that the
-check acceptance of the application ever came to the knowledge of the applicant
 (2) there had to be approval of the application by the head office
of the company; and - check
 (3) this approval had in some way to be communicated by the
company to the applicant - ?
 November 26, 1917: The head office at Montreal, Canada gave notice of
acceptance by cable to Manila but this was not mailed Tang v CA, GR L48563, 25 May 1979
 December 4, 1917: policy was issued at Montreal
 December 18, 1917: attorney Aurelio A. Torres wrote to the Manila office FACTS:
of the company stating that Herrer desired to withdraw his application  September 24, 1917: Joaquin Herrer made application to the Sun Life
 December 19, 1917: local office replied to Mr. Torres, stating that the Assurance Company of Canada through its office in Manila for a life
policy had been issued, and called attention to the notification of annuity
November 26, 1917  2 days later: he paid P6,000 to the manager of the company's Manila
 December 21, 1917 morning: received by Mr. Torres office and was given a receipt
 December 20, 1917: Mr. Herrer died  according to the provisional receipt, 3 things had to be accomplished by
 Rafael Enriquez, as administrator of the estate of the late Joaquin Ma. the insurance company before there was a contract: 
Herrer filed to recover from Sun Life Assurance Company of  (1) There had to be a medical examination of the applicant;
Canada through its office in Manila for a life annuity -check
 RTC: favored Sun Life Insurance  (2) there had to be approval of the application by the head office
 of the company; and - check
ISSUE: W/N Mr. Herrera received notice of acceptance of his application  (3) this approval had in some way to be communicated by the
thereby perfecting his life annuity company to the applicant - ?
 November 26, 1917: The head office at Montreal, Canada gave notice of
acceptance by cable to Manila but this was not mailed
HELD: NO. Judgment is reversed, and the Enriquez shall have and recover from  December 4, 1917: policy was issued at Montreal
the Sun Life the sum of P6,000 with legal interest from November 20, 1918, until  December 18, 1917: attorney Aurelio A. Torres wrote to the Manila office
paid, without special finding as to costs in either instance. So ordered. of the company stating that Herrer desired to withdraw his application

3 – EXECUTIVE | Ab Initio Group

65
INSURANCE MIDTERMS (Weeks 1 to 8)
 December 19, 1917: local office replied to Mr. Torres, stating that the  RCBC Binondo Branch initially granted a credit facility of P30M to Goyu &
policy had been issued, and called attention to the notification of Sons, Inc.  GOYU’s applied again and through Binondo Branch key officer's
November 26, 1917 Uy’s and Lao’s recommendation, RCBC’s executive committee increased its
 December 21, 1917 morning: received by Mr. Torres credit facility to P50M to P90M and finally to P117M.
 December 20, 1917: Mr. Herrer died  As security, GOYU executed 2 real estate mortgages and 2 chattel
 Rafael Enriquez, as administrator of the estate of the late Joaquin Ma. mortgages in favor of RCBC.  
Herrer filed to recover from Sun Life Assurance Company of  GOYU obtained in its name 10 insurance policy on the mortgaged
Canada through its office in Manila for a life annuity properties from Malayan Insurance Company, Inc. (MICO). In
 RTC: favored Sun Life Insurance February 1992, he was issued 8 insurance policies in favor of
 RCBC.
ISSUE: W/N Mr. Herrera received notice of acceptance of his application  April 27, 1992: One of GOYU’s factory buildings was burned so he claimed
thereby perfecting his life annuity against MICO for the loss who denied contending that the insurance
policies were either attached pursuant to writs of
attachments/garnishments or that creditors are claiming to have a better
HELD: NO. Judgment is reversed, and the Enriquez shall have and recover from right
the Sun Life the sum of P6,000 with legal interest from November 20, 1918, until  GOYU filed a complaint for specific performance and damages at the RTC
paid, without special finding as to costs in either instance. So ordered.  RCBC, one of GOYU’s creditors, also filed with MICO its formal claim over
the proceeds of the insurance policies, but said claims were also denied
for the same reasons that MICO denied GOYU’s claims
 RTC: Confirmed GOYU’s other creditors (Urban Bank, Alfredo Sebastian,
and Philippine Trust Company) obtained their writs of attachment covering
Civil Code an aggregate amount of P14,938,080.23 and ordered that 10 insurance
Art. 1319 (formerly Art.1262) policies be deposited with the court minus the said amount so MICO
deposited P50,505,594.60.  
Art. 1319. Consent is manifested by the meeting of the offer and the
 Another Garnishment of P8,696,838.75 was handed down
acceptance upon the thing and the cause which are to constitute the
 RTC: favored GOYU against MICO for the claim, RCBC for damages and to
contract. The offer must be certain and the acceptance absolute. A
pay RCBC its loan
qualified acceptance constitutes a counter-offer.
 CA: Modified by increasing the damages in favor of GOYU
Acceptance made by letter or telegram does not bind the offerer
 In G.R. No. 128834, RCBC seeks right to intervene in the action between
except from the time it came to his knowledge. The contract, in such a
Alfredo C. Sebastian (the creditor) and GOYU (the debtor), where the
case, is presumed to have been entered into in the place where the
subject insurance policies were attached in favor of Sebastian
offer was made.
 RTC and CA: endorsements do not bear the signature of any officer of
 not perfected because it has not been proved satisfactorily that the
GOYU concluded that the endorsements favoring RCBC as defective.
acceptance of the application ever came to the knowledge of the applicant
ISSUE: W/N RCBC as mortgagee, has any right over the insurance
Rizal Commercial Banking v CA, GR 128833,20 April 1998
policies taken by GOYU, the mortgagor, in case of the occurrence of loss
FACTS:
HELD: YES.

3 – EXECUTIVE | Ab Initio Group

66
INSURANCE MIDTERMS (Weeks 1 to 8)
 mortgagor and a mortgagee have separate and distinct insurable interests issued to private respondent Ngo Hing. Likewise, petitioner Mondragon hand wrote
in the same mortgaged property, such that each one of them may insure at the bottom of the back page of the application form his strong recommendation
the same property for his own sole benefit for the approval of the insurance application.
 although it appears that GOYU obtained the subject insurance policies
naming itself as the sole payee, the intentions of the parties as shown by Then Mondragon received a letter from Grepalife disapproving the insurance
their contemporaneous acts, must be given due consideration in order to application. The letter stated that the said life insurance application for 20-year
better serve the interest of justice and equity endowment plan is not available for minors below seven years old, but Grepalife
 8 endorsement documents were prepared by Alchester in favor of can consider the same under the Juvenile Triple Action Plan, and advised that if
RCBC the offer is acceptable, the Juvenile Non-Medical Declaration be sent to the
 MICO, a sister company of RCBC company. The non-acceptance of the insurance plan by Grepallife was allegedly
 GOYU continued to enjoy the benefits of the credit facilities not communicated by Mondragon to Ngo Hing. Instead, Mondragon wrote back
extended to it by RCBC.  Grepalife again strongly recommending the approval of the 20-year endowment
 GOYU is at the very least estopped from assailing their insurance plan to children, pointing out that since 1954 the customer, especially
operative effects.  the Chinese, were asking for such coverage. It was when things were in such state
 The two courts below erred in failing to see that the promissory notes that on May 29, 1957 Helen Go died of influenza with complication of
which they ruled should be excluded for bearing dates which are after that bronchopneumonia. Thereupon, Ngo Hing sought the payment of the proceeds of
of  the fire, are mere renewals of previous ones the insurance but having failed in his effort, he filed the action for the recovery of
 RCBC has the right to claim the insurance proceeds, in substitution of the the same before the CFI of Cebu, which rendered the adverse decision.
property lost in the fire. Having assigned its rights, GOYU lost its standing
as the beneficiary of the said insurance policies ISSUES:
 insurance company to be held liable for unreasonably delaying and
withholding payment of insurance proceeds, the delay must be wanton, 1) Whether or not binding deposit receipt constituted a temporary
oppressive, or malevolent - not shown contract of the life insurance in question
 Sebastian’s right as attaching creditor must yield to the preferential rights 2) Whether or not private respondent Ngo Hing concealed the state
of RCBC over the Malayan insurance policies as first mortgagee. of health and physical condition if Helen Go, which rendered void
the aforesaid deposit receipt.

Great Pacific Life CA, GR L-31845, 30 April 1979 HELD:

FACTS: The provisions printed on the deposit receipt provide that the binding deposit
receipt is intended to be merely a provisional or temporary insurance contract and
Ngo Hing filed an application with the Great Pacific Assurance Company (Grepalife) only upon compliance of the following conditions:
for twenty-years endowment policy in the amount of P50,000.00 on the life of his
one-year old daughter Helen Go. Said respondent supplied the essential data 1) That the company shall be satisfied that the applicant was insurable on
which petitioner Lapulapu D. Mondrago, Branch Manager of Grepalie in Cebu City standard rates;
wrote on the corresponding form in his own hand writing. Mondragon finally type- 2) That if the company does not accept the application and offers to issue a
wrote the data on the application form which was signed by private respondent policy for a different plan, the insurance contract shall not be binding until
Ngo Hing. The latter paid the annual premium to the Company, but he retained a applicant accepts the policy offered; otherwise the deposit shall be
certain amount as his commission for being duly authorized agent of Grepalife. refunded and
Upon the payment of the insurance premium, the binding deposit receipt was
3 – EXECUTIVE | Ab Initio Group

67
INSURANCE MIDTERMS (Weeks 1 to 8)
3) That if the applicant is not able according to the standard rates, and the The contract of insurance is one of perfect good faith uberrimas fides meaning
company disapproves the application, the insurance applied for shall not good faith, absolute and perfect candor or openness and honesty; the absence of
be in force at any time, and the premium paid shall be returned to the any concealment or demotion, however slight, not for the alone but equally so for
applicant. the insurer. Concealment is not neglect to communicate that which a party knows
and ought to communicate. Whether intentional or unintentional the concealment
Clearly implied from the aforesaid conditions is that the binding deposit receipt in entitles the insurer to rescind the contract of insurance. SC held that no insurance
question is merely an acknowledgment, on behalf of the company, that the latter’s contract was perfected between the parties with the noncompliance of the
branch office had received from the applicant the insurance premium and had conditions provided in the binding receipt and concealment, as legally defined,
accepted the application subject for processing by the insurance company; and having been committed by herein private respondet.
that the latter either approve or reject the same on the basis of whether or not the
applicant is insurable on standard rates. Since petitioner Grapalife disapproved the De Lim v Sun Life of Canada, GR 15774,29 Nov 1920
insurance application of respondent Ngo Hing, the binding deposit receipt in
question had never become in force at any time. FACTS:

Upon this premise, the binding deposit receipt is, manifestly, merely conditional Ngo Hing filed an application with the Great Pacific Assurance Company (Grepalife)
and does not insure outright. As held by this court, where an agreement is made for twenty-years endowment policy in the amount of P50,000.00 on the life of his
between the applicant and the agent, no liability shall attach until the principal one-year old daughter Helen Go. Said respondent supplied the essential data
approves the risk and a receipt is given by the agent. The acceptance is merely which petitioner Lapulapu D. Mondrago, Branch Manager of Grepalie in Cebu City
conditional and is subordinated to the act of the company in approving or rejecting wrote on the corresponding form in his own hand writing. Mondragon finally type-
the application. Thus, in life insurance, a binding slip or binding receipt does not wrote the data on the application form which was signed by private respondent
insure by itself Ngo Hing. The latter paid the annual premium to the Company, but he retained a
certain amount as his commission for being duly authorized agent of Grepalife.
A contract of insurance, like other contract, must be assented to by both parties Upon the payment of the insurance premium, the binding deposit receipt was
either in person or by their agents. The contract, to be binding from the date of issued to private respondent Ngo Hing. Likewise, petitioner Mondragon hand wrote
the application, must have been a completed contract, one that leaves nothing to at the bottom of the back page of the application form his strong recommendation
be done, nothing to be completed, nothing to be passed upon, or determined, for the approval of the insurance application.
before it shall take effect. There can be no contract of insurance unless the minds
of the parties have met in agreement. Then Mondragon received a letter from Grepalife disapproving the insurance
application. The letter stated that the said life insurance application for 20-year
Relative to the second issue of alleged concealment, the Court is of the firm belief endowment plan is not available for minors below seven years old, but Grepalife
that private respondent had deliberately concealed the state of health and physical can consider the same under the Juvenile Triple Action Plan, and advised that if
condition of his daughter Helen Go. Where private respondent supplied the the offer is acceptable, the Juvenile Non-Medical Declaration be sent to the
required essential data for the insurance application form, he was fully aware that company. The non-acceptance of the insurance plan by Grepallife was allegedly
his one-year old daughter is typically a mongoloid child. Such a congenital physical not communicated by Mondragon to Ngo Hing. Instead, Mondragon wrote back
defect could never be ensconced nor disguised. Nonetheless, private respondent, Grepalife again strongly recommending the approval of the 20-year endowment
in apparent bad faith, withheld the fact material to the risk to be assumed by the insurance plan to children, pointing out that since 1954 the customer, especially
insurance company. As an insurance agent of Grepalife, he ought to know, as he the Chinese, were asking for such coverage. It was when things were in such state
surely must have known. that on May 29, 1957 Helen Go died of influenza with complication of
bronchopneumonia. Thereupon, Ngo Hing sought the payment of the proceeds of

3 – EXECUTIVE | Ab Initio Group

68
INSURANCE MIDTERMS (Weeks 1 to 8)
the insurance but having failed in his effort, he filed the action for the recovery of approves the risk and a receipt is given by the agent. The acceptance is merely
the same before the CFI of Cebu, which rendered the adverse decision. conditional and is subordinated to the act of the company in approving or rejecting
the application. Thus, in life insurance, a binding slip or binding receipt does not
ISSUES: insure by itself

1 Whether or not binding deposit receipt constituted a A contract of insurance, like other contract, must be assented to by both parties
temporary contract of the life insurance in question either in person or by their agents. The contract, to be binding from the date of
2 Whether or not private respondent Ngo Hing concealed the application, must have been a completed contract, one that leaves nothing to
the state of health and physical condition if Helen Go, be done, nothing to be completed, nothing to be passed upon, or determined,
which rendered void the aforesaid deposit receipt. before it shall take effect. There can be no contract of insurance unless the minds
of the parties have met in agreement.
HELD:
Relative to the second issue of alleged concealment, the Court is of the firm belief
The provisions printed on the deposit receipt provide that the binding deposit that private respondent had deliberately concealed the state of health and physical
receipt is intended to be merely a provisional or temporary insurance contract and condition of his daughter Helen Go. Where private respondent supplied the
only upon compliance of the following conditions: required essential data for the insurance application form, he was fully aware that
his one-year old daughter is typically a mongoloid child. Such a congenital physical
4) That the company shall be satisfied that the applicant was insurable on defect could never be ensconced nor disguised. Nonetheless, private respondent,
standard rates; in apparent bad faith, withheld the fact material to the risk to be assumed by the
5) That if the company does not accept the application and offers to issue a insurance company. As an insurance agent of Grepalife, he ought to know, as he
policy for a different plan, the insurance contract shall not be binding until surely must have known.
applicant accepts the policy offered; otherwise the deposit shall be
refunded and The contract of insurance is one of perfect good faith uberrimas fides meaning
6) That if the applicant is not able according to the standard rates, and the good faith, absolute and perfect candor or openness and honesty; the absence of
company disapproves the application, the insurance applied for shall not any concealment or demotion, however slight, not for the alone but equally so for
be in force at any time, and the premium paid shall be returned to the the insurer. Concealment is not neglect to communicate that which a party knows
applicant. and ought to communicate. Whether intentional or unintentional the concealment
entitles the insurer to rescind the contract of insurance. SC held that no insurance
Clearly implied from the aforesaid conditions is that the binding deposit receipt in contract was perfected between the parties with the noncompliance of the
question is merely an acknowledgment, on behalf of the company, that the latter’s conditions provided in the binding receipt and concealment, as legally defined,
branch office had received from the applicant the insurance premium and had having been committed by herein private respondent.
accepted the application subject for processing by the insurance company; and
that the latter either approve or reject the same on the basis of whether or not the Sec. 53. The insurance proceeds shall be applied exclusively to the proper interest
applicant is insurable on standard rates. Since petitioner Grapalife disapproved the of the person in whose name or for whose benefit it is made unless otherwise
insurance application of respondent Ngo Hing, the binding deposit receipt in specified in the policy.
question had never become in force at any time.
Sec. 54. When an insurance contract is executed with an agent or trustee as the
Upon this premise, the binding deposit receipt is, manifestly, merely conditional insured, the fact that his principal or beneficiary is the real party in interest may be
and does not insure outright. As held by this court, where an agreement is made indicated by describing the insured as agent or trustee, or by other general words
between the applicant and the agent, no liability shall attach until the principal in the policy.
3 – EXECUTIVE | Ab Initio Group

69
INSURANCE MIDTERMS (Weeks 1 to 8)
ISSUE: Whether or not there is privity between Bonifacio Bro and Ayala
Sec. 55. To render an insurance effected by one partner or part-owner, applicable Auto against the insurance company.
to the interest of his co-partners or other part-owners, it is necessary that the
terms of the policy should be such as are applicable to the joint or common
interest. RULING: No, there is no privity between Bonifacio Bro and Ayala Auto against the
insurance company.  Judgment of the CFI is affirmed.
Sec. 56. When the description of the insured in a policy is so general that it may
comprehend any person or any class of persons, only he who can show that it was General rule is that, contracts take effect only between the parties thereto
intended to include him can claim the benefit of the policy. exception is when some specific instances provided by law where the contract
contains some stipulation in favor of a third person - stipulation pour autrui.
Sec. 57. A policy may be so framed that it will inure to the benefit of
whomsoever, during the continuance of the risk, may become the owner of the The provision is in favor of a third person not a party to the contract. The third
interest insured. person is allowed to avail himself of a benefit granted to him by the terms of the
contract, provided that the contracting parties have clearly and deliberately
Sec. 58. The mere transfer of a thing insured does not transfer the policy, but conferred a favor upon such person.
suspends it until the same person becomes the owner of both the policy and the
thing insured. The stipulation pour autrui must be clearly expressed, however, in the [resent case
no stipulation was expressed. The "loss payable" clause of the insurance policy
stipulates that "Loss, if any, is payable to H.S. Reyes, Inc." indicating that it was
Bonifacio Brothers v Mora, GR L-20853,29 May 1967 only the H.S. Reyes, Inc. which they intended to benefit. The stipulation merely
establishes the procedure that the insured has to follow in order to be entitled to
FACTS: Enrique Mora, owner of Oldsmobile sedan model 1956, mortgaged it to indemnity for repair.
H.S. Reyes, Inc., with the condition that they would be the beneficiary of its
insurance. A policy of insurance is a distinct and independent contract between the insured
and insurer, and third persons have no right either in a court of equity, or in a
On June 23, 1959 the sedan was insured with State Bonding & Insurance Co., Inc. court of law, to the proceeds of it, unless there be some contract of trust,
During the period of effectivity, the sedan met an accident and it was appraised expressed or implied between the insured and third person. The word "loss" in
by Bayne Adjustment Co. and repaired it with Bonifacio Bros. and the parts were insurance law embraces injury or damage. The injury or damage sustained by the
supplied by Ayala Auto Parts Co. This was all done without the knowledge of H.S. insured in consequence of the happening of one or more of the accidents or
Reyes. Enrique was billed P2,102.73 through Bayne. The insurance company drew misfortune against which the insurer, in consideration of the premium, has
a check deducting P100 for franchise and entrusted it to Bayne payable to Enrique undertaken to indemnify the insured.
or H.S. Reyes. Still unpaid, the sedan was delivered to Enrique without the
Knowledge of H.S. Reyes. Heirs of Maramag v Maramag, GR 181132,5 June 2009

Bonifacio Bros and Ayala Auto filed in the MTC on the theory that the insurance FACTS: The case stems from a petition filed against respondents with the RTC
proceeds should be paid directly to them. for revocation and/or reduction of insurance proceeds for being void and/or
inofficious.
CFI affirmed MTC’s decision that H.S. Reyes, Inc. as having a better right.
The petition alleged that:

3 – EXECUTIVE | Ab Initio Group

70
INSURANCE MIDTERMS (Weeks 1 to 8)
(1) Petitioners were the legitimate wife and children of Loreto Maramag (Loreto),
while respondents were Loreto’s illegitimate family; RULING: Yes, illegitimate children can be beneficiaries in an
(2) Eva de Guzman Maramag (Eva) was a concubine of Loreto and a suspect in insurance contract. Section 53 of the Insurance Code states that the insurance
the killing of the latter, thus, she is disqualified to receive any proceeds from his proceeds shall be applied exclusively to the proper interest of the person in whose
insurance policies from Insular Life Assurance Company, Ltd. (Insular) and Great name or for whose benefit it is made unless otherwise specified in the policy.
Pacific Life Assurance Corporation (Grepalife) Pursuant thereto, it is obvious that the only persons entitled to claim the insurance
(3) The illegitimate children of Loreto—Odessa, Karl Brian, and Trisha Angelie— proceeds are either the insured, if still alive; or the beneficiary, if the insured is
were entitled only to one-half of the legitime of the legitimate children, thus, the already deceased, upon the maturation of the policy. The exception to this rule is
proceeds released to Odessa and those to be released to Karl Brian and Trisha a situation where the insurance contract was intended to benefit third persons
Angelie were inofficious and should be reduced; and who are not parties to the same in the form of favorable stipulations or indemnity.
(4) Petitioners could not be deprived of their legitimes, which should be satisfied In such a case, third parties may directly sue and claim from the insurer.
first.
Petitioners are third parties to the insurance contracts with Insular and
Insular admitted that Loreto misrepresented Eva as his legitimate wife and Grepalife and, thus, are not entitled to the proceeds thereof. Accordingly,
Odessa, Karl Brian, and Trisha Angelie as his legitimate children, and that they respondents Insular and Grepalife have no legal obligation to turn over the
filed their claims for the insurance proceeds of the insurance policies; that when it insurance proceeds to petitioners. The revocation of Eva as a beneficiary in one
ascertained that Eva was not the legal wife of Loreto, it disqualified her as policy and her disqualification as such in another are of no moment considering
a beneficiary and divided the proceeds among Odessa, Karl Brian, and Trisha that the designation of the illegitimate children as beneficiaries in Loreto’s
Angelie, as the remaining designated beneficiaries; and that it released Odessa’s insurance policies remains valid. Because no legal proscription exists in naming as
share as she was of age, but withheld the release of the shares of  minors Karl beneficiaries the children of illicit relationships by the insured, the shares of Eva in
Brian and Trisha Angelie pending submission of letters of guardianship. Insular the insurance proceeds, whether forfeited by the court in view of the prohibition
alleged that the complaint or petition failed to state a cause of action insofar as on donations under Article 739 of the Civil Code or by the insurers themselves for
it sought to declare as void the designation of Eva as beneficiary, because reasons based on the insurance contracts, must be awarded to the said illegitimate
Loreto revoked her designation as such in Policy No. A001544070 and it children, the designated beneficiaries, to the exclusion of petitioners. It is only
disqualified her in Policy No. A001693029; and insofar as it sought to declare as in cases where the insured has not designated any beneficiary, or when
inofficious the shares of Odessa, Karl Brian, and Trisha Angelie, considering that the designated beneficiary is disqualified by law to receive the proceeds, that the
no settlement of Loreto’s estate had been filed nor had the respective shares of insurance policy proceeds shall redound to the benefit of the estate of the insured.
the heirs been determined. Insular further claimed that it was bound to honor the
insurance policies designating the children of Loreto with Eva as beneficiaries Coquia v Fieldmen's lnsurance, GR L-23276,29 November 1968
pursuant to Section 53 of the Insurance Code. Grepalife alleged that Eva was not
designated as an insurance policy beneficiary; that the claims filed by Odessa, Karl FACTS: Enrique Mora, owner of Oldsmobile sedan model 1956, mortgaged it to
Brian, and Trisha Angelie were denied because Loreto was ineligible for insurance H.S. Reyes, Inc., with the condition that they would be the beneficiary of its
due to a misrepresentation in his application form that he was born on December insurance.
10, 1936 and, thus, not more than 65 years old when he signed it in September
2001; that the case was premature, there being no claim filed by the legitimate On June 23, 1959 the sedan was insured with State Bonding & Insurance Co., Inc.
family of Loreto; and that the law on succession does not apply where During the period of effectivity, the sedan met an accident and it was appraised
the designation of insurance beneficiaries is clear. by Bayne Adjustment Co. and repaired it with Bonifacio Bros. and the parts were
supplied by Ayala Auto Parts Co. This was all done without the knowledge of H.S.
ISSUE: Whether or not illegitimate children can be beneficiaries in an Reyes. Enrique was billed P2,102.73 through Bayne. The insurance company drew
insurance contract. a check deducting P100 for franchise and entrusted it to Bayne payable to Enrique
3 – EXECUTIVE | Ab Initio Group

71
INSURANCE MIDTERMS (Weeks 1 to 8)
or H.S. Reyes. Still unpaid, the sedan was delivered to Enrique without the
Knowledge of H.S. Reyes. Sec. 59. A policy is either open, valued or running.

Bonifacio Bros and Ayala Auto filed in the MTC on the theory that the insurance Sec. 60. An open policy is one in which the value of the thing insured is not
proceeds should be paid directly to them. agreed upon, but is left to be ascertained in case of loss.

CFI affirmed MTC’s decision that H.S. Reyes, Inc. as having a better right. Sec. 61. A valued policy is one which expresses on its face an agreement that the
thing insured shall be valued at a specific sum.

ISSUE: Whether or not there is privity between Bonifacio Bro and Ayala Sec. 62. A running policy is one which contemplates successive insurances, and
Auto against the insurance company. which provides that the object of the policy may be from time to time defined,
especially as to the subjects of insurance, by additional statements or
indorsements.
RULING: No, there is no privity between Bonifacio Bro and Ayala Auto
against the insurance company.  Judgment of the CFI is affirmed. Development lnsurance v. lAC, GR 71360, 16 July 1986

General rule is that, contracts take effect only between the parties thereto Facts: Phil. Union Realty Development Corp. (PURDC) insured its building against
exception is when some specific instances provided by law where the contract fire with Development Insurance Corp. (DIC). The policy contained the following
contains some stipulation in favor of a third person - stipulation pour autrui. provision: “This is an open policy as defined in Section 57 of the Insurance Act.  In
the event of loss, whether total or partial, it is understood that the amount of the
The provision is in favor of a third person not a party to the contract. The third loss shall be subject to appraisal and the liability of the company, if established,
person is allowed to avail himself of a benefit granted to him by the terms of the shall be limited to the actual loss, subject to the applicable terms, conditions,
contract, provided that the contracting parties have clearly and deliberately warranties and clauses of this Policy, and in no case shall exceed the amount of
conferred a favor upon such person. the policy." After a fire consumed apart of the building, with PURDC suffering
an appraised value of loss of P508, 867 (later adopted by the trial court and the
The stipulation pour autrui must be clearly expressed, however, in the [resent case appellate court), PURDC filed its claim with DIC, but the latter refused. DIC argued
no stipulation was expressed. The "loss payable" clause of the insurance policy that since the building was worth more than the sum insured, PURDC must be
stipulates that "Loss, if any, is payable to H.S. Reyes, Inc." indicating that it was considered its own insurer for the difference of the amount and the face value
only the H.S. Reyes, Inc. which they intended to benefit. The stipulation merely of the policy and should share pro rata on the loss sustained.
establishes the procedure that the insured has to follow in order to be entitled to
indemnity for repair. Issue: Whether or not DIC is liable for the appraised value of actual loss
sustained by PURDC.
A policy of insurance is a distinct and independent contract between the insured
and insurer, and third persons have no right either in a court of equity, or in a Held: Yes, DIC is liable for the appraised value of actual loss sustained
court of law, to the proceeds of it, unless there be some contract of trust, by PURDC.
expressed or implied between the insured and third person. The word "loss" in
insurance law embraces injury or damage. The injury or damage sustained by the As defined in the provision, which is now Section 60 of the Insurance Code, "an
insured in consequence of the happening of one or more of the accidents or open policy is one in which the value of the thing insured is not agreed upon but
misfortune against which the insurer, in consideration of the premium, has is left to be ascertained in case of loss."
undertaken to indemnify the insured.
3 – EXECUTIVE | Ab Initio Group

72
INSURANCE MIDTERMS (Weeks 1 to 8)
This means that the actual loss, as determined, will represent the total indemnity
due the insured from the insurer except only that the total indemnity shall not Held: Yes
exceed the face value of the policy. The actual loss having been ascertained in this Ratio:
case, the Court will respect such factual determination in the absence of proof that  Fidelity bond is in the nature of a contract of insurance against loss from
it was arrived at arbitrarily. There is no such showing. Hence, applying the open misconduct. Subject to Insurance Act
policy clause as expressly agreed upon by the parties in their contract, PURDC is  Sec 61-A: A condition, stipulation or agreement in any policy of insurance,
entitled to the payment of indemnity under the said contract in the full amount of limiting the time for commencing an action thereunder to a period of less than 1
the appraised value of actual loss year from the time when the cause of action accrues is VOID
 Cause of action (1) legal right (2) obligation (3) act or omission in violation of
Sec. 63. A condition, stipulation, or agreement in any policy of insurance, limiting legal right
the time for commencing an action thereunder to a period of less than one year  Cause of action ACCRUED when insurance company REFUSED to comply with
from the time when the cause of action accrues, is void. the bond NOT from the time of filing claim of loss
 Stipulation in contract is VOID because of Sec 61-A
 Condition of previous conviction was deleted by express agreement
 Surety assumed solidary liability so creditor may go against any one of the
solidary debtors.

Case remanded to CFI with instructions to require defendant to answer


ACCFA v Alpha lnsurance, GR L-24566, 29 July 1968 Ang v Fulton Fire lnsurance, GR L-15862, 31 July 1961

FACTS: FACTS:
 Asingan Farmers’ Cooperative Marketing Assoc Inc wanted to have a guarantee (chronological arrangement of facts)
against loss on account of personal dishonesty, amounting to larceny or estafa of 1. Sept 1953: P&S Department Store (Sally Ang) was insured with Fulton Fire
its Secretary-Treasurer, Ricardo Ladines. Insurance Coover stocks of general merchandise, consisting principally of dried
 Alpha Insurance & Surety then issued a fidelity bond for P5k with Ladines as goods. It contained a stipulation stating “if the claim is made and rejected but no
principal and Alpha as solidary surety. FACOMA then assigned its rights to ACCFA action is commenced within 12 months after such rejection, all benefits under the
(Agricultural Credit Cooperative and Financing Admin). policy would be forfeited.”
 Ladines misappropriated P11,513 of FACOMA funds to his personal benefit. On
October 1958, ACCFA notified in writing the survey company and presented proof 2. Dec 1954: fire consumed the store and Ang filed claims. However, on April
of loss. Surety Alpha refused to pay. ACCFA filed suit 30 May 1960. 1956, Fulton denied claims.
 Alpha seeks to dismiss due to a provision in the fidelity bond saying that no
action shall be had unless commenced within one year from the making of the 3. January 1955: Paulo Ang and 10 others were charged for arson in the Justice of
claim for loss. Another is that the complaint failed to show an civil or criminal the Peace Court of Ilocos Norte. Remanded to CFI and the latter acquitted Paulo
action filed against Ladines as required by the conditions of the bond. Last, Ang of the crime of arson.
Ladines is an indispensable party but was not joined.
 CFI denied dismissal but, upon reconsideration, reversed and dismissed the 4. May 1956: Ang filed case against Fulton’s agent. On Sept 1957, this case was
complaint because the action was filed beyond the contractual limitation period dismissed without prejudice.

Issue: Was contractual limitation void due to Sec. 61-A of Insurance


Act?
3 – EXECUTIVE | Ab Initio Group

73
INSURANCE MIDTERMS (Weeks 1 to 8)
5. May 1958: Ang filed the present case against Fulton. CFI ruled in favor of Ang, Tan took from Sun Insurance a Php 300,000 policy to cover his electrical store in
holding that the 12-month prescription period (from insurer’s denial of claim) was Iloilo city. Tan’s request for an indemnity in 1983 was repeatedly denied, firstly in
suspended by the case against the agent. 1984. He wrote for a reconsideration in the same year. This was rejected in 1985,
prompting him to file a civil case in the same year.  The insurance company filed a
ISSUE: whether or not the filing of the previous suit against the agent motion to dismiss due to prescription in 1987, but this was denied.
suspended the running of the prescriptive period. The company went to the court of appeals to petition the same thing, but this was
denied.
HELD:
1. SC ruled that no, period was not suspended, and action had already prescribed. Issue:
CFI ruling set aside, case dismissed. 1. WON the filing of a motion for reconsideration interrupts the twelve
months prescriptive period to contest the denial of the insurance claim.
2. The condition contained that claims must be presented within 12 months or 2. WON the rejection of the claim shall be deemed final only if it contains
one year after rejection is not merely a procedural requirement, but is essential words to the effect that denial is final. (ie. the first letter in 1984)
to a prompt settlement of claims against insurance companies. 3. When does the cause of action accrue? 

3. It demands that insurance suits be brought by the insured while the evidence as Held:
to the origins and causes of destruction have not yet disappeared. 1.No
2.No
4. Its purpose is to terminate all liabilities in case the action is not filed by the 3. At the time of the first rejection of the insurance company
insured
within the period stipulated. Ratio:
1. The policy states in section 27.
5. The action against the agent cannot have any other legal effect except that of Action or suit clause — If a claim be made and rejected and an action or
notifying the agent of the claim if there is no condition in the policy that an action suit be not commenced either in the Insurance Commission or in any
should be filed by the insured against the agent for his claim. There is no law court of competent jurisdiction within twelve (12) months from receipt
giving any effect to such action upon the principal, and courts cannot by of notice of such rejection, or in case of arbitration taking place as
interpretation extend the clear scope of the agreement beyond what is agreed provided herein, within twelve (12) months after due notice of the
upon by the parties. award made by the arbitrator or arbitrators or umpire, then the claim
shall for all purposes be deemed to have been abandoned and shall not
6. Contractual limitations in insurance policies prevail over the statutory limitations, thereafter be recoverable hereunder.
as well as over the exceptions to the latter, because the rights of the parties flow Respondent Tan  admitted that he received a copy of the letter of
from the contract of insurance. Their contract is the law between the parties, and rejection on April 2, 1984. Thus, the 12-month prescriptive period
their agreement that an action on a claim denied by the insurer must be brought started to run from the said date of April 2, 1984, under section 27.
within one year from the denial, governs, not the rules on the prescription of 2. It was clear in the letter.
actions. Ang v. Fulton Fire Insurance Co.- The condition contained in an
insurance policy that claims must be presented within one year after
Sun lnsurance CA, GR 89741; 13 March 1991 rejection is not merely a procedural requirement but an important
matter essential to a prompt settlement of claims against insurance
Facts: companies as it demands that insurance suits be brought by the insured

3 – EXECUTIVE | Ab Initio Group

74
INSURANCE MIDTERMS (Weeks 1 to 8)
while the evidence as to the origin and cause of destruction have not yet Pacific filed for sum of money against Oriental. Oriental alleged that Pacific
disappeared. prematurely filed a suit, for neither filing a formal claim over loss pursuant to
Therefore, there was a necessity of bringing suits against the Insurer policy nor submitting any proof of loss.
within one year from the rejection of the claim. (1984) The contention of
the respondents that the one-year prescriptive period does not start to Trial court decided in favor of Pacific. Decision based on technicality. The defense
run until the petition for reconsideration had been resolved by the of lack of proof of loss and defects were raised for the 1st time. (On presentation
insurer (1985), runs counter to the doctrine. of evidences by Pacific, it was revealed there was violation of Condition No.3,
The provision in the contract was pursuant to Sec. 63.    there were undeclared co-insurances under same property –Wellington, Empire,
A condition, stipulation or agreement in any policy of insurance, limiting Asian. The only declared co-insurances were Malayan, South Sea, and Victory)
the time for commencing an action thereunder to a period of less than
one year from the time when the cause of action accrues, is void. CA reversed decision. Concealment of other co-insurances is a misrepresentation
and can easily be fraud.
3. Eagle star- The right of the insured to the payment of his
loss accrues from the happening of the loss. However, the cause of Issues:
action in an insurance contract does not accrue until the insured's claim (1) Whether or not unrevealed con-insurances is a violation of Policy
is finally rejected by the insurer. This is because before such final Condition No.3
rejection there is no real necessity for bringing suit. (2) Whether or not there was premature filing of action
The cause of action, then, started when the insurer denied his claim in
the first instance(1984).  This rejection of a petition for reconsideration Held: (1) Yes. Policy Condition 3 provides that the insured must give notice of any
as insisted by respondents wasn’t the beginning of the cause of action. insurance already in effect or subsequently be in effect covering same property
being insured. Failure to do so, the policy shall be forfeited.
Failure to reveal before the loss of the 3 other insurances is a clear
misrepresentation or a false declaration. The material fact was asked for but was
Pacific Banking v CA, GR L-41014,28 November 1988 not revealed. Representations of facts are the foundations of the contract. Pacific
itself provided for the evidences in trial court that proved existence of
Facts: An open Fire Policy issued to Paramount Shirt Manufacturing for Php61,000 misrepresentation.
on the following: stocks, materils, supplies, furniture, fixture, machinery,
equipment contained on the 1st to 3rd floors. Insurance is for a year starting 21 (2) Yes. Policy Condition 11 is a sine qua non requirement for maintaining action.
OCTOBER 1964. It requires that documents necessary to prove and estimate the loss should be
included with notice of loss. Pacific failed to submit formal claim of loss with
Paramount Shirt is debtor of Pacific Banking amounting to Php800,000. Goods in supporting documents but shifted the burden to the insurance company. Failing to
policy were held in trust by Paramount for Pacific under thrust receipts. Fire broke submit claim is failure for insurance company to reject claim. Thus, a lack of cause
out on 4 January 1964. of action to file suit.

Pacific sent letter of demand to Oriental. Insurance Adjuster of Oriental notified Furthermore, the mortgage clause in the policy specifically provides that the policy
Pacific to submit proof of loss pursuant to Policy Condition 11. Pacific did not is invalidated by reasons of FRAUD, MISREPRESENTATION and FRAUD.
accede but asked Insurance Adjuster to verify records form Bureau of Customs. Concealment can easily be fraud or misrepresentation.

The insured – PARAMOUNT is not entitled to proceeds. Moreso, Pacific as indorsee


of policy is not entitled.
3 – EXECUTIVE | Ab Initio Group

75
INSURANCE MIDTERMS (Weeks 1 to 8)
-With this rejection, the plaintiff filed his complaint with the CFI of Manila on
September 19,1961.
Lopez v Cia de Seguros, GR L-19613, 30 April 1966
-Against the above complaint, the defendant-appellee filed on September 29, 1961
FACTS: a motion to dismiss on the ground of prescription. The latter argued that the
-Plaintiff applied with the defendant company for the insurance of his properties: plaintiff's claim had already prescribed since it was not filed within twelve months
Biederman truck tractor and a Winter Weils trailer from loss or damage in the from its rejection by the insurance company as stipulated under paragraph 9 of
amount of P20,000.00 and P10,000.00, respectively. the General Conditions of Commercial Vehicle Comprehensive Policy Nos. 5598 and
5599, to wit:
-During the application, the defendant company inquired of the plaintiff
the ff: If a claim be made and rejected and an action or suit be not commenced within
twelve months after such rejection or (in case of an arbitration taking place as
-Has any company in respect of the insurance of any car or vehicle provided herein) within twelve months after the arbitrator, arbitrators, or umpire
-(A) declined, cancelled or refused to renew your insurance shall have made their award then the claim shall for all purposes be deemed to
-(B) increased your premium renewal have been abandoned and shall not thereafter be recovered hereunder.
-Plaintiff answered in negative but the truth was that the American International
Underwriters of the Philippines (AIU) had already declined similar application for ISSUE:
insurance by the plaintiff with respect of the above-mentioned vehicles. Whether the complaint filed by the plaintiff-appellant with the Office of
the Insurance Comm. on May 27,1960 a commencement of an "action or
-The defendant issued to the plaintiff two Commercial Vehicle Comprehensive suit" within the meaning and intent of general condition? No.
Policies covering the said properties.

-The vehicles mentioned figured in an accident resulting in the total loss


of the tractor and partial damage to the trailer. Plaintiff demand upon the RATIO:
defendant for the payment to him the total amt. of damages resulting from the -"Action" and "suit":
accident. Rule 2, Section 1 of the Rules of Court
Section 1. Action defined.—Action means an ordinary suit in a Court of Justice by
-On April 28, 1960, defendant rejected the claim on the ground of concealment of which one party prosecutes another for the enforcement or protection of a right,
a material fact: that the insured property previously been declined insurance by or the prevention or redress of a wrong. (Emphasis supplied.)
another company.
Jurisprudence
-May 27, 1960, the plaintiff filed with the Office of the Insurance Commissioner a Suit is the prosecution or pursuit of some claim or demand in a court of justice or
complaint against the said company. any proceeding in a court of justice in which a plaintiff pursues his remedy to
recover a right or claim. (Emphasis supplied.)
-As suggested, the plaintiff was willing to submit his claim to arbitration but was
contested by the defendant since "the claim of the plaintiff cannot be resolved by - Upon the authorities, therefore, it is settled that the terms "action" and "suit" are
arbitration, as recourse to arbitration referred to in the policy contract, envisioned synonymous. Moreover, it is clear that the determinative or operative fact which
only differences or disputes, 'with respect to the amount of the company's liability,' converts a claim into an "action or suit" is the filing of the same with a "court or
and not to cases where the company does not admit its liability to the insured. justice." Filed elsewhere, as with some other body or office not a court of justice,
the claim may not properly be categorized under either term.
3 – EXECUTIVE | Ab Initio Group

76
INSURANCE MIDTERMS (Weeks 1 to 8)
expiration date shall be considered as if written for successive policy periods or
-An "action or suit" is essentially "for the enforcement or protection of a right, or terms of one year.
the prevention or redress of a wrong." (Rule 2, Sec. 1, Rules of Court). There is
nothing in the Insurance Law, which empowers the Insurance Commissioner to
adjudicate on disputes relating to an insurance company's liability to an insured Saura lmport & Export v Phil lnt'l Surety, GR L-15'184, 31 May 1963
under a policy issued by the former to the latter. The validity of an insured's claim
under a specific policy, its amount, and all such other matters as might involve the Saura mortgaged its property to PNB. Saura insured its mortgaged property to
interpretation and construction of the insurance policy, are issues which only a Philippine International Surety Co. from October 2, 1954 to October 2, 1955.
regular court of justice may resolve and settle. Consequently, the complaint filed However, on October 15, 1954 Philippine International Surety Co. cancelled the
by the appellant herein with the Office of the Insurance Commission could not insurance policy without informing the insured Saura. SC: If a mortgage or lien
have been an "action or suit." exists against the property insured, and the policy contains a clause stating that
loss, if any, shall be payable to such mortgagee or the holder of such lien as his
Sec. 64. No policy of insurance other than life shall be cancelled by the insurer interest may appear, notice of cancellation to the mortgagee or lienholder alone is
except upon prior notice thereof to the insured, and no notice of cancellation shall ineffective as a cancellation of the policy as to the owner of the property.
be effective unless it is based on the occurrence, after the effective date of the
policy, of one or more of the FACTS:
(a) non-payment of premium; -On Dec. 26, 1952, Saura mortgaged to PNB its registered parcel of land in Davao
(b) conviction of a crime arising out of acts increasing the hazard insured against; to secure the payment of a promissory note amounting to Php 27,000.
(c) discovery of fraud or material misrepresentation; -A building of strong materials which was also owned by Saura, was erected on
(d) discovery of willful or reckless acts or omissions increasing the hazard insured the parcel of land and the building had always been covered by insurance even
against; before the execution of the mortgage contract.
(e) physical changes in the property insured which result in the property becoming
uninsurable; or -Pursuant to the mortgage agreement which required Saura to insure the building
(f) a determination by the Commissioner that the continuation of the policy would and its contents, it obtained a fire insurance for Php 29,000 from PISC for a period
violate or would place the insurer in violation of this Code. of 1 year starting Oct. 2, 1954 to Oct. 2, 1955.

Sec. 65. All notices of cancellation mentioned in the preceding section shall be in -The mortgage also required Saura to endorse the insurance policy to PNB. The
writing, mailed or delivered to the named insured at the address shown in the memo stated: Loss if any, payable to PNG as their interest may appear, subject to
policy, and shall state (a) which of the grounds set forth in section sixty-four is the terms, conditions and warranties of this policy.
relied upon and (b) that, upon written request of the named insured, the insurer
will furnish the facts on which the cancellation is based. -The policy was delivered to PNB by Saura.

Sec. 66. In case of insurance other than life, unless the insurer at least forty-five -However on Oct. 15, 1954, barely 13 days after the issuance of the fire insurance,
days in advance of the end of the policy period mails or delivers to the named PISC canceled the same, effective as of the date of issue. Notice of the
insured at the address shown in the policy notice of its intention not to renew the cancellation was sent to PNB (but not to Saura) in writing and was received by the
policy or to condition its renewal upon reduction of limits or elimination of bank on Nov. 8, 1954.
coverages, the named insured shall be entitled to renew the policy upon payment
of the premium due on the effective date of the renewal.  Any policy written for a -On April 6, 1955, the building and its contents worth Php 4,685 were burned.
term of less than one year shall be considered as if written for a term of one year. 
Any policy written for a term longer than one year or any policy with no fixed -On April 11, 1985, Saura filed a claim with PISC and mortgagee bank.
3 – EXECUTIVE | Ab Initio Group

77
INSURANCE MIDTERMS (Weeks 1 to 8)
payment to MICO, together with other payments. On January 18, 1982, Pinca's
-Upon presentation of notice of loss with PNB, Saura learned for the first time that property was completely burned.
the policy had been previously canceled by PISC, when Saura’s folder in the bank’s
file was opened and the notice of the cancellation by PISC was found. DECISION OF LOWER COURTS: 
(1) Insurance Commission: granted claim for compensation for burned property. 
ISSUE:
W/N cancellation by PISC was proper.
ISSUE: 
HELD: Whether there was a valid insurance contract at the time of the loss.
No. SC ordered PISC to pay Saura in the amount of Php 29,000.
RULING: 
RATIO: Yes.
The policy in question does NOT provide for the notice of cancellation, its form or A valid cancellation must, therefore, require concurrence of the following
period. The Insurance Law does not likewise provide for such notice. This being conditions:
the case, it devolves upon the Court to apply the generally accepted principles of
insurance, regarding cancellation of the insurance policy by the insurer. (1) There must be prior notice of cancellation to the insured;
Actual notice of cancellation in a clear and unequivocal manner, preferably in
writing should be given by the insurer to the insured so that the latter might be (2) The notice must be based on the occurrence, after the effective date of the
given an opportunity to obtain other insurance for his own protection. The notice policy, of one or more of the grounds mentioned;
should be personal to the insurer and not to and/or through any unauthorized
person by the policy. Both the PSIC and the PNB failed, wittingly or unwittingly to (3) The notice must be
notify Saura of the cancellation made. (a) in writing,
(b) mailed, or delivered to the named insured,
The insurer contends that it gave notice to PNB as mortgagee of the property and (c) at the address shown in the policy;
that was already substantial compliance with its duty to notify the insured of the
cancellation of the policy. But notice to the bank, as far as Saura herein is (4) It must state
concerned, is not effective notice. PISC is then ordered to pay Saura Php 29,000, (a) which of the grounds mentioned in Section 64 is relied upon and
the amount involved in the policy subject matter of this case. (b) that upon written request of the insured, the insurer will furnish the facts on
which the cancellation is based. 

Malayan lnsurance v Cruz-Arnaldo, GR L-67835, 12 October 1987 MICO's claims it cancelled the policy in question on October 15, 1981, for non-
payment of premium. To support this assertion, it presented one of its employees,
FACTS:  who testified that "the original of the endorsement and credit memo" —
On June 7, 1981, the petitioner (hereinafter called (MICO) issued to the private presumably meaning the alleged cancellation — "were sent the assured by mail
respondent, Coronacion Pinca, Fire Insurance Policy No. F-001-17212 on her through our mailing section" However, there is no proof that the notice, assuming
property for the amount of P14,000.00 effective July 22, 1981, until July 22, 1982. it complied with the other requisites mentioned above, was actually mailed to and
On October 15,1981, MICO allegedly cancelled the policy for non-payment, of the received by Pinca.
premium and sent the corresponding notice to Pinca. We also look askance at the alleged cancellation, of which the insured and MICO's
On December 24, 1981, payment of the premium for Pinca was received by agent himself had no knowledge, and the curious fact that although Pinca's
Domingo Adora, agent of MICO. On January 15, 1982, Adora remitted this payment was remitted to MICO's by its agent on January 15, 1982, MICO sought
3 – EXECUTIVE | Ab Initio Group

78
INSURANCE MIDTERMS (Weeks 1 to 8)
to return it to Adora only on February 5, 1982, after it presumably had learned of
the occurrence of the loss insured against on January 18, 1982. These
circumstances make the motives of the petitioner highly suspect, to say the least,
and cast serious doubts upon its candor and bona fides. 

3 – EXECUTIVE | Ab Initio Group

79

You might also like